HESI CAT EXAM 700 QUESTIONS AND CORRECT ANSWERS - Learnexams (2024)

HESI CAT EXAM 700 QUESTIONS AND CORRECT ANSWERS

A nurse is counseling the spouse of a client who has a history of alcohol abuse. What does the nurse explain is the main reason for drinking alcohol in people with a long history of alcohol abuse?
1
They are dependent on it.
2
They lack the motivation to stop.
3
They use it for coping.
4
They enjoy the associated socialization.
1
Alcohol causes both physical and psychological dependence; the individual needs the alcohol to function. Alcoholism is a disorder that entails physical and psychological dependence. Because alcohol is so physiologically addictive, the client’s body craves the alcohol, so most clients lack the motivation to stop because they will go into withdrawal. Clients who abuse alcohol have numbed their ability to utilize other coping mechanisms, so alcohol is used as an excuse for coping. People with alcoholism usually drink alone or feel alone in a crowd; socialization is not the prime reason for their drinking.

How do adolescents establish family identity during psychosocial development? Select all that apply.
1
By acting independently to make his or her own decisions
2
By evaluating his or her own health with a feeling of well-being
3
By fostering his or her own development within a balanced family structure
4
By building close peer relationships to achieve acceptance in the society
5
By achieving marked physical changes
13
An adolescent establishes family identity by acting independently for taking important decisions about self. They also need to foster their development along with maintaining a balanced family structure. Health identity is associated with the evaluation of one’s own health with a feeling of well-being. By building close peer relationships, an adolescent develops a sense of belonging, approval, and the opportunity to learn acceptable behavior. These actions establish an adolescent’s group identity. The sound and healthy growth of the adolescent, with marked physical changes, helps to build an adolescent’s sexual identity.

We have an expert-written solution to this problem!
A clinic nurse observes a 2-year-old client sitting alone, rocking and staring at a small, shiny top that she is spinning. Later the father relates his concerns, stating, “She pushes me away. She doesn’t speak, and she only shows feelings when I take her top away. Is it something I’ve done?” What is the most therapeutic initial response by the nurse?
1
Asking the father about his relationship with his wife
2
Asking the father how he held the child when she was an infant
3
Telling the father that it is nothing he has done and sharing the nurse’s observations of the child
4
Telling the father not to be concerned and stressing that the child will outgrow this developmental phase
3
The nurse provides support in a nonjudgmental way by sharing information and observations about the child. This child exhibits symptoms of autism, which is not attributable to the actions of the parents. Asking the father about his relationship with his wife or how he held the child when she was an infant indirectly indicates that the parent may be at fault; it negates the father’s need for support and increases his sense of guilt. Telling the father not to be concerned and stressing that the child will outgrow this developmental phase is false reassurance that does not provide support; the father recognizes that something is wrong.

What is most appropriate for a nurse to say when interviewing a newly admitted depressed client whose thoughts are focused on feelings of worthlessness and failure?
1
“Tell me how you feel about yourself.”
2
“Tell me what has been bothering you.”
3
“Why do you feel so bad about yourself?”
4
“What can we do to help you while you’re here?”
1
Because major depression is a result of the client’s feelings of self-rejection, it is important for the nurse to have the client initially identify these feelings before developing a plan of care. Later discussion should be focused on other topics to prevent reinforcement of negative thoughts and feelings. “Tell me what has been bothering you” is asking the client to draw a conclusion; the client may be unable to do so at this time. Also, depression may be related not to external events but instead to a client’s psychobiology. Asking why does not let a client explore feelings; it usually elicits an “I don’t know” response. “What can we do to help you while you’re here?” is beyond the scope of the client’s abilities at this time.

We have an expert-written solution to this problem!
A client is admitted to the mental health unit with the diagnosis of major depressive disorder. Which statement alerts the nurse to the possibility of a suicide attempt?
1
“I don’t feel too good today.”
2
“I feel much better; today is a lovely day.”
3
“I feel a little better, but it probably won’t last.”
4
“I’m really tired today, so I’ll take things a little slower.”
2
A rapid mood upswing and psychom*otor change may signal that the client has made a decision and has developed a plan for suicide. “I don’t feel too good today”; “I feel a little better, but it probably won’t last”; and “I’m really tired today, so I’ll take things a little slower” are all typical of the depressed client; none of these statements signals a change in mood.

During a group discussion it is learned that a group member hid suicidal urges and committed suicide several days ago. What should the nurse leading the group be prepared to manage?
1
Guilt of the co-leaders for failing to anticipate and prevent the suicide
2
Guilt of group members because they could not prevent another’s suicide
3
Lack of concern over the suicide expressed by several of the members in the group
4
Fear by some members that their own suicidal urges may go unnoticed and that they may go unprotected
4
Ambivalence about life and death, plus the introspection commonly found in clients with emotional problems, can lead to increased anxiety and fear among the group members. These feelings must be handled within the support and supervisory systems for the staff; the group members are the primary concern. Guilt that the group’s leaders or members might feel because they could not prevent another’s suicide will probably be a secondary concern of the group leader. Lack of concern over the suicide expressed by several of the members in the group is not a primary concern, but this should be explored later to determine the reason for such apparent indifference, which may be a mask to cover true feelings.

Which screening report will help the nurse determine skeletal growth in a child?
1
Electroencephalogram reports
2
Radiographs of the hand and wrist
3
Magnetic resonance imaging (MRI)
4
Denver Developmental Screening Test
2
Skeletal growth in a child can be determined from the ossification centers. At 5 to 6 months of age, the capitate and hamate bones in the wrist are the earliest centers. Therefore radiographs of the hand and wrist will help determine skeletal growth in the child. Electroencephalogram reports will help assess a child’s brain activity. MRI is used to scan the internal structures of a client. The Denver Developmental Screening Test is used to understand developmental issues of a child.

A client describes his delusions in minute detail to the nurse. How should the nurse respond?
1
Changing the topic to reality-based events
2
Continuing to discuss the delusion with the client
3
Getting the client involved in a social project with peers
4
Disputing the perceptions with the use of logical thinking
1
Decreasing time spent on delusions prevents reinforcement of psychotic thinking. Discussing reality-based events improves contact with reality. Encouraging discussion will give validity to the delusion. The client will have difficulty getting involved in a social activity; the activity will not stop the delusion. Challenging the client may increase anxiety.

A nurse working on a mental health unit is caring for several clients who are at risk for suicide. Which client is at the greatest risk for successful suicide?
1
Young adult who is acutely psychotic
2
Adolescent who was recently sexually abused
3
Older single man just found to have pancreatic cancer
4
Middle-age woman experiencing dysfunctional grieving
3
Older single men with chronic health problems are at the highest risk of suicide. This is because men have fewer social supports than women do. (Men are less social then women in general.) Less social support at times of stress can increase the risk of suicide. Also, chronic health problems can lead to learned helplessness, which can lead to depression. People who are acutely psychotic as a group are at higher risk for suicide, but they do not have the suicide rate of older single adult men with chronic health problems. An adolescent who was recently sexually abused, although severely traumatized, does not have the risk of suicide of an older single man with chronic health problems. Dysfunctional grieving is prolonged grieving that is characterized by greater disability and dysfunctional patterns of behavior. Although people with complicated dysfunctional grieving may be at risk for self-directed violence, they do not have the suicide risk of older single men with chronic health problems.

Which stages would the nurse explain that a toddler goes through, according to Freud’s theory? Select all that apply.
1
Oral
2
Anal
3
Phallic
4
Genital
5
Latency
12
According to Freud’s theory, a toddler goes through the oral and anal stages. The phallic stage is seen in children between the ages of 3 to 6 years. The genital stage is seen during puberty through adulthood. The latency stage is seen in children ages 6 to 12 years of age.

A client is found to have a borderline personality disorder. What behavior does the nurse consider is most typical of these clients?
1
Inept
2
Eccentric
3
Impulsive
4
Dependent
3
Impulsive, potentially self-damaging behaviors are typical of clients with this personality disorder. Inept behavior, by itself, is not typical of clients with any specific personality disorder. Eccentric behavior is more typical of the client with a schizotypal personality disorder. Dependent behavior is more typical of the client with a dependent personality disorder.

An older adult, accompanied by family members, is admitted to a long-term care facility with symptoms of dementia. What initial statement by the nurse during the admission procedure would be most helpful to this client?
1
“You’re a little disoriented now, but don’t worry. You’ll be all right in a few days.”
2
“Don’t be afraid. I’m your nurse, and everyone here in the hospital is here to help you.”
3
“I’m the nurse on duty today. You’re in the hospital. Your family can stay with you for a while.”
4
“Let me introduce you to the staff here first. In a little while I’ll get you acquainted with our unit routine.”
2
Familiarity with the environment and a self-introduction may help promote security and feelings of trust. Telling the client “You’re a little disoriented now, but don’t worry. You’ll be all right in a few days” denies the client’s feelings and provides false reassurance. A self-introducing one’s self followed by telling the client that of being in the hospital and that the family may stay for a while denies the client’s feelings but does provide self-introduction and orientation regarding the client’s location. A person under stress cannot assimilate much information; verbiage could lead to more confusion.

Which identity may fail to develop if the adolescent fails to feel a sense of belonging and acceptance?
1
Sexual identity
2
Group identity
3
Family identity
4
Health identity
2
Failure to feel acceptance and belonging results in failure to establish a group identity. A lack of physical evidence of maturity can predispose the adolescent to fail to establish a sexual identity. Adolescents depend on these physical cues because they want assurance of maleness or femaleness and do not wish to be different from their peers. If an adolescent fails to foster independence and balance in the family structure, it may hamper family identity. Healthy adolescents evaluate their own health on the basis of feelings of well-being, ability to function normally, and absence of symptoms.

We have an expert-written solution to this problem!
In her eighth month of pregnancy, a 24-year-old client is brought to the hospital by the police, who were called when she barricaded herself in a ladies’ restroom of a restaurant. During admission the client shouts, “Don’t come near me! My stomach is filled with bombs, and I’ll blow up this place if anyone comes near me.” What does the nurse conclude that the client is exhibiting?
1
Ideas of reference
2
Loose associations
3
Delusional thinking
4
Tactile hallucinations
3
Delusions are false fixed beliefs that have a minimal basis in reality. This is a somatic delusion. Ideas of reference are false beliefs that every statement or action of others relates to the individual. Loose associations are verbalizations that sound disjointed to the listener. Tactile hallucinations are false sensory perceptions of touch without external stimuli.

Which should the nurse encourage for a school-age client diagnosed with a chronic illness to enhance a sense of accomplishment?
1
Wearing make-up
2
Making up missed work
3
Participating in sports activities
4
Participating in creative activities
2
Making up missed work is an activity the nurse can encourage to enhance a sense of accomplishment for a school-age client who is diagnosed with a chronic illness. Wearing make-up is often encouraged for an adolescent client. Participation in sports activities enhances the development of peer relationship in the school-age child. Participating in creative activities allows the school-age child to learn through concrete operations.

We have an expert-written solution to this problem!
A nurse is caring for a client exhibiting compulsive behaviors. The nurse concludes that the compulsive behavior usually incorporates the use of which defense mechanism?
1
Projection
2
Regression
3
Displacement
4
Rationalization
3
Displacement is the unconscious redirection of an emotion from a threatening source to a nonthreatening source. Projection is the attribution of one’s unacceptable feelings and thoughts to someone else. Regression is the return to an earlier, more comfortable level of behavior; it is a retreat from the present. Rationalization is the attempt to make unacceptable behavior or feelings acceptable by justifying the reasons for them.

A client is admitted for a biopsy of a tumor in her left breast. The client states, “I know it can’t be cancer, because it doesn’t hurt.” What is the nurse’s most therapeutic response?
1
“Let’s hope that it isn’t malignant.”
2
“What do you know about breast cancer?”
3
“Most lumps in the breast are not malignant.”
4
“Has your primary healthcare provider told you that it wasn’t cancer?”
2
Asking what the client knows about breast cancer allows the nurse to assess the client’s understanding of breast cancer and to clarify any misconceptions. Saying that they should hope that the growth isn’t malignant avoids an opportunity to teach, and it is a type of false reassurance. The statement may actually increase feelings of hopelessness if the lesion is determined to be malignant. Although correct, stating that most lesions are benign provides a false sense of security and avoids an opportunity to teach. Asking whether the primary healthcare provider has told the client that it wasn’t cancer focuses on what the primary healthcare provider said rather than on what the client knows and may limit further communication of feelings and beliefs.

A nurse in the emergency department is assessing a client who has been physically and sexually assaulted. What is the nurse’s priority during assessment?
1
The family’s feelings about the attack
2
The client’s feelings of social isolation
3
The client’s ability to cope with the situation
4
Disturbance in the client’s thought processes
3
The situation is so traumatic that the individual may be unable to use past coping behaviors to comprehend what has occurred. Assessing emotions that occur in response to news of the attack will occur later. The client should be the focus of care at this time. Social isolation is not an immediate concern. Coping skills, not thought processes, are challenged at this time.

We have an expert-written solution to this problem!
A client is admitted to the psychiatric unit with the diagnosis of obsessive-compulsive disorder. The client washes her hands more than 20 times a day, and they are raw and bloody. What defense mechanism does the nurse conclude that the client is using to ease anxiety?
1
Undoing
2
Projection
3
Introjection
4
Displacement
1
Undoing is an act that partially negates a previous one; the client is using this defense mechanism to atone for unacceptable acts or wishes. The client is not attributing self-thoughts or impulses to another person or group, which is called projection. The client is not absorbing into the self a hated or loved object (introjection). Displacement is the transferring of feelings from one person, object, or experience onto another, less threatening person, object, or experience.

The parents tell the nurse that their preschooler often awakes from sleep screaming in the middle of the night. The preschooler is not easily comforted and screams if the parents try to restrain the child. What does the nurse instruct the parents?
1
“Always read a story to the child before bedtime.”
2
“Intervene only if necessary to protect the child from injury.”
3
“Discuss counseling options with the primary health care provider.”
4
“Try to wake the child and ask the child to describe the dream.”
2
Waking up screaming from sleep at night indicates sleep terrors. The nurse should advise the parents to observe the child and intervene only if there is a risk for injury. Reading a story before bedtime helps to calm the child before sleeping, but it does not ensure that the child will not have a sleep terror. There is no need for professional counseling, because sleep terrors are a common phenomenon in preschool-age children. The child is not aware of anybody’s presence during a sleep terror, so it is not appropriate to wake up the child; this may cause the child to scream and thrash more.

We have an expert-written solution to this problem!
A client who was forced into early retirement is found to have severe depression. The client says, “I feel useless, and I’ve got nothing to do.” What is the best initial response by the nurse?
1
“Tell me more about feeling useless.”
2
“Volunteering can help you fill your time.”
3
“Your illness is adding to your current feelings.”
4
“Let’s talk about what you’d like to be doing right now.”
1
An open-ended response encourages further discussion and allows exploration of feelings. Telling the client that volunteering will help pass the time ignores the client’s feelings. The depression is not adding to the feelings; the feelings are causing the depression. Asking the client to talk about what the client would rather be doing ignores the client’s feelings.

What characteristic is most essential for the nurse caring for a client undergoing mental health care?
1
Empathy
2
Sympathy
3
Organization
4
Authoritarianism
1
Empathy—understanding and to some extent sharing the emotions of another—encourages the expression of feelings. Empathy is an essential tool in caring for emotionally ill clients. Sympathy, or feeling sorry for someone, may further decrease the client’s feelings of self-worth. Although organization may help the client accept limits and organize activities, it is not as important as empathy. An authoritarian approach will emphasize the client’s weak ego and lack of self-esteem.

We have an expert-written solution to this problem!
When visiting hours are over, a nurse approaches a client with paranoid schizophrenia, who shouts, “You’re the one that made my lover leave me.” What conclusion does the nurse make about the client?
1
The patient is disoriented.
2
The patient is actively hallucinating.
3
The patient feels a sense of vulnerability.
4
The patient needs to have limits set after calming down.
3
The client’s low self-esteem precipitates doubt of the lover’s feelings, creating a sense of vulnerability. This statement reflects the client’s low self-esteem, which is projected onto the nurse as part of the delusion. The client’s statements do not reflect disorientation but instead reflect false beliefs, which are common in clients with paranoid schizophrenia. The client’s statements do not represent hallucinations because they are not false sensory perceptions. Setting limits after the fact is not effective in any situation; limits must be set when the problem occurs.

We have an expert-written solution to this problem!
During a survey, the community nurse meets a client who has not visited a gynecologist after the birth of her second child. The client says that her mother or sister never had annual gynecologic examinations. Which factor is influencing the client’s health practice?
1
Spiritual belief
2
Family practices
3
Emotional factors
4
Cultural background
2
Family practices influence the client’s perception of the seriousness of diseases. The client does not feel the need to seek preventive care measures because no family member practices preventive care. The client is not influenced by spiritual beliefs in this instance. An individual’s spiritual beliefs and religious practices may restrict the use of certain forms of medical treatment. Emotional factors such as stress, depression, or fear may influence an individual’s health practice; however, this client does not show signs of being affected by emotional factors. The client is said to be influenced by cultural background if he or she follows certain beliefs about the causes of illness and uses customary practices to restore health.

A client tells the nurse, “A man is speaking to me from the corner of the room. Can you hear him?” How should the nurse respond?
1
“What’s he saying to you? Does it make any sense?”
2
“Yes, I hear him, but I can’t understand what he’s saying.”
3
“I don’t hear him. There’s no one in the corner of the room.”
4
“No, I don’t hear him, but is it making you uncomfortable to hear him?”
4
The statement “No, I don’t hear him, but is it making you uncomfortable to hear him?” points out reality, identifies potential feelings, and prevents the nurse from supporting the hallucination. Asking what the man is saying to the client and whether it makes any sense is nontherapeutic because it supports and focuses on the hallucination. “Yes, I hear him, but I can’t understand what he is saying” is nontherapeutic because it supports and focuses on the hallucination; also, it is not truthful. Although denying hearing the voice and pointing out that there is no one else in the room points out reality, this statement does not focus on the client’s feelings.

What is the priority nursing objective of the therapeutic psychiatric environment for a confused client?
1
Helping the client relate to others
2
Making the hospital atmosphere more homelike
3
Helping the client become accepted in a controlled setting
4
Maintaining the highest level of safe, independent function
4
The therapeutic milieu is directed toward helping the client develop effective ways of functioning safely and independently. Helping the client relate to others is one small part of the overall objectives. The therapeutic milieu allows some items from home to make the client less anxious; however, the objective is not to duplicate a home situation. Helping the client become accepted in a controlled setting is a worthwhile objective but not as important as working toward the maximal degree of safe, independent function.

Before an amniocentesis, both parents express anxiety about the fetus’s safety during the test. Which nursing intervention is most appropriate in promoting the parents’ ability to cope?
1
Initiating a parent-primary healthcare provider conference
2
Reassuring them that the procedure is safe
3
Explaining the procedure, step by step
4
Arranging for the father to be present during the test
3
Giving the parents information about what to expect during the procedure will help allay their fears and encourage their cooperation. The nurse should be able to provide information and interpretation of procedures for clients; a delay in answering questions may increase a client’s concerns. Amniocentesis is a low-risk procedure; however, some complications may occur. If the father is uninformed, viewing the procedure may increase his anxiety, even though his presence may be comforting to the mother.

A young client who has just lost her first job comes to the mental health clinic very upset and says, “I just start crying without any reason and without any warning.” How should the nurse respond initially?
1
“Do you know what makes you cry?”
2
“Most of us need to cry from time to time.”
3
“Crying unexpectedly can be very upsetting.”
4
“Are you having any other problems at this time?”
3
The response “Crying unexpectedly can be very upsetting” identifies the client’s feelings. Asking, “Do you know what makes you cry?” is an unrealistic question; the cause of anxiety may not be known. “Most of us need to cry from time to time” moves the focus away from the client. “Are you having any other problems at this time?” disregards the client’s comment; it is a direct question that may impede communication.

We have an expert-written solution to this problem!
A client is admitted to the hospital with the diagnosis of severe anxiety. What should the nurse’s plan of care for a client with an anxiety disorder include?
1
Promoting the suppression of anger by the client
2
Supporting the verbalization of feelings by the client
3
Encouraging the client to limit anxiety-related behaviors
4
Restricting the involvement of the client’s family during the acute phase
2
Freedom to ventilate feelings serves as a safety valve to reduce anxiety. The suppression of anger may increase the client’s anxiety. Encouraging the client to limit anxiety-related behaviors is not therapeutic; it may increase the anxiety that the client is feeling. Restricting the involvement of the client’s family during the acute phase may or may not be helpful; the client’s family may provide support to the client.

Windows in the recreation room of the adolescent psychiatric unit have been broken on numerous occasions. After a group discussion one of the adolescents confides that another adolescent client broke them. What should the nurse do when using an assertive intervention instead of aggressive confrontation?
1
Confront the adolescent openly in the group, using a controlled voice and maintaining direct eye contact.
2
Knock on the door of the adolescent’s room and ask whether the adolescent would come out to talk about the situation.
3
Approach the adolescent when the client is alone and, after making direct eye contact, inquire about the involvement in these incidents.
4
Use a trusting approach toward the adolescent and imply that the staff doubts the adolescent’s involvement but requests a denial for the record.
3
A private confrontation with presentation of reported facts allows verification; a calm, direct manner is most assertive. Confronting the adolescent openly in the group, using a controlled voice and maintaining direct eye contact, is aggressive confrontation, not assertive intervention. Knocking on the door of the adolescent’s room and asking whether the adolescent would come out to talk about the situation places control in the hands of the client rather than the nurse, and this may lead to aggressive confrontation. Using a trusting approach toward the adolescent and implying that the staff doubts the adolescent’s involvement but requests a denial for the record is not assertive intervention; it is manipulation and is not truthful.

A 6-year-old child with autism is nonverbal and makes limited eye contact. What should the nurse do initially to promote social interaction?
1
Encourage the child to sing songs with the nurse.
2
Engage in parallel play while sitting next to the child.
3
Provide opportunities for the child to play with other children.
4
Use therapeutic holding when the child does not respond to verbal interactions.
2
Entering the child’s world in a nonthreatening way helps promote trust and eventual interaction with the nurse. Using therapeutic holding may be necessary when a child initiates self-mutilating behaviors. Singing songs with the child participating or providing opportunities for the child to play with other children is unrealistic at this time; playing with others is a long-term objective.

What is an important aspect of nursing care for a client exhibiting psychotic patterns of thinking and behavior?
1
Helping keep the client oriented to reality
2
Involving the client in activities throughout the day
3
Helping the client understand that it is harmful to withdraw from situations
4
Encouraging the client to discuss why interacting with other people is being avoided
1
Keeping the withdrawn client oriented to reality prevents further withdrawal into a private world. A gradual involvement in selected activities is best. Helping the client understand that it is harmful to withdraw from situations is futile at this time. The psychotic client is unable to tell anyone the reason for avoiding interaction with others.

We have an expert-written solution to this problem!
A nurse is volunteering on the community crisis hotline. What is the final objective of the counseling process?
1
Reducing anxiety
2
Exploring feelings
3
Developing constructive coping skills
4
Accomplishing the debriefing process
3
Past coping behaviors have been inadequate in resolving the current crisis; new coping skills are needed to manage anxiety-producing conflicts. Reduction of anxiety is an early objective. Exploration of feelings is an immediate objective. Accomplishment of the debriefing process is an early objective.

An infant is born with a bilateral cleft palate. Plans are made to begin reconstruction immediately. What nursing intervention should be included to promote parent-infant attachment?
1
Demonstrating positive acceptance of the infant
2
Placing the infant in a nursery away from view of the general public
3
Explaining to the parents that the infant will look normal after the surgery
4
Encouraging the parents to limit contact with the infant until after the surgery
1
By demonstrating acceptance of the infant, without regard for the defect, the nurse acts as a role model for the parents, thereby encouraging their acceptance. Infants with cleft palates can remain in the newborn nursery; they should not be hidden. Telling the parents that the child will look normal after surgery is false reassurance; it does not promote parent-infant attachment behaviors. Encouraging the parents to limit contact will delay attachment; the parents should be encouraged to have frequent contact with their infant.

A nurse is working with a client experiencing a major depressive episode. What is a long-term outcome for this client?
1
Talking openly about the depressed feelings
2
Identifying and using new defense mechanisms
3
Discussing the unconscious source of the anger
4
Verbalizing realistic perceptions of self and others
4
A major part of depression involves an inability to accept the self as is, which leads to making demands on others to meet unrealistic needs. Talking about the client’s depressed feelings is a short-term goal; looking at what is causing those feelings is a long-term goal. Developing new defense mechanisms is not the priority, because they tend to help the client avoid reality. Discussing the unconscious source of the anger is not important or crucial to the client’s recovery.

The nurse finds a client with schizophrenia lying under a bench in the hall. The client says, “God told me to lie here.” What is the best response by the nurse?
1
“I didn’t hear anyone talking; come with me to your room.”
2
“What you heard was in your head; it was your imagination.”
3
“Come to the dayroom and watch television; you’ll feel better.”
4
“God wouldn’t tell you to lie there in the hall. God wants you to behave reasonably.”
1
The nurse is focusing on reality and trying to distract and refocus the client’s attention. “What you heard was in your head; it was your imagination” is too blunt and belittling; this approach rarely is effective. “Come to the dayroom and watch television; you’ll feel better” is false reassurance; the nurse does not know that the client will feel better. “God wouldn’t tell you to lie in the hall; God wants you to behave reasonably” may be interpreted as belittling or an attempt to convince the client that the behavior is irrational, which is usually ineffective.

We have an expert-written solution to this problem!
A nurse is obtaining a health history from a client who is known to be verbally abusive. The client tells the nurse, “You’re ugly, and you’re probably stupid, too. Why am I stuck with you as my nurse?” What is the best response by the nurse?
1
“It doesn’t matter what you think, because I know I’m a capable nurse.”
2
“Tell me more about why my caring for you today is so upsetting to you.”
3
“If you like, I will arrange to switch assignments so you can have another nurse.”
4
“You are talking inappropriately, so I’m going to leave and will come back when you stop being verbally abusive.”
4
The response “You are talking inappropriately, so I’m going to leave and will come back when you stop being verbally abusive” provides specific realistic feedback without rejecting the client. The reply “It doesn’t matter what you think, because I know I’m a capable nurse” is defensive and insulting to the client. The reply “Tell me more about why my caring for you today is so upsetting to you” will most likely encourage more inappropriate communication. The client’s behavior is the issue, and switching assignments does not address this. The client may view a change of nurse as rejection.

A nurse anticipates that most clients with phobias will use which defense mechanisms?
1
Dissociation and denial
2
Introjection and sublimation
3
Projection and displacement
4
Substitution and reaction formation
3
Clients with phobias cope with anxiety by placing it on specific persons, objects, or situations through displacement, projection, or both. The person with a phobia recognizes and admits the exaggerated fear as a real part of the self and does not deny it. Neither introjection, whereby a person internalizes and incorporates the traits of another, nor sublimation, whereby socially acceptable behavior is substituted for unacceptable instincts, is related to phobic activity. A less valued object is not substituted for one more highly valued (substitution), nor are the expressed feelings opposite of the experienced feelings of fear (reaction formation).

A depressed client cries when the family does not visit. What is the most therapeutic response by the nurse?
1
“It’s difficult to realize that no one cares about you.”
2
“Your family didn’t visit, and now you’re feeling rejected.”
3
“It’s terrible to have such negative thoughts about yourself.”
4
“Your family members work—that’s why they don’t visit you.”
2
The statement “Your family didn’t visit, and now you’re feeling rejected” accurately reflects the client’s emotions and may encourage exploration of feelings. The nurse does not know that no one cares about the client, and the statement may increase the client’s unhappiness. The client is upset about the lack of visitors; discussing negative self-thoughts changes the subject. The defensive statement “Your family members work—that’s why they don’t visit you” may worsen the client’s self-derogatory feelings.

A nurse is caring for a client with the diagnosis of bulimia nervosa. What does the nurse understand to be the function of food for individuals with bulimia?
1
Gain attention
2
Control others
3
Avoid growing up
4
Meet emotional needs
4
Clients with bulimia [1] [2] eat to blunt emotional pain because they frequently feel unloved, inadequate, or unworthy; purging is precipitated to relieve feelings of guilt for bingeing or out of fear of obesity. The bingeing and purging are usually done alone and in secret. Clients with bulimia often feel out of control and perform their behaviors in secret. A protest against growing up is one of the psychodynamic theories regarding anorexia nervosa, not to bulimia nervosa.

A 56-year-old man is admitted to the inpatient unit after family members report that he seems to be experiencing auditory hallucinations. The man has a history of schizophrenia and has had several previous admissions. Which statement indicates to the nurse that the client is experiencing auditory hallucinations?
1
“Get these horrible snakes out of my room!”
2
“I am not the devil! Stop calling me those names!”
3
“The food on this plate has poison in it, so take it away—I won’t eat it.”
4
“I did see an alien spaceship last night outside in my yard, and I’ve felt worse ever since.”
2
The client is responding to messages that he is hearing, which are auditory hallucinations. The responses regarding the snakes and the spaceship are examples of visual hallucinations because they describe what the client sees. The accusation of poisoning is the statement of a client who is suspicious and paranoid but not hallucinating.

What statements about culturally congruent care by the student nurse are correct? Select all that apply.
1
“It is the main goal of transcultural nursing.”
2
“It is provided through cultural competence.”
3
“It is provided in accordance with set criteria.”
4
“It is bound to the professional health care system.”
5
“It depends on the patterns and needs of an individual.”
125
Culturally congruent care is tailored to the needs of people themselves, not delivered in accordance with predetermined criteria. This care may be different from the values and meanings of the professional health care system. The main goal of transcultural nursing is to provide culturally congruent care. Cultural competence is applied to ensure the delivery of this care. Culturally congruent care is provided in accordance with people’s life patterns, values, and beliefs.

A hospitalized psychiatric client with the diagnosis of histrionic personality disorder demands a sleeping pill before going to bed. After being refused the sleeping pill, the client throws a book at the nurse. What does the nurse recognize this behavior to be?
1
Exploitive
2
Acting out
3
Manipulative
4
Reaction formation
2
Acting out is the process of expressing feelings behaviorally. The action is not exploitive, because no evidence is provided to demonstrate that anyone has been used to get what the client wants. The action is not manipulative, because no evidence is provided to demonstrate that anyone has been influenced against his or her wishes. The action is not disguising unacceptable feelings by expressing opposite emotions (reaction formation).

A woman who is frequently physically abused tells the nurse in the emergency department that it is her fault that her husband beats her. What is the most therapeutic response by the nurse?
1
“Maybe it was your husband’s fault, too.”
2
“I can’t agree with that—no one should be beaten.”
3
“Tell me why you believe that you deserve to be beaten.”
4
“You say that it was your fault—help me understand that.”
4
Paraphrasing and clarifying are interviewing techniques that promote communication between the nurse and client and help the client hear and explore her words and gain insight into her behavior. “Maybe it was your husband’s fault, too” is a declarative statement that is closed, will limit dialog, and is not therapeutic. When the nurse voices her opinion saying, “I can’t agree with that—no one should be beaten”, the nurse is shutting off communication with the client. Nurses are to be nonjudgmental and not offer an opinion, and should ask open-ended questions to facilitate communication with the client. Asking a “why” question is generally not therapeutic because most clients cannot respond to these questions with logical explanations.

After counseling an older widowed client, a nurse concludes that the grieving process has been successfully completed when the client does what?
1
Is able to plan to start new relationships
2
Talks about the deceased spouse at great length
3
Ignores the deceased spouse’s less-than-perfect qualities
4
Decides to leave the deceased spouse’s study as it was before the death
1
A healthy resolution helps the person move away from the old, safe, familiar relationship to establish new ones. Talking about the deceased spouse at great length is termed obsessional review; the mourner can talk of nothing else but the deceased and events surrounding the death. A reduction in obsessional review is a healthy sign. With a positive outcome to the grieving process, the mourner is able to see and accept the dead person’s negative and positive qualities. Leaving the deceased spouse’s study as it was before the death is an example of mummification, a pathological outcome to the grieving process.

The mental health nurse is facilitating a therapy group. How can the nurse further develop trust among the members of the group?
1
By discussing the importance of their trusting one another
2
By revealing some personal data as an example of trusting behavior
3
By having group members reveal some personal information about themselves
4
By reminding group members about the need for confidentiality within the group
4
Members must feel comfortable discussing things in the group; there must be an understanding that what is discussed in the group will remain in the group. Talking about trust does little to foster it. Revealing personal data about one’s self or having group members reveal information will not establish trust and may increase anxiety because the members may feel that their turn for exposure will come whether they want it or not.

According to Kübler-Ross, during which stage of grieving are individuals with serious health problems most likely to seek other medical opinions?
1
Anger
2
Denial
3
Bargaining
4
Depression
2
Denial includes feelings that the healthcare provider has made a mistake, so the client seeks additional opinions. Anger follows denial; behavior will be hostile and critical. Bargaining occurs after anger; the client verbally or secretly may promise something in return for wellness or a prolonged life. Depression occurs after bargaining; the client feels sadness and despair and may be withdrawn.

We have an expert-written solution to this problem!
Which parental statement indicates the need for further education regarding the psychosocial development that occurs during infancy?
1
“My older kids are so excited that our 10-month-old can play hide-and-seek with them.”
2
“Peek-a-boo is an appropriate activity to initiate with my baby around 9 months of age.”
3
“I just bought my 6-month-old some new rattles to play with because they are easy to grasp.”
4
“It is important that my baby develops trust so we always respond when he cries for us at night.”
1
Play is an important aspect of an infant’s psychosocial development. While more complex interactive games, such as hide-and-seek involving objects is expected by 1-year of age, this statement would require additional education regarding age-appropriate play for the infant. Peek-a-boo and rattles are both appropriate play for the infant. Responding promptly to an infant’s cry will establish trust.

A hospice nurse is caring for a dying client and the client’s family members during the developing awareness stage of grief. What is the most important thing about the family that the nurse should assess before providing care?
1
Cohesiveness
2
Educational level
3
Cultural background
4
Socioeconomic status
3
During the developing awareness stage of grief the degree of anguish experienced or expressed is influenced by the cultural background of the individual and family. Although cohesiveness does enter into the grief process, it is not as important in the developing awareness stage as cultural background is. Educational level has no relationship to the grieving process. Socioeconomic status is not a defining factor in how a family will respond to the loss of a loved one.

A client’s severe anxiety and panic are often considered “contagious.” What action should be taken when a nurse’s personal feelings of anxiety are increasing?
1
Refocusing the conversation to more pleasant topics
2
Saying to the client, “Calm down. You’re making me anxious, too.”
3
Saying, “Another staff member is coming in. I’ll leave and come back later.”
4
Remaining quiet so personal feelings of anxiety do not become apparent to the client
3
The nurse who is anxious should leave the situation after ensuring continuity of care; the client will be aware of the nurse’s anxiety, and the nurse’s presence will be nonproductive and nontherapeutic. The client will probably sense the nurse’s anxiety through nonverbal channels, if not through verbal responses. Refocusing and asking the client to calm down both meet the nurse’s need; this response may make the client feel guilty that something was said that upset the nurse. The client will be aware of the nurse’s anxiety, which will increase the client’s own anxiety.

A client undergoes dilation and curettage (D & C) after an early miscarriage (spontaneous abortion). The nurse finds her crying later in the day. What is the most appropriate statement by the nurse at this time?
1
“This must be a very difficult experience for you to deal with.”
2
“You’ll have other children to take the place of the one you lost.”
3
“Of course you’re sad now, but at least you know you can get pregnant.”
4
“I know how you feel, but when a woman miscarries, it’s usually for the best.”
1
Saying that this must be a difficult experience acknowledges the validity of the client’s grief and provides the client an opportunity to talk if she wishes. Other children cannot and should not be substituted for a lost fetus. Getting pregnant is not the issue; this statement belittles the lost fetus. The nurse cannot know how the client feels. Stating that a miscarriage is for the best is patronizing and diminishes the significance of the lost fetus.

A 15-year-old client tearfully states that her father has been sexually abusing her for the past 8 years. What statement should the nurse initially respond with?
1
“Which type of incidents preceded the abuse?”
2
“Sharing this information is a positive step in getting help.”
3
“I have to report this to child protective services right now.”
4
“What kinds of things does he do to you when he abuses you?”
2
“Sharing this information is a positive step in getting help” is an emotionally supportive response; it demonstrates that sharing this information is acceptable and provides hope that the client will get help. The client needs support, and asking what incidents preceded the abuse may precipitate or increase feelings of guilt. Telling the client that the abuse must be reported immediately to child protective services is not a priority at this time and may interfere with future sharing; the client needs immediate emotional support. Asking what the father did as part of the abuse implies that the client does not know what she is talking about; the client needs support, whether the abuse is real or imagined.

During the course of treatment a toddler is to receive an intramuscular injection. What is the priority nursing intervention that should be included in the plan of care to comfort the child?
1
Distracting the toddler’s attention with a toy car
2
Telling the parents exactly what will be done to the toddler
3
Giving the toddler the choice of having the injection now or later
4
Involving the parents in comforting the toddler after the injection
4
The parents are the most significant people in the young child’s life, and their involvement in comforting the child is the most supportive intervention for the toddler. Distraction does not provide an outlet for the toddler’s feelings. Explaining the procedure to the parents does not comfort the child. Offering choices for the toddler is incorrect because this type of choice is not a viable option; the medication must be administered as prescribed.

A preterm newborn is admitted to the neonatal intensive care unit (NICU). Which concern is most commonly expressed by NICU parents?
1
Fear of handling the infant
2
Delayed ability to bond with the infant
3
Prolonged hospital stay needed by the infant
4
Inability to provide breast milk for the infant
1
Because these infants are so tiny and frail, parents most commonly fear handling or touching them; they should be encouraged to do so by the NICU staff. The primary concern is the infant’s fragility, not bonding; however, bonding should be encouraged. Although there may be concerns about a long hospital stay, they are not commonly expressed by mothers. The primary concern is the infant’s fragility, not breast-feeding. Breasts may be pumped and breast milk given in gavage feedings.

A father asks a nurse for strategies to convince his 5-year-old to wear a helmet while bicycling. What should the nurse suggest to the father?
1
“You should forbid your child from riding a bicycle.”
2
“You should wear your helmet while riding your bicycle.”
3
“You should limit your child’s bicycling to a defined area.”
4
“You should tell your child about the risks associated with not wearing a helmet.”
2
The nurse should suggest the father to wear a helmet when he rides his bicycle. This sets a positive example for the child to wear his or her helmet. Restricting the child will interfere with the cognitive development. Limiting the child’s bicycling to a defined area may lead to conflicts between the father and the child. Telling the child about the risks of not wearing a helmet may interfere with the child’s willingness to ride.

After a difficult labor a client gives birth to a 9-lb (4 kg) boy who expires shortly afterward. That evening the client tearfully describes to the nurse her projected image of her son and what his future might have been. What is the nurse’s most therapeutic response?
1
“I guess you wanted a son very much.”
2
“It must be difficult to think of him now.”
3
“I’m sure he would have been a wonderful child.”
4
“If you dwell on this now, your grief will be harder to bear.”
2
Stating that it must be difficult to think of him now demonstrates empathy; the nurse is attempting to show understanding of the client’s feelings. Stating that the patient must have wanted a son very much is nontherapeutic; the nurse has no way of knowing this. Stating the certainty that the infant would have been a wonderful child switches the focus away from the client, whose needs should be met at this time. Stating that dwelling on the death will make her grief harder to bear denies the client’s feelings and implies that the client should curb painful emotions.

At 1 am a 28-month-old toddler is admitted to the pediatric unit with suspected meningitis. At 3 am, after the child is settled in, the mother tells the nurse, “I have to leave now, but whenever I try to go my child gets upset and then I start to cry.” What is the best action by the nurse?
1
Walking the mother to the elevator
2
Encouraging the mother to spend the night
3
Staying with the child while the mother leaves
4
Telling the mother to wait until the child falls asleep
3
Staying with the child enables the mother to leave and reassures her and the child that someone will be with and comfort the child. The mother has indicated that she is upset when the child is upset; walking the mother to the elevator meets neither the mother’s nor the child’s needs. The mother has said she must leave; trying to persuade her to stay will make her feel guilty about having to leave. Telling the mother to leave after the child is asleep is a dishonest solution; the child should be aware that the mother is leaving and reassured in terms that a toddler will understand that she will return.

Which intervention does the nurse implement to develop a caring relationship with the client’s family?
1
Deciding healthcare options for the client
2
Identifying the client’s family members and their roles
3
Declining to inform the client’s family after performing a procedure
4
Refraining from discussing the client’s health with the family
2
The family is an important resource in client care. Therefore, the nurse must first identify the client’s family members and their roles in the client’s life. This action helps the nurse to determine his or her contribution towards the client’s healthcare. The nurse should not decide healthcare options for the client. The nurse should inform the client and family about a procedure and take permission before implementing the procedure. The nurse ensures the client’s well-being by accepting the client as an active partner in healthcare.

We have an expert-written solution to this problem!
A client is admitted to a psychiatric hospital with the diagnosis of schizoid personality disorder. Which initial nursing intervention is a priority for this client?
1
Helping the client enter into group recreational activities
2
Convincing the client that the hospital staff is trying to help
3
Helping the client learn to trust the staff through selected experiences
4
Limiting the client’s contact with others while in the hospital
3
Demonstrating that the staff can be trusted is a vital initial step in the therapy program. The client is not ready to enter group activities yet and will not be until trust is established. Even proof will not convince the client with a schizoid personality that feelings of distrust are false. Arranging the client’s contact with others is not realistic even if it is possible; limiting contact with other clients will not enhance trust.

During the first session of a therapy group, one of the clients asks, “What’s supposed to happen in this group?” What is the most appropriate response by the nurse facilitator?
1
“First I’d like for you to tell me what you want to happen.”
2
“This is your group, and your participation will largely determine what happens.”
3
“The purpose of this group is to examine the way each of you interacts with the others.”
4
“You and the others are supposed to discuss any reality-based concerns you have about your illness.”
1
To achieve the greatest therapeutic value from a group session, the members must be involved in deciding what will be discussed. By telling the clients that this is their group, the nurse facilitator abdicates the leadership role and places responsibility for the success of the group entirely on its members. The responses “The purpose of this group is to examine the way each of you interacts with the others” and “You and the others are supposed to discuss any reality-based concerns you have about your illness” present a structured view of the purpose of a therapy group; the members must be involved in selection of the topics to be discussed.

A client with mild preeclampsia is instructed to rest at home. She asks the nurse, “What do you mean by rest?” What is the most appropriate response?
1
“Tell me what you consider rest.”
2
“Take three or four naps a day.”
3
“Stay off your feet as much as possible.”
4
“Would you like to know what I think it means?”
1
Responding by asking what the client considers rest reflects the client’s statement and permits clarification, which will yield information that can be used in planning. Recommending three or four naps each day is too specific an interpretation of a rest requirement; there is more to maintaining rest than naps. There is also more to maintaining rest than staying off one’s feet; this response is a vague interpretation of a rest requirement. What the nurse thinks rest means does not provide a clear picture of what the client interprets as rest.

A client reports fever, headache, extreme tiredness, dry cough, sore throat, runny nose, muscle aches, nausea, vomiting, and diarrhea. Which organism is responsible for this condition?
1
Influenza virus
2
Toxoplasma gondii
3
Human herpes virus-8
4
Cryptosporidium muris
1
Fever, headache, extreme tiredness, dry cough, sore throat, runny nose, muscle aches, nausea, vomiting, and diarrhea are symptoms of influenza. Influenza is caused by the Influenza virus. Toxoplasma gondii causes fever, altered mental status, headache, and seizures. Human herpes virus-8 causes vascular lesions on the skin. Cryptosporidium muris causes watery diarrhea and weight loss.

A nurse is caring for a client with hyperthyroidism. Which laboratory test will be most beneficial in monitoring the effectiveness of drug therapy?
1
Free thyroxine (FT4)
2
Thyroxine (T4), total
3
Free triiodothyronine (FT3)
4
Triiodothyronine (T3), total
2
The thyroxine (T4) total study is the best method of monitoring thyroid therapy. A free thyroxine (FT4) study measures the active component of total T4; this test is an indicator of thyroid function. Free triiodothyronine (FT3) measures the active component of triiodothyronine (T3) total. Total T3 helps to diagnose hyperthyroidism when T4 levels are normal.

A client’s relative asks the nurse what a cataract is. Which explanation should the nurse provide?
1
An opacity of the lens
2
A thin film over the cornea
3
A crystallization of the pupil
4
An increase in the density of the conjunctiva
1
A cataract [1] [2] is a clouding (opacity) of the crystalline lens or its capsule. A thin film over the cornea, a crystallization of the pupil, and an increase in the density of the conjunctiva are not the pathophysiology related to cataracts.

When assessing an 85-year-old client’s vital signs, the nurse anticipates a number of changes in cardiac output that result from the aging process. Which finding is consistent with a pathologic condition rather than the aging process?
1
A pulse rate irregularity
2
Equal apical and radial pulse rates
3
A pulse rate of 60 beats per minute
4
An apical rate obtainable at the fifth intercostal space and midclavicular line
1
Dysrhythmias are abnormal and are associated with acute or chronic pathologic conditions. An equal apical and radial pulse is expected; the radial pulse reflects ventricular contractions. The expected range in adults is 60 to 100 beats per minute. An apical rate obtainable at the fifth intercostal space and midclavicular line are the anatomical landmarks for locating the apex of the heart; they are unaffected by aging.

We have an expert-written solution to this problem!
A nurse notices a firm, edematous, irregularly shaped skin lesion on a client who reports an insect bite. Which skin lesion is this?
1
Wheal
2
Plaque
3
Vesicle
4
Pustule
1
A wheal is a firm, edematous, irregularly shaped skin lesion, formed as an inflammatory response to an allergen or insect bite. A plaque is a circ*mscribed, elevated, superficial lesion, like psoriasis. A vesicle is a circ*mscribed, superficial collection of serous fluid. A pustule is an elevated, superficial lesion filled with purulent fluid.

A pregnant woman who is in the third trimester arrives in the emergency department with vagin*l bleeding. She states that she snorted cocaine approximately 2 hours ago. Which complication does the nurse suspect as the cause of the bleeding?
1
Placenta previa
2
Tubal pregnancy
3
Abruptio placentae
4
Spontaneous abortion
3
Abruptio placentae is associated with cocaine use; it occurs in the third trimester. Placenta previa is seen in the third trimester; however, it is not associated with cocaine use. A tubal pregnancy is identified in the first trimester. Spontaneous abortion occurs in the first two trimesters.

The nurse is assessing a term newborn. Which sign should the nurse report to the pediatric primary healthcare provider?
1
Temperature of 97.7° F (36.5° C)
2
Pale-pink to rust-colored stain in the diaper
3
Heart rate that decreases to 115 beats/min
4
Breathing pattern with recurrent sternal retractions
4
A breathing pattern with recurrent sternal retractions is indicative of respiratory distress; the expected pattern is abdominal with synchronous chest movement. A temperature of 97.7° F (36.5° C) is within the expected range of 97.6° F (36.4° C) to 99° F (37.2° C) for a newborn. Pale-pink to rust-colored staining in the diaper is caused by uric acid crystals from the immature kidneys; it is a common occurrence. A decrease in heart rate to 115 beats/min is within the expected range of 110 to 160 beats/min for a newborn.

Which type of burn/injury may cause a client to have a cervical spine injury?
1
Electrical burns
2
Chemical burns
3
Inhalation injury
4
Cold thermal injury
1
Electrical burns may cause injuries to the cervical spine because intense electrical currents can fracture long bones and vertebrae. Chemical burns may cause eye and tissue damage. Inhalation injuries may damage the respiratory tract. Cold thermal injuries may cause tissue damage.

We have an expert-written solution to this problem!
A client is noted to have thickened toenails that overhang the toes. The registered nurse suspects a fungal infection and instructs the student nurse to examine the fungal infection to confirm the diagnosis. Which action of the student nurse needs correction?
1
Cutting the client’s fingernails straight across
2
Using the client’s fingernails for assessing capillary refill
3
Using the nail appearance alone for assessing fungal infection
4
Assessing skin next to the nail to determine whether the thick nail is irritating the skin
3
The nurse should not consider the nail appearance alone for assessing a fungal infection but should assess the toenails and the underlying skin to confirm the diagnosis. The nurse should cut the thickened toenails straight across to prevent discomfort. Fingernails are used for assessing capillary refill. The nurse should assess the skin next to the nail to determine the irritation caused by the thickened nail.

A primary healthcare provider diagnoses late-stage (tertiary) syphilis in a client. Which statement made by the client supports this diagnosis?
1
“I noticed a wart on my penis.”
2
“I have sores all over my mouth.”
3
“I’ve been having a sore throat lately.”
4
“I’m having trouble keeping my balance.”
4
Neurotoxicity, as manifested by ataxia (balance problems), is evidence of tertiary syphilis, which may involve the central nervous system (CNS) or cardiovascular system. A wart on the penis occurs in the secondary stage of syphilis. Sores all over the mouth occur in the first and secondary stage of syphilis. Sore throat with flulike symptoms occurs in the secondary stage of syphilis.

A nurse assesses a newly admitted client with a diagnosis of pulmonary tuberculosis (TB). Which clinical findings support this diagnosis? Select all that apply.
1
Fatigue
2
Polyphagia
3
Hemoptysis
4
Night sweats
5
Black tongue
134
The general adaptation syndrome is activated in response to Mycobacterium tuberculosis (a gram-positive, acid-fast bacillus), causing an infectious response, which contributes to fatigue; the altered gas exchange also contributes to fatigue because it decreases the available oxygen. Hemoptysis is a response caused by damage to lung tissue; it is associated with more advanced tuberculosis. Night sweats are a common symptom of infectious diseases; the infectious process influences the temperature-regulating center of the brain. Anorexia, not polyphagia, is a common response to most infections. A black, hairy tongue is associated with fungal infection often seen with antibiotic therapy.

A client visits a primary healthcare provider with a report of burning and a sharp pain in the sole of the foot that intensifies in the morning. Which abnormal condition does the nurse anticipate in the client?
1
Torticollis
2
Pes planus
3
Tenosynovitis
4
Plantar fasciitis
4
Plantar fasciitis is a condition associated with a burning sensation and sharp pain in the sole of the foot that worsens in the morning. Torticollis is a condition where the neck is twisted in an unusual position to one side. Pes planus is the abnormal flatness of the sole and arch of the foot. Tenosynovitis is a condition associated with a superficial swelling pain and tenderness along a tendon sheath.

Which nursing assessment is most important for a large-for-gestational-age (LGA) infant of a diabetic mother (IDM)?
1
Temperature less than 98° F (36.6° C)
2
Heart rate of 110 beats/min
3
Blood glucose level less than 40 mg/dL (2.2 mmol/L)
4
Increasing bilirubin during the first 24 hours
3
At birth, circulating maternal glucose is removed; however, the IDM still has a high level of insulin, and rebound hypoglycemia may develop. The temperature-regulating ability of an IDM is similar to that of a healthy neonate, unless the IDM is preterm. A heart rate of 110 beats/min is within the expected range for a newborn. Pathologic jaundice is associated with hemolytic diseases such as Rh and ABO incompatibilities and sepsis, not maternal diabetes.

We have an expert-written solution to this problem!
A client is admitted to the hospital with multiple signs and symptoms associated with a cardiac problem. What clinical finding alerts the nurse that the primary healthcare provider probably will insert a pacemaker?
1
Angina
2
Chest pain
3
Heart block
4
Tachycardia
3
Heart block is the primary indication for a pacemaker because there is an interference with the electrical conduction of impulses from the atria to the ventricles of the heart. The primary treatment for angina is medication; angina is not an indication for a pacemaker. The primary treatment for chest pain is medication; chest pain is not an indication for a pacemaker. The primary treatment for tachycardia is medication; tachycardia is not an indication for a pacemaker.

A nurse is teaching Hands Only Basic Life Support for adults in the community. What should the rescuer do first after determining that the person is not responding and the emergency medical system has been activated?
1
Identify the absence of pulse.
2
Give two rescue breaths with a CPR mask.
3
Perform the head tilt-chin lift maneuver.
4
Perform chest compression at a rate of 100/min.
1
Once it is verified that the person is unresponsive and the emergency medical system has been activated, then whether the client is breathing should be established. Rescue breaths are not given with the hands-only basic life support method of CPR. Chest compressions are initiated as soon as it is identified that the person is not breathing; they are given at a rate of 100/min, to a depth of 2 inches (5 cm) each for 2 minutes, allowing full chest recoil between compressions. This quickly circulates the blood.

We have an expert-written solution to this problem!
Which criteria should a nurse assess before preparing to discharge a pediatric client after the administration of conscious or moderate sedation? Select all that apply.
1
Check if gag reflexes are intact
2
Check if pain and ambulation are at a base level
3
Check if a reversal drug is administered just before discharge
4
Check if the oxygen saturation is below 95% on room air half an hour after the administration of the last dose
5
Check if an adult is accompanying the child and can remain with him or her for at least two half-lives of the anesthetic drug
125
A child’s protective reflexes, such as swallowing and gag reflexes, should be intact before he or she can be discharged. The client should be alert and checked to make sure that pain, ambulation, and orientation are maintained at base level values. An adult should accompany the client to home and should remain with the client until at least two half-lives of the anesthetic drugs are metabolized. The reversal drug should be excreted before discharge. Oxygen saturation levels should be at least at 95% on room air half an hour after the administration of the last dose.

We have an expert-written solution to this problem!
Which order of actions should a nurse follow when performing a chest examination of a client with a pulmonary disorder?
1.
Monitor the respiratory rate.

2.
Observe for any abnormalities of the sternum.

3.
Observe for any evidence of respiratory distress.

4.
Observe the shape and symmetry of the chest.
The first step is to observe the client`s appearance and note any evidence of respiratory distress. Next, the shape and symmetry of the chest along with the chest movements should be determined. After this step, the nurse should check for any abnormalities of the sternum such as pectus carinatum and pectus excavatum. Finally, the nurse should observe for any respiratory rates and abnormal breathing patterns such as Kussmaul breathing or Cheyne-Stokes respiration.

We have an expert-written solution to this problem!
An older client experiences a cerebral vascular accident (CVA) and has right-sided hemiplegia and expressive aphasia. The client’s children ask the nurse which functions will be impaired. Which abilities does the nurse explain will be affected?
1
Stating wishes verbally
2
Recognizing familiar objects
3
Comprehending written words
4
Understanding verbal communication
1
Impaired ability to state wishes verbally is a characteristic of expressive aphasia [1] [2] from damage to Broca area in the dominant hemisphere of the brain. Not recognizing familiar objects is known as agnosia; it is not related to expressive aphasia. Not comprehending written words is known as alexia or dyslexia, a type of receptive aphasia. Not understanding verbal communication is related to receptive aphasia.

A client is admitted to the hospital for surgery for rectosigmoid colon cancer, and the nurse is obtaining a health history as part of the admission process. What clinical findings associated with rectosigmoid colon cancer does the nurse expect the client to report? Select all that apply.
1
Feeling tired
2
Rectal bleeding
3
Inability to digest fat
4
Change in the shape of stools
5
Feeling of abdominal bloating
1245
Anemia may manifest as fatigue, feeling tired, and/or generalized weakness. Anemia is common with rectosigmoid colon cancer from the loss of blood rectally. Passage of red blood (hematochezia) is one of the cardinal signs of rectosigmoid colon cancer; ulceration of the tumor and straining to pass stool precipitate this clinical finding. A cancerous mass can grow into the lumen of the sigmoid colon, altering the shape of stool; stools may be ribbonlike or pencil thin. Tumors in the rectosigmoid colon cause partial and eventually complete obstruction of the intestinal lumen. Stool in the descending and sigmoid colon is more formed, and thus straining to pass stools, gas pains, cramping, and incomplete evacuation commonly occur. An inability to digest fat is not specific to rectosigmoid colon cancer.

The nurse is assessing a client who has syndrome of inappropriate antidiuretic hormone (SIADH). Which finding in the client is consistent with the diagnosis?
1
Preservation of salt
2
Retention of water
3
Decrease of vasopressin
4
Presence of pedal edema
2
SIADH is manifested in the form of retention of free water. This is because of excessive secretion of vasopressin causing reabsorption of water in renal tubules. There is hyponatremia and dilution of serum sodium in SIADH. Decreased vasopressin is seen in diabetes insipidus. Generally pedal (dependent) edema is not seen in SIADH despite the water retention.

A client has returned from surgery with a nephrostomy tube. Which is the most essential nursing intervention for this client?
1
Ensure free drainage of urine.
2
Milk the tube every 2 hours.
3
Keep an accurate record of intake and output.
4
Instill 12 mL of normal saline every 8 hours.
1
The tube must be kept patent to prevent urine backup, hydronephrosis, and kidney damage. Milking the tube every 2 hours is unnecessary. Instill no more than 5 mL of normal saline only if prescribed. Although keeping an accurate record of intake and output is important, it will not ensure free drainage of urine, which is the priority.

We have an expert-written solution to this problem!
The primary healthcare provider diagnoses placenta previa in a primiparous client. What does this indicate to the nurse regarding the condition of the placenta?
1
Infarcted
2
Low-lying
3
Immaturely developed
4
Separating prematurely
2
Implantation should occur in the upper third of the uterus; a low-lying placenta is termed placenta previa. Infarctions may appear on a placenta because of some interference with the blood supply; this is not related to its location within the uterus. Placenta previa indicates where the placenta is implanted and has no relationship to placental aging. Abruptio placentae, not placenta previa, is the premature separation of a normally implanted placenta.

What should the nurse include in the plan of care for a client who just had a total laryngectomy?
1
Instructing the client to whisper
2
Removing the outer tracheostomy tube as needed
3
Placing the client in the orthopneic position
4
Suctioning the tracheostomy tube whenever necessary
4
Secretions are increased because of alterations in structure and function. A patent airway must be maintained. Whispering can put tension on the suture line; initially nonverbal and written forms of communication should be encouraged. The outer tube is not removed because the stoma may close. The orthopneic position may cause neck flexion and block the airway.

The parents of a 12-year-old boy with cystic fibrosis (CF) ask the nurse why he needs a glucose tolerance test. What information should the nurse consider before replying?
1
Pancreatic scarring predisposes the child to diabetes.
2
The thickened mucus blocks the insulin-secreting glands.
3
The test reveals the degree to which the child adheres to the diet.
4
Adjustments of the dosage of pancreatic enzymes are based on the results of the test.
1
Pancreatic scarring affects the ability of the islets of Langerhans to produce insulin; about half of all children with CF have altered glucose tolerance. The endocrine glands, which produce insulin, are ductless and are not affected by the thickened mucus in the ducts. However, the general scarring throughout the pancreas does affect the insulin-producing glands. The glucose tolerance test is a measure of the body’s ability to produce and metabolize carbohydrates, not a measure of the child’s adherence to the diet. The dosage of pancreatic enzymes is based on food consumption, not the blood glucose level.

A 1-month-old infant with a ventricular septal defect (VSD) is examined in the cardiology clinic. What sign related to this disorder does the nurse expect to find when assessing this infant?
1
Bradycardia at rest
2
Activity-related cyanosis
3
Bounding peripheral pulses
4
Murmur at the left sternal border
4
A murmur at the left sternal border is the most characteristic finding in infants and children with a VSD. A left-to-right shunt is caused by the flow of blood from the higher pressure left ventricle to the lower pressure right ventricle. Children with VSDs generally have tachycardia and are often acyanotic. A bounding peripheral pulse is not a common finding in children with a VSD.

An adolescent who works out 6 hours a day reports not eating well, weight loss, and an absence of menses for the past few months. Which nursing intervention is most appropriate?
1
Ask the adolescent to stop exercising for a few days.
2
Talk to the client to find out any reasons for stress.
3
Perform a β-human chorionic gonadotropin pregnancy test.
4
Modify the adolescent’s diet to incorporate more nutrients.
4
If an athletic adolescent experiences symptoms of eating disorders, weight loss, and an absence of menses indicating female athlete triad, then her diet should be modified incorporate more nutrients. Asking the adolescent to stop exercising for a few days would not solve the problem. Stress does not cause amenorrhea. Being an athlete and having eating disorders rules out the chances for becoming pregnant.

An obese smoker complains of feeling sleepy during the daytime, waking up tired in the morning, and snoring heavily while sleeping. The client is found to have enlarged tonsils. Which condition may the client have?
1
Laryngeal trauma
2
Vocal cord paralysis
3
Obstructive sleep apnea
4
Subcutaneous emphysema
3
Obstructive sleep apnea (OSA) is a condition in which the client may feel tired upon waking in the morning and may feel sleepy during the daytime. These clients may also snore heavily while sleeping. Smoking and enlarged tonsils increase the risk of sleep apnea. Laryngeal trauma occurs secondary to a crushing or direct blow injury, fracture, or prolonged endotracheal intubation. Vocal cord paralysis occurs in clients with neurologic disorders or with conditions that damage either the vagus nerve or the laryngeal nerves. Subcutaneous emphysema is a manifestation of laryngeal trauma, a condition in which there is the presence of air in the subcutaneous tissue.

An older adult in an acute care setting is having urinary incontinence. Which interventions would help the client? Select all that apply.
1
Provide nutritional support
2
Provide voiding opportunities
3
Avoid indwelling catheterization
4
Provide beverages and snacks frequently
5
Promote measures to prevent skin breakdown
235
An older adult should be provided voiding opportunities to minimize urinary incontinence. Indwelling catheterization should be avoided because this action increases the risk of infection and may cause discomfort. Measures to prevent skin breakdown should be taken because the client may develop skin problems due to incontinence. Nutritional support and frequent beverages and snacks should be provided to a client with malnutrition.

A nurse is caring for a client who was diagnosed with a myocardial infarction. While caring for the client 2 days after the event, the nurse identifies that the client’s temperature is elevated. The nurse concludes that this increase in temperature is most likely the result of what?
1
Tissue necrosis
2
Venous thrombosis
3
Pulmonary infarction
4
Respiratory infection
1
The body’s inflammatory response to myocardial necrosis causes an elevation of temperature as well as leukocytosis within 24 to 48 hours after the event. Venous thrombosis and pulmonary infarction are not expected findings after a myocardial infarction. Respiratory infection is not common after myocardial infarction.

The nurse is caring for a client with a body surface burn injury of 55%. Which information will the nurse consider when planning care for this client?
1
Is prone to poor healing because of a hypermetabolic state
2
Has a decreased risk of infection when in a hypermetabolic state
3
Needs a cool environment to decrease caloric need
4
Will need 20 calories/kg during the healing process
1
Burn injuries cause a hypermetabolic state. This results in lipid and protein catabolism, which in turn can inhibit wound healing. A hypermetabolic state increases the risk for slowed wound healing, increasing the chance for infection. Cooling the environment would cause an increase in caloric need as the body tries to warm to core temperature. Clients with burn injuries require increased calories and protein to promote wound healing. For an adult client, 20 calories/kg does not provide an adequate increase of calories or protein for the hypermetabolic state associated with burns.

When providing preoperative teaching, what should the nurse focus primarily on?
1
Helping the client and family decide if surgery is necessary
2
Providing emotional support to the client and family
3
Giving minute-by-minute details of the surgery to the client and family
4
Providing general information to reduce client and family anxiety
4
The primary role of the nurse during preoperative teaching is to provide general information about the surgical experience and what to expect before and after surgery. Helping the client and family decide if surgery is necessary is not an appropriate intervention for the nurse. Emotional support is important and would be included as part of providing general information to reduce client and family anxiety. It is also not appropriate for the nurse to describe minute-by-minute details of the surgery unless the client and family request this information, at which time the surgeon should answer the questions.

We have an expert-written solution to this problem!
A nurse is caring for a client who is cachectic. What information about the function of adipose tissue in fat metabolism is necessary to better address the needs of this client?
1
Releases glucose for energy
2
Regulates cholesterol production
3
Uses lipoproteins for fat transport
4
Stores triglycerides for energy reserves
4
A triglyceride is composed of three fatty acids and a glycerol molecule. When energy is required, the fatty acids are mobilized from adipose tissue for fuel. The nurse needs to consider that a client who is cachectic will have limited reserves to meet energy needs. Releasing glucose for energy is not the function of adipose tissue; its main function is storage. Regulating cholesterol production is not a function of adipose tissue; cholesterol is produced in the liver. Using lipoproteins for fat transport is not the function of adipose tissue in fat metabolism.

We have an expert-written solution to this problem!
A client arrives at the emergency room complaining of chest pain and dizziness. The client has a history of angina. The primary healthcare provider prescribes an electrocardiogram (ECG) and lab tests. A change in which component of the ECG tracing should the nurse recognize as the client actively having a myocardial infarction (MI)?
1
QRS complex
2
S-T segment
3
P wave
4
R wave
2
In ECG tracing, the displacement of the S-T segment is caused by an active ischemic injury in the myocardium. The QRS complex, the P wave, and the R wave are not associated with an MI.

A client feeling increasingly tired seeks medical care. Type 1 diabetes is diagnosed. What causes increased fatigue with type 1 diabetes?
1
Increased metabolism at the cellular level
2
Increased glucose absorption from the intestine
3
Decreased production of insulin by the pancreas
4
Decreased glucose secretion into the renal tubules
3
Insulin facilitates transport of glucose across the cell membrane to meet metabolic needs and prevent fatigue. With diabetes there is decreased cellular metabolism because of the decrease in glucose entering the cells. Glucose is not absorbed from the intestinal tract by the cells; fatigue is caused by decreased, not increased, cellular levels of glucose. Filtration and excretion of glucose by the kidneys do not regulate energy levels; if insulin production is adequate, glucose does not spill into the urine.

While assessing the skin of a light-skinned client, the nurse concludes that the client has ecchymosis. Which skin color variation would confirm this diagnosis?
1
Gray color
2
Dark red color
3
Deep brown color
4
White color
2
Dark red skin coloring is identified as ecchymosis. A grayish skin color is due to cyanosis. A deep brown skin coloring is caused by erythema in dark-skinned clients. A white or ashen skin color is found in clients with pallor.

We have an expert-written solution to this problem!
A female adolescent complains of breast pain. Which antigonadotropic herb may alleviate breast pain by decreasing prolactin levels?
1
Catnip
2
Black haw
3
Bugleweed
4
Chaste tree fruit
3
Bugleweed is an herb used to decrease breast pain by decreasing prolactin levels and facilitating an antigonadotropic effect. Black haw and catnip are herbs that act as uterine antispasmodics. Chaste tree fruit also decreases breast pain by decreasing prolactin levels, but it is not antigonadotropic.

A client develops a nonhealing ulcer of a lower extremity and complains of leg cramps after walking short distances. The client asks the nurse what causes these leg pains. Which would be the best response by the nurse?
1
“Muscle weakness occurs in the legs because of a lack of exercise.”
2
“Edema and cyanosis occur in the legs because they are dependent.”
3
“Pain occurs in the legs while walking because there is a lack of oxygen to the muscles.”
4
“Pressure occurs in the legs because of vasodilation and pooling of blood in the extremities.”
3
Intermittent claudication is the pain that occurs during exercise because of a lack of oxygen to muscles in the involved extremities. It is exercise, not the lack of exercise, that precipitates muscle weakness. Edema and cyanosis in the legs and pressure in the legs are related to venous problems, not an arterial problem.

Which symptoms indicate to the nurse that the client has an inadequate fluid volume? Select all that apply.
1
Decreased urine
2
Hypotension
3
Dyspnea
4
Dry mucous membranes
5
Pulmonary edema
6
Poor skin turgor
1246
Lowered urinary output, hypotension, dry mucous membranes, and poor skin turgor are all symptomatic of dehydration. Dyspnea and pulmonary edema may be caused by fluid overload.

We have an expert-written solution to this problem!
A 28-year-old woman comes into the clinic and tells the nurse that she fears that she is infertile, because she has been trying to become pregnant unsuccessfully for 2 years. While collecting the health history the nurse learns that the client experiences irregular and infrequent menstrual periods. The client is overweight and has severe acne and alopecia. The primary healthcare provider diagnoses the condition polycystic ovarian syndrome (PCOS). Which of the following interventions is the most important?
1
Consoling the client over her inability to have children
2
Discussing weight loss, exercise, and a balanced low-fat diet
3
Providing information to the client on how to prepare for surgery
4
Informing the client that there are no long-term complications of PCOS
2
Weight loss, exercise, and a balanced low-fat diet can reduce insulin and androgen levels related to PCOS. Meeting with a dietitian may be helpful. Surgery is not necessary at this time. The primary healthcare provider would most likely prescribe hormones, other medications, or both. If pregnancy does not occur, surgery is an option. Pregnancy may be possible with hormones, other medications, or both. Early detection of PCOS is important, because the condition can lead to type 2 diabetes; hypertension; cardiovascular disease; and ovarian, breast, and endometrial cancers. The nurse should encourage treatment compliance and positive lifestyle changes.

A client has laparoscopic surgery to remove a calculus from the common bile duct. What postoperative client response indicates to the nurse that bile flow into the duodenum is reestablished?
1
Stools become brown
2
Liver tenderness is relieved
3
Colic is absent after ingestion of fats
4
Serum bilirubin level returns to the expected range
1
The return of brown color to the stool indicates that bile is entering the duodenum and being converted to urobilinogen by bacteria. Liver tenderness is unrelated to bile flow. The absence of biliary colic is related to the removal of the calculus, not the flow of bile. The serum bilirubin level is not affected.

Which statement is true regarding varicocele?
1
It is commonly seen in prepubertal children.
2
It results in partial or complete venous occlusion.
3
It results in the red, warm, and edematous scrotum.
4
It causes elongation of the veins of the spermatic cord.
4
Varicocele is characterized by dilation and elongation of the veins of spermatic cord that is presently superior to a testicl*. It is rarely seen in prepubertal children. Testicular torsion results in partial or complete venous occlusion. In cases of severe torsion, scrotum becomes red, warm, and edematous and appears to be immobile.

We have an expert-written solution to this problem!
The nurse is caring for a client who underwent intestinal surgery 3 days ago and notices brownish pus with a fecal odor draining from the incision. What should the nurse infer from this finding?
1
Colonization with Proteus
2
Colonization with Pseudomonas
3
Colonization with Staphylococcus
4
Colonization with aerobic coliform and Bacteroides
4
A client who underwent intestinal surgery is more susceptible to developing colonization of aerobic coliform and Bacteroides, which results in brown pus with a fecal odor. Beige pus that has a fishy odor is formed due to colonization with Proteus. Greenish-blue pus that has a fruity smell is formed due to colonization with Pseudomonas. Creamy-yellow pus indicates a colonization of Staphylococcus.

A nurse is teaching the parents of a school-aged child with sickle cell anemia about ways to prevent sickling. What should the nurse explain as the primary cause of sickling?
1
Hypoxia
2
Hemodilution
3
Hypocalcemia
4
Hemoglobin
1
Under conditions of decreased oxygen, the relatively insoluble hemoglobin S changes its molecular structure to form long, slender crystals and eventually the crescent, or sickled, shape. Hemodilution, accomplished with increased fluids, helps prevent sickling. Hypocalcemia will not influence the sickling process. Between 65% and 100% of hemoglobin F is found in individuals with thalassemia major (Cooley anemia).

Which clinical findings are observed in a client suffering from an imbalance of adrenocorticotropic hormone? Select all that apply.
1
Anorexia
2
Hyponatremia
3
Slowed cognition
4
Postural hypotension
5
Decreased muscle strength
124
A client with adrenocorticotropic imbalance experiences anorexia, decreased sodium levels (hyponatremia), and postural hypotension. Slowed cognition is observed in clients with thyroid hormone imbalance. Decreased muscle strength is seen in clients with growth hormone imbalance.

We have an expert-written solution to this problem!
A nurse is caring for a client who is admitted to the hospital with severe dyspnea and a diagnosis of cancer of the lung. What does the nurse conclude is the probable cause of the severe dyspnea?
1
Abdominal distention or pressure
2
Bronchial obstruction or pleural effusion
3
Fluid retention as a result of renal failure
4
Anxiety associated with pain on inspiration
2
Proliferation of malignant cells may obstruct the bronchial tree or foster development of exudate in the pleural space, decreasing the availability of oxygen and increasing retention of carbon dioxide. A tumor of the lung does not cause abdominal distention or pressure. Fluid retention as a result of renal failure is not associated with cancer of the lung. Although anxiety associated with pain may increase the respiratory rate, it will not cause difficulty with breathing.

We have an expert-written solution to this problem!
Which information may be obtained by palpation? Select all that apply.
1
Turgor
2
Bruises
3
Texture
4
Lesions
5
Moisture content
6
Tissue integrity
1345
Using palpation, information about turgor, texture, lesions, and moisture content of the skin can be gathered. Information related to bruises and tissue integrity of the skin will not be gathered by using palpation.

We have an expert-written solution to this problem!
A nurse is caring for a client after abdominal surgery and encourages the client to turn from side to side and to engage in deep-breathing exercises. What complication is the nurse trying to prevent?
1
Metabolic acidosis
2
Metabolic alkalosis
3
Respiratory acidosis
4
Respiratory alkalosis
3
Shallow respirations, bronchial tree obstruction, and atelectasis compromise gas exchange in the lungs; an increased carbon dioxide level leads to respiratory acidosis [1] [2]. Metabolic acidosis occurs with diarrhea; alkaline fluid is lost from the lower gastrointestinal tract. Metabolic alkalosis is caused by excessive loss of hydrogen ions through gastric decompression or excessive vomiting. Respiratory alkalosis is caused by increased expiration of carbon dioxide, a component of carbonic acid.

A client at 36 hours’ postpartum is being treated with subcutaneous enoxaparin for deep vein thrombosis of the left calf. Which client adaptation is of most concern to the nurse?
1
Dyspnea
2
Pulse rate of 62 beats/min
3
Blood pressure of 136/88 mm Hg
4
Homan sign in the left leg
1
One complication of deep vein thrombosis is pulmonary embolism; dyspnea is a significant sign that should be reported immediately. A low pulse rate is common for several days after birth because of the cardiovascular changes that occur during the early postpartum period. A blood pressure of 136/88 mm Hg is not significant in a client with a deep vein thrombosis. Checking for the Homan sign is contraindicated, because the clot could be dislodged.

The nurse provides discharge teaching for a client with a history of hypertension who had a femoropopliteal bypass graft. Which client statement indicates teaching is effective?
1
“I should massage my calves and feet every day.”
2
“I should keep my foot elevated when I am in bed.”
3
“I should sit in a hot bath for half an hour twice a day.”
4
“I should observe the color and pulses of my legs every day.”
4
Presence of pulses and a normal skin color indicate adequate arterial perfusion and graft viability. Massaging calves and feet every day is contraindicated in peripheral vascular disease because it may traumatize vessels; it can cause a thrombus to become an embolus. Keeping feet elevated when in bed is appropriate for venous, not arterial, problems. The peripheral dilation produced by a hot bath will increase the workload on the heart, which is undesirable in a client with hypertension.

When a client has a myocardial infarction, one of the major manifestations is a decrease in the conductive energy provided to the heart. When assessing this client, the nurse is aware that the existing action potential is in direct relationship to what?
1
Heart rate
2
Refractory period
3
Pulmonary pressure
4
Strength of contraction
4
A direct relationship exists between the strength of cardiac contractions and the electrical conductions through the myocardium. The heart rate is related to factors such as sinoatrial (SA) node function, partial pressures of oxygen and carbon dioxide, and emotions. The refractory period is when the heart is at rest, not when it is contracting. Pulmonary pressure does not influence action potential; it becomes elevated in the presence of left ventricular failure.

We have an expert-written solution to this problem!
The nurse is caring for a 70-year-old client who presents with dilute urine even when fluid intake is low. What could be the possible cause of the client’s condition?
1
Decreased glucose tolerance
2
Decreased general metabolism
3
Decreased ovarian production of estrogen
4
Decreased antidiuretic hormone production
4
Dilute urine with decreased fluid intake indicates a decrease in antidiuretic hormone production. Decreased glucose tolerance causes elevated fasting and random blood glucose levels. The clinical manifestations of decreased general metabolism are decreased heart rate and blood pressure, decreased appetite, and decreased tolerance to cold. Decreased ovarian production of estrogen may result in decreased bone density and thin and dry skin.

The parents of a school-aged child with leukemia ask the nurse why irradiation of the spine and skull is necessary. What is the most accurate response by the nurse?
1
“Radiation retards the growth of cells in the bone marrow of the cranium.”
2
“This therapy decreases cerebral edema and prevents increased intracranial pressure.”
3
“Leukemic cells may invade the nervous system, but the usual drugs are ineffective in the brain.”
4
“Neoplastic drug therapy without radiation is effective in most cases, but this is a precautionary treatment.”
3
The protective blood-brain barrier initially screens leukemic cells from the central nervous system. However, in advanced stages leukemic infiltration occurs. Chemotherapeutic agents, also screened out by the blood-brain barrier, are ineffective. Radiation destroys, not just retards, malignant cells. Radiation does not decrease cerebral edema. Irradiation of the cranium is needed because chemotherapy does not pass the blood-brain barrier.

Test-Taking Tip: The following are crucial requisites for doing well on the NCLEX exam: (1) A sound understanding of the subject; (2) The ability to follow explicitly the directions given at the beginning of the test; (3) The ability to comprehend what is read; (4) The patience to read each question and set of options carefully before deciding how to answer the question; (5) The ability to use the computer correctly to record answers; (6) The determination to do well; (7) A degree of confidence.

During the progressive stage of shock, anaerobic metabolism occurs. Which complication should the nurse anticipate in this client?
1
Metabolic acidosis
2
Metabolic alkalosis
3
Respiratory acidosis
4
Respiratory alkalosis
1
Metabolic acidosis occurs during the progressive stage of shock as a result of accumulated lactic acid. Metabolic alkalosis cannot occur with the buildup of lactic acid associated with the progressive stage of shock. Respiratory acidosis can result from decreased respiratory function in late shock, further compounding metabolic acidosis. Respiratory alkalosis occurs as a result of hyperventilation during early shock.

We have an expert-written solution to this problem!
A nurse is reviewing the history, physical examination, and diagnostic test results of a client with colitis. What clinical findings are associated with this disorder? Select all that apply.
1
Anemia
2
Diarrhea
3
Hemoptysis
4
Abdominal cramps
5
Decreased white blood cells
124
Ulceration of the intestinal mucosa commonly occurs, causing blood loss and anemia. The inflammatory process tends to increase peristalsis, causing diarrhea, electrolyte imbalances, and weight loss. The inflammatory process tends to increase peristalsis, causing abdominal cramping and diarrhea. Coughing up blood from the respiratory tract (hemoptysis) is not associated with colitis. A decreased number of white blood cells (leukopenia) is not associated with colitis.

A client has an endotracheal tube and is receiving mechanical ventilation. Periodic suctioning is necessary, and the nurse follows a specific protocol when performing this procedure. Select in order of priority the nursing actions that should be taken when suctioning.
1.
Insert the catheter without applying suction

2.
Assess client’s vital signs and lung sounds

3.
Rotate the catheter while suction is applied

4.
Administer oxygen via a ventilation bag
The nurse should first assess the client’s vital signs and lung sounds to determine if suctioning is needed. Then 100% oxygen should be administered to compensate for the lack of oxygen intake during the suctioning process. Suctioning should not be applied during catheter insertion to limit trauma. Rotating the catheter during withdrawal ensures thorough removal of secretions.

A registered nurse is supervising a student nurse while assessing a 70-year-old client who is receiving aminoglycoside therapy. Which statement about the client’s condition requires correction?
1
“The client may have deterioration of the cochlea.”
2
“The client may have thinning of the tympanic membrane.”
3
“The client may have an inability to hear high-frequency sounds.”
4
“The client may have an inability to differentiate between consonants.”
2
An older adult who is on aminoglycoside antibiotic therapy is at a high risk of developing ototoxicity. The client with ototoxicity may have thickening of the tympanic membrane, but not thinning of the tympanic membrane. Deterioration of the cochlea may cause older adults to gradually lose hearing. They may experience an inability to hear high-frequency sounds and differentiate between consonants.

The laboratory reports of a client who underwent a hypophysectomy show an intracranial pressure (ICP) of 20 mmHg. Which action made by the client is responsible for this condition?
1
Drinking lots of water
2
Eating high-fiber foods
3
Bending over at the waist
4
Bending knees when lowering body
3
Bending over at the waist should be avoided as this position increases intracranial pressure in clients who underwent hypophysectomy. Drinking lots of water and eating high-fiber foods reduce the risk of constipation, so this should not cause increased intracranial pressure. The client should bend the knees then lowering their body to reduce the risk of intracranial pressure.

We have an expert-written solution to this problem!
Pharmaco*kinetic factors determine the concentration of a drug at its site of action. These are different in neonates and infants than they are in small children or adults. Arrange the factors in the order in which they occur.
1.
Excretion

2.
Absorption

3.
Distribution

4.
Metabolism
The first step after administering the drug through any route is absorption. For instance, orally administered drugs are generally absorbed in the gastrointestinal tract. After absorption, drugs may be distributed through protein binding, or some may enter the central nervous system through the blood-brain barrier. Most drugs are then metabolized in the liver; this capacity is low in newborns. Drugs do not remain in the body indefinitely; rather they are excreted by the kidneys.

While assessing a client with acquired immunodeficiency syndrome (AIDS), the nurse suspects that the client has developed cryptococcosis. Which clinical manifestations support the nurse’s suspicion of a cryptococcosis infection? Select all that apply.
1
Seizures
2
Dyspnea
3
Blurred vision
4
Neurologic deficits
5
Enlarged lymph nodes
134
Seizures, neurologic problems/deficits, and blurred vision are the manifestations of cryptococcosis. Cryptococcosis is a debilitating meningitis and can be a widely spread infection in clients who have AIDS. It is caused by Cryptococcus neoformans. Histoplasmosis is a respiratory infection caused by Histoplasma capsulatum, which progresses to widespread infection in a client with AIDS. The symptoms of histoplasmosis are dyspnea and enlarged lymph nodes.

The nurse is caring for a client who reports excessive tearing. Which disorders does the nurse suspect could be responsible for the client’s condition? Select all that apply.
1
Chalazion
2
Entropion
3
Hordeolum
4
Conjunctivitis
5
Keratoconjunctivitis sicca
124
A chalazion is an inflammation of a sebaceous gland in the eyelid manifested by excessive tearing and light sensitivity. An entropion is a disorder of the eyelid that causes pain and excessive tearing. Conjunctivitis also causes excessive tearing, a bloodshot appearance, and itching. A hordeolum is an infection of the eyelid sweat glands that causes red, swollen, and painful eyes. Keratoconjunctivitis sicca is dry eye syndrome; this disorder causes decreased tear production.

The nurse in the postanesthesia care unit is caring for a client who had a left-sided pneumonectomy. Which goal is priority?
1
Replace blood loss
2
Maintain ventilatory exchange
3
Maintain closed chest drainage
4
Replace supplemental oxygenation
2
Oxygen and carbon dioxide exchange is essential for life and is the priority. Blood replacement is not the priority. Closed chest drainage is unnecessary with a left-sided pneumonectomy because there is no lung to reinflate. Supplemental oxygenation is not the priority.

STUDY TIP: Laughter is a great stress reliever. Watching a short program that makes you laugh, reading something funny, or sharing humor with friends helps decrease stress.

A client diagnosed with multiple myeloma has been given a poor prognosis. After discharge, the client plans to travel on an airplane and attend sporting events with friends and family. The nurse prepares a discharge teaching plan for this client. What should the plan include?
1
Eliminating travel plans to combat anemia-related fatigue
2
Reinforcing a positive mental attitude to improve prognosis
3
Preventing infection; the client is at risk for leukopenia
4
Restricting fluid intake; the client is at risk for congestive heart failure
3
The bone marrow is impaired with multiple myeloma; the effectiveness of white blood cells and immunoglobulin is reduced, which increases susceptibility to bacterial infections. Travel can be accomplished with careful planning and adequate rest periods. Although a positive mental attitude can contribute to quality of life and may even extend life, generally it does not change the prognosis. The client is encouraged to drink plenty of fluids to help dilute the Bence Jones protein fragments in the urine, which may help prevent kidney damage.

The nurse is assessing an older adult client with suspected hearing loss. Which observations made by the nurse in the client indicates a decrease in hearing acuity? Select all that apply.
1
Frequent usage of words such as “what”
2
Postural changes while listening to the speaker
3
Bending towards the other person while talking
4
Mismatch in the questions asked and the responses given
5
Startled expression when there is any unexpected sound in the environment
1234
Hearing assessment begins while observing the client listening to and answering the questions asked by the nurse. Indicators of hearing difficulty in the client frequently include asking the speaker to repeat statements or frequently saying “What?” or “Huh?” Changes in the client’s posture, such as leaning forward when listening to the speaker or tilting the head to one side, can provide information about hearing acuity. The nurse should also assess whether the client’s responses match the questions asked; mismatch in the client’s responses may indicate a decrease in hearing acuity. Startling to an unexpected sound in the environment determines no loss in hearing acuity.

A client with adrenal insufficiency reports feeling weak and dizzy, especially in the morning. What should the nurse determine is the most probable cause of these symptoms?
1
A lack of potassium
2
Postural hypertension
3
A hypoglycemic reaction
4
Increased extracellular fluid volume
3
Deficiency of glucocorticoids causes hypoglycemia in the client with Addison disease. Clinical manifestations of hypoglycemia include nervousness; weakness; dizziness; cool, moist skin; hunger; and tremors. Hypokalemia is evidenced by nausea, vomiting, muscle weakness, and dysrhythmias. Weakness with dizziness on arising is postural hypotension, not hypertension. An increased extracellular fluid volume is evidenced by edema, increased blood pressure, and crackles.

Test-Taking Tip: Choose the best answer for questions asking for a single answer. More than one answer may be correct, but one answer may contain more information or more important information than another answer.

We have an expert-written solution to this problem!
A female client reports excessive hair growth on the face and chest. The nurse suspects ovarian dysfunction. Which findings support this assessment? Select all that apply.
1
Deepened voice
2
Enlarged cl*tor*s
3
Capillary fragility
4
Changes in fat distribution
5
Increased thyroid gland activity
12
The excessive growth of hair on the face and chest in women may be due to hirsutism. It is associated with an endocrine gland dysfunction. However, if accompanied by a deepened voice and enlargement of the cl*tor*s, it may indicate ovarian dysfunction. Capillary fragility and changes in the distribution of fat may be associated with Cushing’s disease. Hyperthyroidism can cause symptoms of softness or smoothness of the skin texture.

Test-Taking Tip: Be alert for details about what you are being asked to do. In this Question Type, you are asked to select all options that apply to a given situation or client. All options likely relate to the situation, but only some of the options may relate directly to the situation.

The nurse is caring for a client who has a lesion in the right upper lobe. A diagnosis of tuberculosis (TB) has been made. What are the clinical manifestations of tuberculosis?
1
Frothy sputum and fever
2
Dry cough and pulmonary congestion
3
Night sweats and blood-tinged sputum
4
Productive cough and engorged neck veins
3
Blood-tinged sputum, in the absence of pronounced coughing, often is the presenting sign of TB; diaphoresis at night is a later sign. Recurrent fever is present; frothy sputum occurs with pulmonary edema. A productive cough occurs with TB. A productive cough occurs with TB, but engorged neck veins occur with heart failure.

While auscultating the heart, a healthcare provider notices S3 heart sounds in four clients. Which client is at more risk for heart failure?
1
Child client
2
Pregnant client
3
Older adult client
4
Young adult client
3
The S3 is the third heart sound heard after the normal “lub-dub.” It is indicative of congestive heart failure in adults over 30 years old. In young, pregnant, and under 30 year old clients, the third heart sound is often considered to be a normal parameter.

Test-Taking Tip: You have at least a 25% chance of selecting the correct response in multiple-choice items. If you are uncertain about a question, eliminate the choices that you believe are wrong and then call on your knowledge, skills, and abilities to choose from the remaining responses.

A nurse is assessing a client for possible malabsorption syndrome. Which stool assessment finding will support this diagnosis?
1
Melena
2
Frank blood
3
Fat globules
4
Currant jelly consistency
3
Undigested fat in the feces (steatorrhea) is associated with diseases of the intestinal mucosa (e.g., celiac sprue) or pancreatic enzyme deficiency. Darkening of feces by blood pigments (melena) is related to upper gastrointestinal (GI) bleeding. Bright red blood in the stool is related to lower GI bleeding (e.g., hemorrhoids). Stools containing blood and mucus (currant jelly stools) are associated with intussusception.

We have an expert-written solution to this problem!
The nurse is reviewing the laboratory reports of a client who has sustained a significant reaction to the tuberculin skin test but has negative findings on bacteriologic studies. The reports further reveal the absence of x-ray findings compatible with tuberculosis (TB) and clinical evidence of TB. Which class of TB does the nurse suspect?
1
Class 0
2
Class 1
3
Class 2
4
Class 3
3
In class 2 TB, the client demonstrates a significant reaction to the tuberculin skin test but bacteriologic studies are negative and there is no clinical or radiographic evidence of TB. The client with class 2 TB has been exposed to latent TB infection but has no disease. In class 0 TB, the client has had no exposure to TB and has negative results on skin testing. In class 1, a client has been exposed to TB but demonstrates no evidence of infection (e.g., a negative result on tuberculin skin testing). In class 3, the client has clinically active TB infection.

A client is at high risk for developing ascites because of cirrhosis of the liver. How should the nurse assess for the presence of ascites?
1
Observe the client for signs of respiratory distress.
2
Percuss the client’s abdomen and listen for dull sounds.
3
Palpate the lower extremities over the tibia and observe for edema.
4
Listen for decreased or absent bowel sounds while auscultating the abdomen.
2
Percussing over the client’s abdomen will produce a dull, not tympanic, sound if fluid is present. Respiratory distress occurs with ascites, but it is not an early sign; the client does not have ascites but is at risk for ascites at this time. Palpating the lower extremities assesses for dependent edema, not ascites. Ascites is fluid within the peritoneal cavity. Bowel sounds may be heard with developing ascites; when ascites is extensive, bowel sounds may diminish.

We have an expert-written solution to this problem!
Which clinical manifestation occurs in a client with adrenal insufficiency?
1
Vitiligo
2
Moon face
3
Hypertension
4
Truncal obesity
1
Adrenal insufficiency is clinically manifested as patchy white areas on the skin (vitiligo). Moon face, hypertension, and truncal obesity are clinical manifestations of Cushing’s syndrome.

We have an expert-written solution to this problem!
A client with acute kidney injury states, “Why am I twitching and my fingers and toes tingling?” Which process should the nurse consider when formulating a response to this client?
1
Acidosis
2
Calcium depletion
3
Potassium retention
4
Sodium chloride depletion
2
In kidney failure, as the glomerular filtration rate decreases, phosphorus is retained. As hyperphosphatemia occurs, calcium is excreted. Calcium depletion hypocalcemia [1] [2] causes tetany, which causes twitching and tingling of the extremities, among other symptoms. Acidosis, potassium retention, and sodium chloride depletion are not characterized by twitching and tingling of the extremities.

A client with varicose veins is scheduled for surgery. Which clinical finding does the nurse expect to identify when assessing the lower extremities of this client?
1
Pallor
2
Ankle edema
3
Yellowed toenails
4
Diminished pedal pulses
2
Ankle edema results from increased venous pressure. Pigmentation, not pallor, may occur with varicosities. Yellowed toenails occur with arterial, not venous, insufficiency. Diminished pedal pulses occur with arterial, not venous, insufficiency.

On reviewing the data of a client with thyroid disorder, the primary healthcare provider prescribed atenolol. Which assessment findings would indicate the need for atenolol therapy? Select all that apply.
1
Tachycardia
2
Atrial fibrillation
3
Distant heart sounds
4
Systolic hypertension
5
Decreased cardiac output
124
In hyperthyroidism, atenolol is prescribed to reduce cardiac manifestations. Tachycardia, atrial fibrillation, and systolic hypertension are cardiac manifestations associated with hyperthyroidism. Distant heart sounds are associated with hypothyroidism. The cardiac output is increased in hyperthyroidism.

The nurse is caring for a client who is admitted with a crushing injury to the spinal cord above the level of phrenic nerve origin. What should the nurse consider about this type of injury when planning care?
1
Ventricular fibrillation
2
Vagus nerve dysfunction
3
Retention of sensation and paralysis of lower extremities
4
Lack of diaphragmatic contractions and respiratory paralysis
4
The phrenic nerves innervate the diaphragm; therefore, a crushing spinal cord injury above the level of phrenic origin will stop diaphragmatic contractions and result in respiratory paralysis. Cardiac activity is not affected; the heart is regulated by the autonomic nervous system from fibers originating in the medulla. Activities regulated by the vagus nerve are unaffected; the vagi originate in the medulla, which is superior to the cervical region. In a crushing spinal cord injury, both motor and sensory conduction are affected.

A 2-year-old child is admitted to the pediatric unit with a diagnosis of thalassemia major (Cooley anemia). The parents are told that there is no cure, but the anemia can be treated with frequent blood transfusions. The father tells the nurse he is glad that there is a treatment that “fixes” his child’s problem. Before responding, the nurse should recall that blood transfusions do what?
1
Correct the anemia, but may cause other problems
2
Reverse the anemia, but also present a risk of hepatitis
3
Are a supportive treatment; fewer will be needed as the child grows older
4
Are a replacement for defective red blood cells; they are like giving insulin to a person with diabetes
1
Excess iron from hemolysis of the replaced red blood cells is deposited in the organs and body tissue, causing hemosiderosis. Chelation therapy is then required. With the practice of aseptic technique and screening of donated blood, hepatitis should not occur. Red blood cell replacement depends on the child’s hematologic picture; the number of transfusions is not related to age. Although red blood cells are replaced, it is erroneous to compare this treatment with insulin therapy.

We have an expert-written solution to this problem!
The nurse educates a client on decreasing the risk of developing antibiotic-resistant infections. Which statement made by the nurse will be most significant?
1
“Wash your hands frequently.”
2
“Do not skip any dose of your antibiotics.”
3
“Save the unfinished antibiotics for later use.”
4
“Stop taking the antibiotics when you feel better.”
2
Antibiotic-resistant infection develops when the hardiest bacteria survive and multiply. This may happen when a client stops taking an entire course of antibiotics, which leads to infections that are resistant to many antibiotics. Therefore a client should not skip any dose of an antibiotic. Hand washing is required to prevent infections; it is not related to antibiotic-resistant infections. Antibiotics should not be stopped even if the client has started feeling better; the full course of treatment should be taken. Non-compliance in taking the full course of prescribed antibiotics can lead to an antibiotic-resistant infection. It is dangerous to take the unfinished antibiotics at a later time; it may prove fatal if the antibiotics are outdated.

When the fetal head begins to crown during an emergency precipitous birth, how should the nurse respond?
1
Pressing firmly on the fundus
2
Applying gentle perineal pressure
3
Encouraging the client to push forcefully
4
Telling the client to take prolonged deep breaths
2
Applying gentle perineal pressure prevents too-rapid expulsion of the head, which can lead to perineal laceration in the mother. Pressing firmly on the fundus is contraindicated; a precipitate birth is caused by forceful uterine contractions that expel uterine contents. Pushing may cause too-rapid expulsion and perineal laceration. At this time the urge to push is uncontrollable; the client will be unable to take prolonged deep breaths.

Test-Taking Tip: What happens if you find yourself in a slump over the examination? Take a time-out to refocus and reenergize! Talk to friends and family who support your efforts in achieving one of your major accomplishments in life. This effort will help you regain confidence in yourself and get you back on track toward the realization of your long-anticipated goal.

We have an expert-written solution to this problem!
A client who had surgery for a ruptured appendix develops peritonitis. Which clinical findings related to peritonitis should the nurse expect the client to exhibit? Select all that apply.
1
Fever
2
Hyperactivity
3
Extreme hunger
4
Urinary retention
5
Abdominal muscle rigidity
15
A moderate fever is associated with inflammation of the peritoneal membrane. Muscular rigidity over the affected area is a classic sign of peritonitis. Malaise, rather than hyperactivity, is often associated with peritonitis. Nausea, not hunger, is a common occurrence with peritonitis. Urinary retention may occur following surgery as a complication of anesthesia, not peritonitis.

A client with mild preeclampsia is admitted to the high-risk prenatal unit because of a progressive increase in her blood pressure. The nurse reviews the primary healthcare provider’s prescriptions. Which prescriptions does the nurse expect to receive for this client? Select all that apply.
1
Daily weight
2
Side-lying bed rest
3
2 g/day sodium diet
4
Deep tendon reflexes
5
Glucose tolerance test
124

Rapid weight gain is a sign of increasing edema. One liter of fluid is equal to 2.2 lb. Maintaining bed rest promotes fluid shift from the interstitial spaces to the intravascular space, which enhances blood flow to the kidneys and uterus; the side-lying position promotes placental perfusion. A 2 g/day sodium diet will deplete the circulating blood volume, limiting blood flow to the placenta. A moderate sodium intake (6 g or less) is permitted as long as the client is alert and has no nausea or indication of an impending seizure. Deep tendon reflexes should be monitored. Reflexes of +2 are indicative of mild preeclampsia; +4 indicates severe preeclampsia. There are no data indicating that a glucose tolerance test is needed.

The registered nurse is teaching a nursing student about ways to minimize heat radiation. Which statements made by the nursing student indicate effective learning? Select all that apply.
1
“I will apply an ice pack to the client.”
2
“I will cover the client with dark clothes.”
3
“I will instruct the client to remove extra clothes.”
4
“I will instruct the client to lie in the fetal position.”
5
“I will advise the client to wear sparsely woven clothes.”
124
Applying an ice pack will increase conductive heat loss, which results in minimizing heat radiation. Wearing dark clothes and lying in the fetal position will minimize heat radiation. Removing extra clothes will increase heat radiation. Wearing sparsely woven clothes will enhance heat radiation.

During a physical assessment, the nurse notes cutaneous fibromas and Lisch nodules (yellow elevations) on a client’s irises. What genetic condition might this client have?
1
Phenylketonuria
2
Neurofibromatosis
3
Huntington disease
4
Myotonic dystrophy
2
Cutaneous fibromas and Lisch nodules (yellow elevations on the iris) are signs of neurofibromatosis. Growth failure, frequent vomiting, irritability, hyperactivity, and erratic behavior are signs of phenylketonuria. Huntington disease is a progressive neurodegenerative disease. Muscle weakness, wasting, myotonia, and cardiac conduction abnormalities are signs of myotonic dystrophy.

A client with chronic obstructive pulmonary disease is admitted to the hospital with a tentative diagnosis of pleuritis. When caring for this client, what should the nurse do?
1
Administer opioids frequently
2
Assess for signs of pneumonia
3
Give medication to suppress coughing
4
Limit fluid intake to prevent pulmonary edema
2
Clients with pleuritic disease are prone to developing pneumonia because of impaired lung expansion, air exchange, and drainage. Opioids are contraindicated because opioids depress respirations. Coughing should not be suppressed; it enhances lung expansion, air exchange, and lung drainage. Oral fluids should be encouraged; pulmonary edema does not develop unless the client has severe cardiovascular disease.

A nurse is caring for a client who has had multiple myocardial infarctions and has now developed cardiogenic shock. Which clinical manifestation supports this diagnosis?
1
Cold, clammy skin
2
Slow, bounding pulse
3
Increased blood pressure
4
Hyperactive bowel sounds
1
The action of the sympathetic nervous system causes vasoconstriction, and as cellular and peripheral hypoperfusion progresses, the skin becomes cold, clammy, cyanotic, or mottled. The heart rate increases in an attempt to meet the body’s oxygen demands and circulate blood to vital organs; it has a low volume (weak, thready) because of peripheral vasoconstriction. The blood pressure decreases because of continued hypoperfusion and multiorgan failure. Bowel sounds are hypoactive or absent, not hyperactive.

After multiple upper respiratory infections, a school-aged child undergoes a tonsillectomy and adenoidectomy. Two weeks after surgery the nurse assesses the child’s condition. On what should the nurse focus? Select all that apply.
1
Taste
2
Smell
3
Hearing
4
Breathing
5
Facial symmetry
1234
Edematous adenoids interfere with nasal breathing, which affects the sense of taste. Enlarged adenoids usually cause mouth breathing, which affects the sense of smell. Because hearing usually is affected by repeated oropharyngeal infections, this is an important postoperative assessment. Breathing is an important postoperative assessment because one goal of a tonsillectomy and adenoidectomy is to convert mouth breathing to nasal breathing. Facial symmetry is not affected by these procedures.

Test-Taking Tip: Pace yourself while taking a quiz or exam. Read the entire question and all answer choices before answering the question. Do not assume that you know what the question is asking without reading it entirely.

After assessing a 5-year-old child, the nurse suspects the child has mumps. Which symptoms present in the child may support the nurse’s suspicion? Select all that apply.
1
Skin rash
2
Sore throat
3
Fever (100° F-104° F [37.8°-40° C])
4
Difficulty swallowing
5
Swelling of the parotid gland
35
Mumps is a viral disease that affects the parotid gland. Fever accompanied by swelling of the parotid gland is a primary symptom of mumps. A skin rash is more likely to occur with measles. Sore throat and difficulty swallowing are symptoms of diphtheria.

The mother of an 11-month-old infant reports that the baby has allergies. After an assessment, the primary healthcare provider also suspects anemia. Which questions would the primary healthcare provider most likely ask the mother? Select all that apply.
1
Do you use 2% cow’s milk?
2
Do you breastfeed?
3
Do you use whole cow’s milk?
4
Do you use alternate milk products?
5
Do you provide 18 to 21 ounces of breast milk per day?
134
The use of 2% or whole cow’s milk in an infant younger than 12 months is not recommended because it may cause intestinal bleeding, anemia, and allergic reactions. Mothers should avoid using any alternate milk products because their use may cause complications in the infant. Breast feeding is recommended for the infant’s nutrition because breast milk contains essential proteins, fats, carbohydrates, and immunoglobulins that help bolster the infant’s ability to resist infection. An average infant of one month old should have 18 to 21 ounces of breast milk or formula per day.

Test-Taking Tip: Identify option components as correct or incorrect. This may help you identify a wrong answer.

We have an expert-written solution to this problem!
A 60-year-old woman is admitted for a vagin*l hysterectomy and anterior and posterior repair of the vagin*l wall. While taking the nursing history the nurse expects the client to state that one of the reasons she is having surgery is because she has been experiencing what?
1
Hematuria
2
Dysmenorrhea
3
Pain on urination
4
Stress incontinence
4
Increased intraabdominal pressure associated with lifting, coughing, or laughing, in conjunction with a relaxed pelvic musculature and a bladder displaced into the vagin*, results in inability to suppress urination. Hematuria is usually associated with urinary tract infection, bladder tumor, or renal calculi, not with cystocele or rectocele. Dysmenorrhea is usually associated with pelvic inflammatory disease, endometriosis, or cervical stenosis, not with cystocele or rectocele; the client is probably postmenopausal. Pain on urination is usually associated with urinary infection, not with cystocele or rectocele.

STUDY TIP: A helpful method for decreasing test stress is to practice self-affirmation. After you have adequately studied and really know the material, start looking in the mirror each time you pass one and say to yourself—preferably out loud—”I know this material, and I will do well on the test.” After several times of watching and hearing yourself reaffirm your knowledge, you will gain inner confidence and be able to perform much better during the test period. This technique really works for students who are adventurous enough to use it. It may feel silly at first, but if it works, who cares? It will work for performing skills in clinical as well, as long as you have practiced the skill sufficiently.

The healthcare provider prescribes a progressive exercise program that includes walking for a client with a history of diminished arterial perfusion to the lower extremities. The nurse explains to the client what to do if leg cramps occur while walking. Which instruction did the nurse give the client?
1
Chew one aspirin twice a day.
2
Stop to rest until the pain resolves.
3
Walk more slowly while pain is present.
4
Take one nitroglycerin tablet sublingually.
2
During an exercise program, the client walks to the point of claudication, stops and rests, and then walks a little farther. Decreasing the demand for oxygen by resting will relieve the pain. Pain will not resolve as long as exercise, thus muscle hypoxia, is continued, regardless of whether aspirin is taken. Walking more slowly while pain is present is appropriate for venous insufficiency, not arterial insufficiency. Sublingual nitroglycerin is not indicated for leg cramps.

Test-Taking Tip: The computerized NCLEX exam is an individualized testing experience in which the computer chooses your next question based on the ability and competency you have demonstrated on previous questions. The minimum number of questions will be 75 and the maximum 265. You must answer each question before the computer will present the next question, and you cannot go back to any previously answered questions. Remember that you do not have to answer all of the questions correctly to pass.

A 62-year-old client reports to the nurse, “My eyes don’t feel right and I have a gritty and sandy sensation in my eyes.” What condition might this client have?
1
Retinal detachment
2
Infection of the cornea
3
Changes in tear composition
4
Hemorrhage in the vitreous humor
3
Decreased tear formation or changes in the tear composition because of aging or various systemic diseases may cause dryness in the eye. This may manifest in eye discomfort and a sandy, gritty, or burning sensation. Retinal detachment may cause blurred vision. Infection of the cornea may result in photophobia. A vitreous humor hemorrhage manifests as spots or floaters seen in the field of vision.

A nurse is assessing a client admitted to the hospital with a tentative diagnosis of a pituitary tumor. What signs of Cushing syndrome does the nurse identify?
1
Retention of sodium and water
2
Hypotension and a rapid, thready pulse
3
Increased fatty deposition in the extremities
4
Hypoglycemic episodes in the early morning
1
Increased levels of steroids and aldosterone cause sodium and water retention in clients with Cushing syndrome. Hypertension, not hypotension, is expected because of sodium and water retention. The extremities will be thin; subcutaneous fat deposits occur in the upper trunk, especially the back between the scapulae. Hyperglycemia, not hypoglycemia, occurs because of increased secretion of glucocorticoids. Hyperglycemia is sustained and not restricted to the morning hours.

Which type of hypersensitivity reaction is present in a client with a body temperature of 102 °F, severe joint pain, rashes on the extremities, and enlarged lymph nodes from serum sickness?
1
Delayed reaction
2
Cytotoxic reaction
3
Immediate reaction
4
Immune complex-mediated reaction
4
Serum sickness is a type III immune complex-mediated reaction. A delayed reaction is a type IV hypersensitivity reaction that may include poison ivy skin rashes, graft rejection, and sarcoidosis. A cytotoxic reaction is a type II hypersensitivity reaction that includes autoimmune hemolytic anemia, Goodpasture syndrome, and myasthenia gravis. An immediate reaction is a type I hypersensitive reaction that includes allergic asthma, hay fever, and anaphylaxis.

Test-Taking Tip: Serum sickness is manifested by elevated body temperature, severe joint pain, rashes on limbs, and enlarged lymph nodes. Recall the type of hypersensitivity reaction to which serum sickness belongs.

We have an expert-written solution to this problem!
Which nursing action is important when suctioning the secretions of a client with a tracheostomy?
1
Use a new sterile catheter with each insertion.
2
Initiate suction as the catheter is being withdrawn.
3
Insert the catheter until the cough reflex is stimulated.
4
Remove the inner cannula before inserting the suction catheter.
2
During suctioning of a client’s secretions, negative pressure (suction) should not be applied until the catheter is ready to be drawn out because, in addition to the removal of secretions, oxygen is being depleted. The sterility of the catheter can be maintained during one suctioning session; a new sterile catheter should be used for each new session of suctioning. A cough reflex may be absent or diminished in some clients; the catheter should be inserted approximately 12 cm (4 to 5 inches) or just past the end of the tracheostomy tube. The inner cannula is not removed during suctioning; it may be removed during tracheostomy care.

We have an expert-written solution to this problem!
A newborn is being treated with phototherapy for hyperbilirubinemia. What is the nurse’s role when providing phototherapy?
1
Turning the infant every 2 hours
2
Measuring the bilirubin level every 2 hours
3
Maintaining the infant on daily 24-hour phototherapy
4
Applying a sterile gauze pad to the infant’s umbilical stump
1
The infant’s position is changed every 2 hours to expose all skin surfaces to the phototherapy for maximum effect. Measuring the bilirubin level every 2 hours is not necessary. The infant may be removed from the lights for feeding and the eye patches removed to assess the eyes for irritation. The lights will dry the cord more quickly, which is a desirable effect.

Test-Taking Tip: After you have eliminated one or more choices, you may discover that two of the options are very similar. This can be very helpful, because it may mean that one of these look-alike answers is the best choice and the other is a very good distractor. Test both of these options against the stem. Ask yourself which one completes the incomplete statement grammatically and which one answers the question more fully and completely. The option that best completes or answers the stem is the one you should choose. Here, too, pause for a few seconds, give your brain time to reflect, and recall may occur.

A 16-year-old client has a blood pressure reading of 119/75. What is the approximate pulse pressure? Record your answer using a whole number. __ mm Hg
The difference between the systolic and diastolic pressure is called the pulse pressure. The given blood pressure is 119/75. The difference between 119 and 75 is 44.

A cast is applied to the involved extremity of an infant with talipes equinovarus (clubfoot). How often does the nurse tell the parents to bring their baby back to the clinic for a cast change?
1
Each week
2
Once a month
3
When the cast edges fray
4
If the cast becomes soiled
1
Casts are changed weekly to accommodate the rapid growth of early infancy. Once a month is not frequent enough in early infancy; the cast may become too tight because of the infant’s rapid growth. The cast is not on the foot long enough for fraying to occur. Soiling is usually not a problem, because casts for clubfoot do not extend to the perineal area.

We have an expert-written solution to this problem!
The nurse is caring for a client who is on a low-carbohydrate diet. With this diet, there is decreased glucose available for energy and fat is metabolized for energy, resulting in an increased production of which substance in the urine?
1
Protein
2
Glucose
3
Ketones
4
Uric acid
3
As a result of fat metabolism, ketone bodies are formed, and the kidneys attempt to decrease the excess by filtration and excretion. Excessive ketones in the blood can cause metabolic acidosis. A low-carbohydrate diet does not cause increased protein, glucose, or uric acid in the urine.

STUDY TIP: The old standbys of enough sleep and adequate nutritional intake also help keep excessive stress at bay. Although nursing students learn about the body’s energy needs in anatomy and physiology classes, somehow they tend to forget that glucose is necessary for brain cells to work. Skipping breakfast or lunch or surviving on junk food puts the brain at a disadvantage.

A nurse is assessing a toddler with the diagnosis of lead poisoning. What is the most harmful adverse effect that the nurse anticipates?
1
Inadequate nutrition
2
Delayed development
3
Anemia and constipation
4
Renal and skeletal damage
2
Irreversible neurologic and intellectual damage is the most serious consequence of lead poisoning, the result of cortical atrophy and encephalopathy. Although there may be a nutritional deficit, it is not the priority. Anemia and constipation, as well as renal and skeletal damage, do occur, but they are reversible.

A nurse is admitting a 2-year-old toddler who ingested half of a bottle of aspirin tablets to the emergency department. What is the origin of the metabolic acidosis caused by aspirin toxicity?
1
Deep rapid breathing
2
Higher pH of gastric contents
3
Rapid absorption of salicylate
4
Increased renal excretion of bicarbonate
3
Rapid absorption of acetylsalicylic acid (aspirin) causes the stomach contents to become more acidic, leading to metabolic acidosis. Hyperventilation is the body’s attempt to blow off excess hydrogen ions; carbon dioxide is converted to hydrogen ions by way of the carbonic anhydrase reaction. The pH of the stomach contents decreases with aspirin toxicity, becoming more acidic, resulting in metabolic acidosis. Although increased renal excretion of bicarbonate can contribute to metabolic acidosis, this is not the mechanism associated with aspirin toxicity. In metabolic acidosis associated with aspirin toxicity the kidneys attempt to decrease the renal excretion of bicarbonate.

While completing an assessment, the nurse finds that a client has decreased thickness and excessive dryness of the epidermis. Which clinical finding is associated with this skin assessment?
1
Skin tears
2
Skin cancer
3
Skin fragility
4
Skin hyperplasia
3
The nurse may assess excessive skin dryness due to decreased epidermal thickness. It is associated with skin transparency and fragility. Skin tears may occur due to the flattening of the dermal- epidermal junction. Decreased mitotic homeostasis in the epidermis may cause skin cancer. Skin hyperplasia may occur due to sun-induced changes that cause a decrease in mitotic homeostasis.

Test-Taking Tip: Identifying content and what is being asked about that content is critical to your choosing the correct response. Be alert for words in the stem of the item that are the same or similar in nature to those in one or two of the options.

We have an expert-written solution to this problem!
A client presents to the emergency department with weakness, dizziness, and difficulty breathing. The nurse performs an electrocardiogram (ECG) and notices this arrhythmia. Which arrhythmia is the client exhibiting?
1
Atrial fibrillation (AF)
2
Ventricular tachycardia (VT)
3
Junctional tachycardia
4
Supraventricular tachycardia (SVT)
1
AF can be chronic or intermittent. Note the wavy baseline with uncoordinated atrial electrical activity and irregular ventricular rhythm. AF clients who have valvular disease are particularly at risk for venous thromboembolism (VTE). Symptoms depend upon the ventricular rate and, if rapid, the client can experience fatigue, weakness, shortness of breath, dizziness, anxiety, syncope, palpitations, chest discomfort, and hypotension. Clients should be monitored carefully for these complications. Some clients can be asymptomatic. VT is a rapid ventricular rate typically between 140 and 250 beats per min and characterized by wide bizarre QRS complexes. Clients exhibiting junctional tachycardia may have no P waves or inverted P waves and a rate greater than 100. SVT involves the rapid stimulation of atrial tissue at a rate of over 150 beats per min in adults. During SVT, P waves may not be visible, especially if there is a 1:1 conduction with rapid rates because the P waves are embedded in the preceding T wave. SVT may occur in healthy young people, especially women.

A client is admitted for repair of bilateral inguinal hernias. Before surgery the nurse assesses the client for indicators that strangulation of the intestine may have occurred. What is an early indicator of strangulation?
1
Increased flatus
2
Projectile vomiting
3
Sharp abdominal pain
4
Decreased bowel sounds
3
Pain is wavelike, colicky, and sharp because of obstruction and localized bowel ischemia. Flatus is impeded by strangulation. Vomiting is persistent, not projectile. Decreased bowel sounds are not an early sign of obstruction; decreased bowel sounds occur after gas and fluid accumulate.

STUDY TIP: Record the information you find to be most difficult to remember on 3″ × 5″ cards and carry them with you in your pocket or purse. When you are waiting in traffic or for an appointment, just pull out the cards and review again. This “found” time may add points to your test scores that you have lost in the past.

A client reports swelling of the scrotum with no pain. Which condition does the nurse anticipate in the client?
1
Chancroid
2
Hydrocele
3
Spermatocele
4
Incarcerated hernia
2
Unilateral or bilateral swelling of the scrotum without pain is a clinical manifestation of a hydrocele; therefore, this is what the nurse anticipates the client to have. Chancroid is manifested by papular or irregularly shaped nonindurated, pus-filled ulcers on the penis. A translucent, wormlike appearance of the scrotum is a characteristic of spermatocele. Swelling and pain in the scrotum are the clinical manifestations of incarcerated hernia.

Test-Taking Tip: Identify option components as correct or incorrect. This may help you identify a wrong answer.

When monitoring fluids and electrolytes, the nurse recalls that the major cation-regulating intracellular osmolarity is what?
1
Sodium
2
Potassium
3
Calcium
4
Calcitonin
2
A decrease in serum potassium causes a decrease in the cell wall pressure gradient and results in water moving out of the cell. Besides intracellular osmolarity regulation, potassium also regulates metabolic activities, transmission and conduction of nerve impulses, cardiac conduction, and smooth and skeletal muscle contraction. Sodium is the most abundant extracellular cation that regulates serum osmolarity as well as nerve impulse transmission and acid-base balance. Calcium is an extracellular cation necessary for bone and teeth formation, blood clotting, hormone secretion, cardiac conduction, transmission of nerve impulses, and muscle contraction. Calcitonin is a hormone secreted by the thyroid gland and works opposite of parathormone to reduce serum calcium and keep calcium in the bones. Calcitonin does not have a direct effect on intracellular osmolarity.

A nurse obtains the history of a client with early colon cancer. Which clinical finding does the nurse consider consistent with a diagnosis of cancer of the descending, rather than the ascending, colon?
1
Pain
2
Fatigue
3
Anemia
4
Obstruction
4
Signs and symptoms of obstruction occur earlier with cancer in the descending colon because the consistency of the stool is formed rather than liquid. Pain, a late symptom of colon cancer, may occur regardless of the location of the primary lesion. Fatigue occurs in colon cancer regardless of the primary site; it is related to anorexia, weight loss, and anemia. Bleeding, which results in anemia, occurs in colon cancer regardless of the primary site because the lesions extend into the intestinal mucosa.

A registered nurse (RN) must assess the body temperature of a client with a history of epilepsy. Which site for measuring temperature is contraindicated in this client?
1
Skin
2
Axilla
3
Oral cavity
4
Temporal artery
3
The oral cavity is not a preferred site to measure the body temperature of a client with epilepsy, oral surgery, trauma, or shaking chills. Epileptic clients become rigid during seizures and any sudden seizure attack during temperature measurement poses the risk of breaking the thermometer in the mouth, lacerations, accidental mercury ingestion, and possibly aspirating the broken pieces. The skin, axilla, and temporal artery are sites that can be safely used to measure topical body temperature in an epileptic client.

Which type of joint is present in between the client’s tarsal bones?
1
Pivot joint
2
Hinge joint
3
Saddle joint
4
Gliding joint
4
The gliding joint is present in between the tarsal bones. The pivot joint is present in the proximal radioulnar joint. The hinge joint is present in the elbows and knees. The saddle joint is present in between the carpometacarpal joints of the thumb.

The nurse is assessing a client with severe nodule-forming rheumatoid arthritis for possible Felty syndrome. Which assessment findings are consistent with Felty syndrome? Select all that apply.
1
Itchy eyes
2
Dry mouth
3
Leukopenia
4
Splenomegaly
5
Photosensitivity
34
Felty syndrome occurs most commonly in clients with severe nodule-forming rheumatoid arthritis; it is characterized by splenomegaly and leukopenia. Itchy eyes, dry mouth, and photosensitivity are all signs of Sjögren syndrome.

Which criteria in a client must be assessed in order to diagnose premenstrual dysphoric disorder (PMDD)? Select all that apply.
1
Symptom-free period in the luteal phase
2
Symptoms that resolve within a few days of menses onset
3
Symptoms caused by the exacerbations of other disorder
4
Symptoms such as irritability, anxiety, or depressed mood
5
Symptoms that interfere with work or interpersonal relationships
45
Premenstrual dysphoric disorder (PMDD) is a variant of premenstrual syndrome. Among the symptoms are irritability, depressed mood, and anxiety. These symptoms would markedly interfere with work or interpersonal relationships. These symptoms are absent in the follicular phase. At least five or more symptoms are present the week before menses; these symptoms resolve within a few days of menses onset if the client has premenstrual syndrome. These symptoms are not caused by an exacerbation of another condition or disorder.

What nursing action will most help a client obtain maximum benefits after postural drainage?
1
Administer oxygen as needed.
2
Encourage coughing deeply.
3
Place the client in a sitting position.
4
Encourage the client to rest for a half hour.
2
Coughing is needed to raise secretions for expectoration. Oxygen will not mobilize the secretions. A sitting position will allow secretions to remain in the lungs unless coughing is encouraged. Rest should be encouraged only after coughing to bring up secretions mobilized by postural drainage.

A client with a traumatic brain injury is demonstrating signs of increasing intracranial pressure, which may exert pressure on the medulla. What should the nurse assess to determine involvement of the medulla? Select all that apply.
1
Taste
2
Breathing
3
Heart rate
4
Fluid balance
5
Voluntary movement
23
The medulla, part of the brainstem just above the foramen magnum, is concerned with vital functions such as breathing. The medulla is concerned with vital functions [1] [2] such as heart rate. The opercular-insular area of the parietal cerebral lobe is concerned with taste sensations. The medulla is not concerned with fluid balance. Osmoreceptors of the hypothalamus cause increased or decreased antidiuretic hormone (ADH) secretion depending on serum osmolarity. Voluntary movements are mediated through the somatomotor area of the cerebral cortex.

A client with liver dysfunction reports bleeding gums. In addition, the nurse observes small facial hemorrhagic lesions. What should the nurse conclude that the client needs?
1
Vitamin C
2
Folic acid
3
Vitamin A
4
Vitamin K
4
Petechiae represent evidence of capillary bleeding; the diseased liver is no longer able to metabolize vitamin K, a process that is necessary to activate blood clotting factors. Although vitamin C does help capillary stability, it is not associated with liver dysfunction. Folic acid is stored in the liver but is not involved in the blood clotting process. Vitamin A is not involved in blood clotting, even though the transformation of carotene to vitamin A takes place in the liver.

We have an expert-written solution to this problem!
Endotracheal intubation and positive-pressure ventilation are instituted because of a client’s deteriorating respiratory status. Which is the priority nursing intervention?
1
Facilitate verbal communication
2
Prepare the client for emergency surgery
3
Maintain sterility of the ventilation system
4
Assess the client’s response to the mechanical ventilation
4
The effectiveness of therapy is measured by the client’s response. Endotracheal intubation does not permit verbal communication. Preparing the client for emergency surgery is presumptive; the database is inadequate for this conclusion. Maintaining sterility of the ventilation system is important but not the priority; the client is a higher priority than equipment.

The laboratory report of a client reveals increased levels of atrial natriuretic peptide. Which other finding does the nurse anticipate to find in the client?
1
Decreased urine output
2
Increased concentration of urine
3
Increased sodium excretion in urine
4
Decreased glomerular filtration rate
3
Atrial natriuretic peptide is secreted by the myocyte cells in the right atrium. Atrial natriuretic peptide acts on the kidneys and causes an increase in the excretion of sodium by inhibiting aldosterone. Atrial natriuretic peptide increases urine output. Atrial natriuretic peptide causes inhibition of renin and angiotensin II, and therefore the resultant urine produced contains more water and is dilute. Because atrial natriuretic peptide relaxes the afferent arteriole in the nephron, glomerular filtration rate is increased.

The nurse suspects the Jarisch-Herxheimer reaction in a client with syphilis who is on antibiotic therapy. Which symptoms in the client support the nurse’s suspicion? Select all that apply.
1
Fever
2
Hypertension
3
Vasoconstriction
4
Generalized ache
5
Pain at the injection site
145
Fever, generalized ache, and pain at the injection site are signs of the Jarisch-Herxheimer reaction in a client with syphilis receiving antibiotic therapy. This reaction is caused by the rapid release of products from the disruption of cells of the organism. Hypotension because of vasodilatation and declining peripheral resistance, not hypertension and vasoconstriction, are additional signs of Jarisch-Herxheimer reaction.

A client is admitted to the hospital with atrial fibrillation. A diagnosis of mitral valve stenosis is suspected. The nurse concludes that it is most significant if the client presents with what history?
1
Cystitis as an adult
2
Pleurisy as an adult
3
Childhood strep throat
4
Childhood German measles
3
Streptococcal infections occurring in childhood may result in damage to heart valves, particularly the mitral valve. Group A streptococcal antigens bind to receptors on heart cells, where an autoimmune response is triggered damaging the heart. Cystitis usually is caused by Escherichia coli, which does not affect heart valves. Pleurisy usually follows pulmonary problems unrelated to streptococcal infection; it does not result in damage to heart valves. The rubella virus does affect the valves of the heart.

A diabetic client survives a fire. The client has a head injury, tachycardia, and pale and ashen-colored skin. Which primary interventions should the nurse perform? Select all that apply.
1
Administering cool intravenous fluids
2
Placing the client in a cool environment
3
Managing and maintaining airway, breathing, and circulation
4
Monitoring heart rhythm, oxygen saturation, and urine output
5
Monitoring temperature, vital signs, and level of consciousness
123
The diabetic client with a head injury displaying tachycardia along with pale and ashen skin has symptoms of heat exhaustion. This is a medical emergency and needs immediate intervention. The intervention for this includes administering cool intravenous (IV) fluids and placing the client in a cool environment. Managing and maintaining airway, breathing, and circulation should be performed immediately to prevent life-threatening risks. Monitoring heart rhythm, oxygen saturation, and urine output is a secondary intervention. Monitoring temperature, vital signs, and level of consciousness is also a secondary intervention.

The nurse is caring for a 1-hour-old newborn. Which assessment characteristics represent a preterm gestational age?
1
Skin: thin, veins visible; breasts: flat areolae, no buds; plantar creases: absent; lanugo: abundant
2
Skin: parchment/wrinkled; breasts: flat areolae, no buds; plantar creases: cover entire sole; lanugo: absent
3
Skin: thin, veins visible; breasts: flat areolae, no buds; plantar creases: covering the entire sole; lanugo: abundant
4
Skin: cracking/few veins; breasts: raised areolae (3- to 4-mm buds); plantar creases: covering the anterior two thirds of the sole; lanugo: thinning
1
The characteristics of preterm, term, and postterm gestational age are based on assessments of physical maturity such as the Ballard or Dubowitz assessment. A preterm infant’s skin is translucent, with many visible veins. A term infant has some cracking of the skin and some visible veins, depending on gestational age. Term is any gestation after 38 weeks; veins are less visible at 40 weeks’ gestation. The postterm infant typically has dry, leathery, parchmentlike skin with numerous deep wrinkles. The areolae of a preterm infant are flat, without buds, and they become more raised during development, averaging 3 to 4 mm at term and 5 to 10 mm in the postterm infant. The plantar creases develop on the foot during gestation, beginning smooth, then covering two thirds at term, and finally covering the entire sole after term. Lanugo is the fine downy hair that diminishes as the infant develops gestationally.

Which is a clinical manifestation of the Landouzy-Déjérine type of muscular dystrophy (MD)?
1
Loss of hearing
2
Cardiomyopathy
3
Respiratory failure
4
Mental impairment
1
Loss of hearing is the clinical manifestation of Landouzy-Déjérine MD. Cardiomyopathy and respiratory failure are the clinical manifestations of both duch*enne and Becker MD. duch*enne MD is clinically manifested by mental impairment.

The nurse is caring for a client who had a colostomy 2 days ago. Which nursing intervention is the priority?
1
Keeping an accurate record of oral fluid intake
2
Emphasizing the importance of regulating the diet to form stool
3
Teaching care of the incision and how to perform colostomy irrigations
4
Observing for drainage and the condition of the abdominal stoma
4
Because of the recent trauma of surgery, hemorrhage and infection at the operative site can occur. The client will have nothing by mouth until peristalsis returns; there might not be any oral intake, and it is not the priority intervention. Emphasizing the importance of regulating the diet to form stool is inappropriate at this time; observing for bleeding and infection takes priority during the first 48 hours. It is not the appropriate time for teaching care of the incision and how to perform colostomy irrigations.

The family of a client with right ventricular heart failure expresses concern about the client’s increasing abdominal girth. What physiologic change should the nurse consider when explaining the client’s condition?
1
Loss of cellular constituents in blood
2
Rapid osmosis from tissue spaces to cells
3
Increased pressure within the circulatory system
4
Rapid diffusion of solutes and solvents into plasma
3
Failure of the right ventricle causes an increase in pressure in the systemic circulation. To equalize this pressure, fluid moves into the tissues, causing edema, and into the abdominal cavity, causing ascites; ascites leads to an increased abdominal girth. There is no loss of the cellular constituents in blood with right ventricular heart failure. Ascites is the accumulation of fluid in an extracellular space, not intracellular. The opposite of rapid diffusion of solutes and solvents into plasma results when there is a pressure increase in the systemic circulation.

The nurse is caring for a client who had a thyroidectomy. Which symptoms will the client exhibit if having a thyrotoxic crisis?
1
An increased pulse deficit
2
A decreased blood pressure
3
A decreased heart rate and respirations
4
An increased temperature and pulse rate
4
Thyrotoxic crisis is severe hyperthyroidism; excessive amounts of thyroxine increase the metabolic rate, thereby raising the pulse and temperature. During crisis there usually is no increase in the difference between the apical and the peripheral pulse rates (pulse deficit). The blood pressure will increase to meet the oxygen demand caused by the increased metabolic rate during crisis. Because of the increased metabolic rate, the pulse and respiratory rates increase to meet the body’s oxygen needs.

We have an expert-written solution to this problem!
The nurse teaching a health awareness class identifies which situation as being the highest risk factor for the development of a deep vein thrombosis (DVT)?
1
Pregnancy
2
Inactivity
3
Aerobic exercise
4
Tight clothing
2
A DVT, or thrombus, may form as a result of venous stasis. It may lodge in a vein and can cause venous occlusion. Inactivity is a major cause of venous stasis leading to DVT. Pregnancy and tight clothing are also risk factors for DVT secondary to inactivity. Aerobic exercise is not a risk factor for DVT.

The nurse caring for a 3-year-old child with meningitis should be alert for which signs and symptoms of increased intracranial pressure? Select all that apply.
1
Vomiting
2
Headache
3
Irritability
4
Tachypnea
5
Hypotension
123
Increased intracranial pressure can precipitate vomiting because of its effect on the chemoreceptor trigger zone in the medulla. Because the cranial sutures are closed by this age, increased pressure can cause headache. Irritability results from increased pressure in the cranium and as a response to related discomforts. Pressure on the respiratory center in the brain results in a decreased, not increased, respiratory rate. Blood pressure is increased, not decreased, in the toddler with closed fontanels.

The left foot of a client with a history of intermittent claudication becomes increasingly cyanotic and numb. Gangrene of the left foot is diagnosed, and because of the high level of arterial insufficiency, an above-the-knee amputation (AKA) is scheduled. Which response by the client best demonstrates emotional readiness for the surgery?
1
Explains the goals of the procedure
2
Displays few signs of anticipatory grief
3
Participates in learning perioperative care
4
Verbalizes acceptance of permanent dependency needs
3
Active participation in self-care indicates a readiness to learn; it demonstrates that the client is interested in future expectations. Explaining the goals of the procedure may indicate intellectual readiness but not necessarily emotional readiness. An expected change in body image precipitates the grieving process; a client may be in denial if no concerns are expressed. The client need not be dependent permanently; verbalizing acceptance of permanent dependency needs indicates the need for more teaching and emotional support.

A nurse assesses a client who had a gastric resection. During the first 24 hours after surgery, what symptom should the nurse expect to identify?
1
Vomiting
2
Gastric distention
3
Intermittent periods of diarrhea
4
Bloody nasogastric drainage
4
Drainage is bright red initially and gradually becomes darker red during the first 24 hours. If the nasogastric tube is functioning correctly, secretions will be removed and vomiting will not occur. If the nasogastric tube is functioning correctly, gastric distention will not occur. Because the bowel was emptied before surgery and the client is now nothing by mouth, intestinal activity is not expected.

A nurse is caring for a client who experienced a crushing chest injury. A chest tube is inserted. Which observation indicates a desired response to this treatment?
1
Increased breath sounds
2
Increased respiratory rate
3
Crepitus detected on palpation of the chest
4
Constant bubbling in the drainage collection chamber
1
The chest tube normalizes intrathoracic pressure, drains fluid and air from the pleural space, and improves pulmonary function [1] [2]. Increased respiratory rate may be a sign of pain, respiratory obstruction, or bleeding. Crepitus detected on palpation of the chest indicates that air has entered the subcutaneous tissue (subcutaneous emphysema). Constant bubbling in the drainage collection chamber indicates a probable leak in the drainage system.

We have an expert-written solution to this problem!
A client is admitted for treatment of partial- and full-thickness burns of the entire right lower extremity and the anterior portion of the right upper extremity. The nurse performs an immediate appraisal, using the rule of nines. Which percentage of body surface area burned will the nurse record?
1
18%
2
22.5%
3
27%
4
36.5%
2
Based on the rule of nines, the entire right lower extremity is 18%, and the anterior portion of the right upper extremity is 4.5%; 18 + 4.5 = 22.5. The calculation 18% is too small a percentage based on the rule of nines; 27% and 36.5% are too large.

A client with bubonic plague has a body temperature of 103° F associated with chills, swollen glands, headache, and weakness. Which microorganism is most likely responsible for the client’s condition?
1
Yersinia pestis
2
Bordetella pertusis
3
Mycobacterium tuberculosis
4
Corynebacterium diphtheria
1
Bubonic plague is a reemerging infection caused by Yersinia pestis, which is associated with elevated body temperature, chills, swollen glands, headache, and weakness. Bordetella pertusis causes pertussis, which is associated with acute, highly contagious respiratory disease characterized by loud whooping cough. Mycobacterium tuberculosis causes tuberculosis, which is transmitted by inhalation of infected droplets. Corynebacterium diphtheria causes a localized infection of mucous membranes or skin such as diphtheria.

We have an expert-written solution to this problem!
Which statements about acne in adolescents are true? Select all that apply.
1
Early acne occurs in the midface region.
2
Acne is more common in girls than boys.
3
Acne usually occurs in middle to late adolescence.
4
Intake of dairy products can contribute to acne severity.
5
Acneiform eruptions are predominant in young children.
134
Acne occurs in more than 50% of adolescents in middle to late adolescence. Early acne occurs in the midface region, such as the chin, nose, and middle of the forehead. The intake of dairy products may contribute to acne severity. Acne is more common in boys than in girls. Acneiform eruptions occur in infants, neonates, and young children.

We have an expert-written solution to this problem!
A client’s cardiac monitor shows a PQRST wave for each beat and indicates a rate of 120 beats per minute. The rhythm is regular. The nurse concludes that the client is experiencing what?
1
Atrial fibrillation
2
Sinus tachycardia
3
Ventricular fibrillation
4
First-degree atrioventricular block
2
The presence of a P wave before each QRS complex indicates a sinus rhythm; a heart rate greater than 100 regular beats per minute is referred to as tachycardia. Atrial fibrillation has no well-defined P waves, there are 350 or more beats per minute, there are random ventricular beats, and the rhythm is irregular. Ventricular fibrillation is irregular and shows no PQRST configurations. A first-degree atrioventricular block pattern has a prolonged PR interval and is regular.

A client newly diagnosed with scleroderma states, “Where did I get this from?” How should the nurse reply?
1
“The exact cause is unknown, but it is thought to be a result of autoimmunity.”
2
“The exact cause is unknown, but it is thought to be a result of ocular motility.”
3
“The exact cause is unknown, but it is thought to be a result of increased amino acid metabolism.”
4
“The exact cause is unknown, but it is thought to be a result of defective sebaceous gland formation.”
1
Scleroderma is an immunologic disorder characterized by inflammatory, fibrotic, and degenerative changes. Ocular motility, increased amino acid metabolism, and defective sebaceous gland formation are not involved in the development of scleroderma.

Place the pathophysiologic process of tuberculosis infection in its correct order.
1.
Necrotic areas calcify or liquefy.

2.
Caseation necrosis occurs in the center of the lesion.

3.
Areas of caseation undergo resorption, degeneration, and fibrosis.

4.
Granulomatous inflammation is created by tuberculosis bacillus in lungs.

5.
Granulomatous inflammation becomes surrounded by collagen, fibroblasts, and lymphocytes.
Tuberculosis is a highly communicable disease caused by Mycobacterium tuberculosis. The process of infection in tuberculosis starts by formation of granulomatous inflammation by tuberculosis bacillus in lungs. This granulomatous inflammation then becomes surrounded by collagen, fibroblasts, and lymphocytes. The necrotic tissue then turns into a granular mass, called a caseation necrosis, which occurs in the center of the lesion. Then the areas of caseation undergo resorption, degeneration, and fibrosis. Finally, the necrotic areas undergo calcification or liquefaction.

After surgery for repair of a myelomeningocele, the nurse places the infant in a side-lying position with the head slightly elevated. What is the main reason the nurse places the infant in this position after this particular surgery?
1
To prevent aspiration
2
To promote respiration
3
To reduce intracranial pressure
4
To maintain cleanliness of the suture site
3
The side-lying position with the head slightly elevated promotes venous return by gravity, which helps reduce intracranial pressure, a problem after myelomeningocele repair. Although preventing aspiration, promoting respiration, and maintaining cleanliness of the suture line are all important, the reason for this position that is unique with this type of surgery is that it minimizes intracranial pressure.

A client is admitted to the postanesthesia care unit after a segmental resection of the right lower lobe of the lung. A chest tube drainage system is in place. When caring for this tube, what should the nurse do?
1
Raise the drainage system to bed level and check its patency
2
Clamp the tube when moving the client from the bed to a chair
3
Mark the time and fluid level on the side of the drainage chamber
4
Secure the chest catheter to the wound dressing with a sterile safety pin
3
The fluid level and time must be marked so that the amount of drainage in the chest tube drainage system can be evaluated. The drainage system must be kept below chest level to promote drainage of the pleural space so the lungs can expand. Clamping the tube can produce backpressure, which may cause fluid to move into the pleural space from which it came, producing a tension pneumothorax. The catheter is secured by skin sutures, not to a dressing with a safety pin.

We have an expert-written solution to this problem!
The parents of a toddler with newly diagnosed cystic fibrosis ask a nurse what causes the problems related to this disorder. What should the nurse consider about the primary pathologic process before responding?
1
Hyperactivity of the eccrine (sweat) glands
2
Hypoactivity of the autonomic nervous system
3
Mechanical obstruction of mucus-secreting glands
4
Atrophic changes in the mucosal lining of the intestines
3
Mucous secretions increase in viscosity and precipitate or coagulate to form concentrations in glands and ducts, resulting in obstructions. Decreased amounts of pancreatic enzymes cause impairment in the digestion and absorption of nutrients. The eccrine (sweat) glands are not hyperactive, but there is an increased concentration of sweat electrolytes (e.g., sodium and chloride). The autonomic nervous system does not play a role in the pathologic process of cystic fibrosis. There is no alteration in the mucosal lining of the intestines.

A client is admitted with paresis of the ciliary muscles of the left eye. What function should the nurse expect to be affected?
1
Closing the eyelids
2
Convergence of both eyes
3
Ability to discriminate colors
4
Focusing the lens on near objects
4
The contraction of the ciliary muscles permits the lens to return to its normal bulge and decreases focal length, promoting the ability to focus on near objects. The ciliary muscles are intrinsic (within the eyeball); the third cranial nerve (oculomotor), an extrinsic nerve, controls some movements of the eyelid. The rectus and oblique muscles of the eye are involved in convergence. Color blindness is an inherited trait.

A nurse is assessing an 18-month-old toddler with suspected developmental dysplasia of the left hip. In what position should the nurse place the toddler to elicit the Trendelenburg sign?
1
Standing on the affected leg
2
Supine with the back arched
3
Side-lying on the unaffected side
4
Sitting upright with the legs separated
1
When the child is standing and bearing weight on the affected hip, the pelvis tilts downward instead of upward—the Trendelenburg sign. The supine, side-lying, and sitting positions do not accomplish the desired effect because weight bearing is needed to tilt the pelvis.

While assessing the client’s skin, a nurse notices a skin condition, the pathophysiology of which involves increased visibility of oxyhemoglobin caused by an increased blood flow due to capillary dilation. Which condition is associated with this client?
1
Pallor
2
Vitiligo
3
Cyanosis
4
Erythema
4
Erythema occurs due to an increased visibility of oxyhemoglobin, which is caused by increased blood flow. Pallor is caused by a reduced amount of oxyhemoglobin or a reduced visibility of oxyhemoglobin. Vitiligo is a pigmentation disorder caused by autoimmune diseases. Cyanosis is a bluish discoloration of the skin around the lips; this occurs due to an increased amount of deoxygenated hemoglobin in the blood.

A pediatric client reports difficulty breathing and swallowing and has a sore throat, headache, and fever. The nurse observes a grayish yellow membranous patch near the tonsils. What disease does the nurse infer from these findings?
1
Mumps
2
Pertussis
3
Diphtheria
4
Pneumonia
3
Diphtheria is caused by Corynebacterium diphtheriae. Symptoms include a sore throat, fever, and headache along with gray or dirty-yellow membrane patches. Mumps is a viral disease that primarily affects the parotid glands. It is accompanied by local pain, tenderness, fever, and swelling. Pertussis, also known as whooping cough, is caused by Bordetella pertussis. Symptoms include rhinorrhea, mild fever, and a persistent cough. Pneumonia is a lung disease; symptoms include a cough and chest pain.

A client who sustained burn injuries due to a fire and explosion has a carbon monoxide level of 14%. Which pathophysiologic risk is increased in the client?
1
Stupor
2
Vertigo
3
Convulsions
4
Slight breathlessness
4
Slight breathlessness may occur when the carbon monoxide level is 14%. Stupor and vertigo may result when the carbon monoxide level is in between 21% and 40%. When the level of carbon monoxide reaches between 41% and 60%, coma or convulsions may occur.

A client is admitted to the birthing unit with uterine tenderness and minimal dark-red vagin*l bleeding. She has a marginal abruptio placentae. The priority assessment includes fetal status, vital signs, skin color, and urine output. What additional assessment is essential?
1
Fundal height
2
Obstetric history
3
Time of the last meal
4
Family history of bleeding disorders
1
It is vital that a baseline measurement be obtained, because increasing fundal height may be a sign of concealed hemorrhage. Taking an obstetric history, ascertaining the time of the last meal, and asking about a family history of bleeding disorders are all appropriate assessments; however, none are a priority at this critical time.

During a newborn assessment the nurse identifies the absence of the red reflex in the eyes. What should the nurse’s next action be?
1
Rinse the eyes with sterile saline
2
Notify the primary healthcare provider
3
Expect edema to subside within a few days
4
Conclude that this is a result of the prescribed eye prophylaxis
2
An absence of the red reflex may be indicative of congenital cataracts. The red reflex is elicited by shining the light of an ophthalmoscope into the newborn’s eyes, which should produce a reddish circle. Rinsing the eyes will not affect the red reflex. The red reflex or its absence is not related to edema, which may occur after eye prophylaxis, or to eye prophylaxis itself.

A client with chronic kidney disease is receiving medication to manage anemia. Which primary goal should the nurse include in the care plan from this information?
1
Prevention of uremic frost
2
Prevention of chronic fatigue
3
Prevention of tubular necrosis
4
Prevention of dependent edema
2
Kidney failure [1] [2] results in impaired erythropoietin production, which causes anemia and chronic fatigue; treating the anemia will help in managing the fatigue. Uremic frost results because urea compounds and other waste products of metabolism that are not excreted by the kidneys are brought to the skin by small superficial capillaries and are excreted and deposited on the skin. Tubular necrosis is a pathologic condition of the kidneys that can lead to kidney failure. The anemia and dependent edema associated with kidney failure are not interrelated.

A client developed acute herpes zoster and was treated with antiviral medication within 72 hours of the appearance of the rash. The client is reporting persistent pain 1 week later. What does the nurse identify as the cause of the posttherapeutic neuralgia?
1
Damage to the nerves
2
Untreated major depression
3
Scarring in the area of the rash
4
Continued presence of the skin rash
1
After the original infection has healed, the virus either remains quiescent or it may return. Posttherapeutic neuralgia, which occurs in some individuals, results from damage to the nerves caused by the varicella-zoster virus; the neuralgia may last for months. Untreated major depression and scarring in the area of the rash are unrelated to posttherapeutic neuralgia. The rash does not cause posttherapeutic neuralgia.

Which clients should be considered for assessing the carotid pulse? Select all that apply.
1
Client with cardiac arrest
2
Client indicated for Allen test
3
Client under physiologic shock
4
Client with impaired circulation to foot
5
Client with impaired circulation to hand
13
Carotid pulse is indicated in clients with physiologic shock or cardiac arrest when other sites are not palpable in the client. Assessment of the ulnar pulse is indicated in clients requiring an Allen test. Assessment of posterior tibial pulse and dorsalis pedis pulse is indicated in clients with impaired circulation to the feet. Assessment of the radial and ulnar pulse is indicated in clients with impaired circulation to the hands.

We have an expert-written solution to this problem!
The mother of an infant with Down syndrome asks the nurse what causes the disorder. Before responding, the nurse recalls that the genetic factor of Down syndrome results from what?
1
An intrauterine infection
2
An X-linked genetic disorder
3
Extra chromosomal material
4
An autosomal recessive gene
3
Down syndrome (trisomy 21) results from extra chromosomal material on chromosome 21. Down syndrome does not result from a maternal infection. Down syndrome is not related to an X-linked or Y-linked gene. An autosomal recessive gene is not the cause of Down syndrome, although translocation of chromosomes 15 and 21 or 22 is a genetic aberration found in some children with Down syndrome.

The nurse is providing postoperative care to a client on the second day after the client had a coronary artery bypass surgery. When assessing the water-seal chamber of the chest drainage device, the nurse observes that the fluid no longer fluctuates. What should the nurse do?
1
Assess for obstructions in the chest tube
2
Increase the amount of continuous suction
3
Add sterile water to the water-seal chamber
4
Make preparations to remove the chest tube
1
Fluid in the water-seal chamber should rise and fall as the client breathes in and out (tidaling) until the lungs have expanded completely; a lack of tidaling on the second postoperative day indicates that the tube is obstructed. Increasing the amount of suction is contraindicated without a prescription because it can traumatize pleural tissue. The level of the fluid, as long as it covers the tube in the water-seal chamber, does not affect tidaling. The lungs will not be fully expanded on the second postoperative day; the chest tube will remain in place.

We have an expert-written solution to this problem!
What safety measure should the nurse instruct parents to follow when their child is a toddler?
1
Place window guards on all windows.
2
Have the toddler sleep on the back or side.
3
Start swimming training for the toddler under supervision.
4
Teach the child how to cross streets and walk in parking lots.
1
Parents should make their home environment safe for the exploring toddler. Parents of toddlers should be advised to install window guards on all windows so that the toddler does not fall out of the window. Infants should be put to sleep on their back or side to lower the lowest risk of sudden infant death syndrome (SIDS). Preschoolers should be taught to swim at an early age under supervision. Preschoolers should be taught to cross streets and walk in parking lots.

A client is admitted to the hospital with a diagnosis of lower extremity arterial disease (LEAD) or peripheral arterial disease. Which is the most beneficial lifestyle modification the nurse should teach this client?
1
Stop smoking
2
Control blood glucose
3
Start a walking program
4
Eat a low-fat, low-cholesterol diet
1
Smoking is the single most important risk factor for peripheral arterial diseases, and cessation should be encouraged. Although hyperglycemia is a contributing factor, it is not the primary risk factor for LEAD. Although a sedentary lifestyle is a contributing factor, it is not the primary risk factor for LEAD. Although a high-fat, high-cholesterol diet is a contributing factor, it is not the primary risk factor for LEAD.

During assessment, the nurse asks a client about developmental milestones such as the age at which thelarche and menarche occurred. The nurse determines that the client experienced pubertal delay. Which finding in the client’s history supports the nurse’s conclusion?
1
Weight increased by 8 to 12 kg.
2
Menarche occurred 2 years after thelarche.
3
Breast development occurred by 15 years of age.
4
Growth in height stopped 2 years after menarche.
3
When the development of breasts has not occurred by 13 years of age in girls, it is considered pubertal delay. An increase in weight between 7 and 25 kg is considered normal during the growth spurt period. The occurrence of menarche within 2 years of onset of breast development, or thelarche, is a normal finding. Generally in girls, growth in height stops 2 to 2.5 years after menarche.

Which data should the nurse anticipate when conducting a developmental assessment for a 5-year-old client? Select all that apply.
1
Names coins correctly
2
Has a vocabulary of 1500 words
3
Participates in parallel play
4
Ties shoe laces independently
5
Has hand dominance established
145
When conducting a developmental assessment for a 5-year-old client, the nurse anticipates the client to be able to name coins correctly, tie shoe laces independently, and have established hand dominance. A vocabulary of 1500 words would indicate a language delay; the 5-year-old client is expected to have a vocabulary of 2100 words. Participation is associative, not parallel play, and is expected by 5 years of age.

The nurse is assessing the rate of involution of a client’s uterus on the second postpartum day. Where does the nurse expect the fundus to be located?
1
At the level of the umbilicus
2
One fingerbreadth above the umbilicus
3
Above and to the right of the umbilicus
4
One or two fingerbreadths below the umbilicus
4
The fundus tends to stay at or slightly above the umbilicus for about 24 hours, then decreases in height by about one fingerbreadth per day. The location of the uterus during the first 24 hours postpartum is at one fingerbreadth above the umbilicus. Location of the fundus above and to the right of the umbilicus indicates that the client’s bladder is distended and the client should void; this is more likely to occur during the first postpartum day.

A 16-year-old boy with a diagnosis of adolescent adjustment disorder and his family are beginning family therapy. What is the best initial nursing approach?
1
Setting long-term goals for the family
2
Letting the client express his feelings first
3
Having the parents explain their rationale for setting firm limits
4
Encouraging each family member to share how the problem is perceived
4
Family therapy must include the whole family. Each member must be considered not just individually from his or her perspective but also as a member of the whole. Identification of the problem by the people involved is the priority. The family, not the nurse, sets goals. The nurse assists the family in setting goals by acting as a facilitator. Feelings should be shared eventually, but this is not the initial focus. Setting limits may or may not be a problem within the family.

A nurse is assessing a newborn with caput succedaneum. How does the nurse explain the cause of this fetal condition to the new mother?
1
Overlap of fetal bones as they pass through the maternal birth canal
2
Swelling of the soft tissue of the scalp as a result of pressure during labor
3
Hemorrhage of ruptured blood vessels that does not cross the suture lines
4
Accumulation of fluid resulting from partial blockage of cerebrospinal fluid drainage
2
Caput succedaneum is a diffuse pattern of edema above the periosteum; it results from an even distribution of pressure on the fetal head during labor. Overlap of fetal scalp bones is called molding. Swelling that does not cross the suture line is cephalhematoma, not caput succedaneum; it occurs when the fetal head is pressing on the rim of the pelvis during the birthing process. Accumulation of fluid resulting from a partial blockage of cerebrospinal fluid is hydrocephalus; in hydrocephalus the circumference of the head is larger than expected.

Which nursing actions are accurate when measuring a head circumference for an infant? Select all that apply.
1
Using a cloth tape for accuracy
2
Obtaining one measurement per visit
3
Documenting the information in the progress notes
4
Using paper tape marked with tenths of a centimeter
5
Placing the tape slightly above the eyebrows and pinna of the ears
45
Nursing actions that are accurate when measuring a head circumference for an infant include using paper tape parked in tenths of a centimeter (to facilitate plotting information on growth chart) and placing the tape slightly above the eyebrows and the pinna of the ears (the largest circumference of the head is the target measurement). The use of cloth tape is not recommended because it may stretch providing an inaccurate measurement; paper or metal tape should be used. The nurse should take several measurements at each visit for accuracy. The head circumference is plotted on the growth chart; the information is not documented in the progress notes.

We have an expert-written solution to this problem!
The nurse accompanies a 3-year-old child to the playroom. The toddler seems afraid to select a toy or activity. What age-appropriate play materials should the nurse offer? Select all that apply.
1
Plastic tea set
2
Mold and clay
3
Play telephone
4
Pencil and paper
5
Simple video game
123
Toddlers are entering the developmental stage of creative and imaginative play. Holding an imaginary tea party is a safe, appropriate activity for a toddler. Using clay to make shapes, both with and without a mold, enhances toddlers’ creativity and improves their fine motor coordination. Creative, imaginative, and imitative play is associated with toddlers. A 3-year-old child is too young to manipulate a pen or pencil and may cause self-injury or an injury to others. A 3-year-old child does not have the cognitive ability or the fine motor coordination to play simple video games.

The nurse is caring for a preschooler diagnosed as suffering from frequent episodes of sleep terrors. Which statements describing the nature of sleep terrors does the nurse know to be true? Select all that apply.
1
It is followed by full waking.
2
It usually occurs 1 to 4 hours after falling asleep.
3
It takes place during rapid-eye movement (REM) sleep.
4
The child rapidly returns to sleep after an episode of sleep terrors.
5
The child is aware of and reassured by another’s presence after an episode of sleep terrors.
24
Sleep terrors occur usually 1 to 4 hours after falling asleep, when non-REM sleep is deepest. After an episode, the child rapidly returns to sleep; it is often difficult to keep the child awake after this. Nightmares are followed by full waking; sleep terrors are followed by a partial arousal. Nightmares take place during REM sleep; sleep terrors take place during state IV, non-REM sleep. After a nightmare, the child is aware of and reassured by another’s presence. After an episode of sleep terrors, however, the child is not very aware of another’s presence, is not comforted, and may push the person away and scream and thrash more if held or restrained.

A registered nurse is teaching a nursing student about Piaget’s theory of cognitive development. What information should the nurse provide about the sensorimotor period? Select all that apply.
1
“In this stage, the child learns that he or she is separate from his or her parents or favorite toy.”
2
“In this stage, the child develops a schema or action pattern for dealing with his or her environment.”
3
“In this stage, the child believes that non-living objects have realistic thoughts, wishes, and feelings.”
4
“In this stage, the child learns about himself or herself and the environment through motor and reflex actions.”
5
“In this stage, the child promotes his or her cognitive development and learns about the world through playing.”
124
According to Piaget’s theory of cognitive development, the sensorimotor period is the time when the child learns that he or she is separate from his or her parents or favorite toy. In addition, the child develops a schema or action pattern for dealing with the environment by hitting, looking, grasping, and kicking. The child also learns about himself or herself and the environment through motor and reflex actions. In the preoperational stage, the child believes that non-living objects have realistic thoughts, wishes, and feelings. The child also promotes his or her cognitive development and learns about the world by playing.

When assessing a client, the nurse auscultates a murmur at the second left intercostal space (ICS) along the sternal border. This reflects sound from which valve?
1
Aortic
2
Mitral
3
Pulmonic
4
Tricuspid
3
The second left intercostal space (ICS) along the sternal border reflects sounds from the pulmonic valve. The correct landmark for auscultating the aortic valve is at the right second ICS at the sternal border; for the mitral valve (apical pulse) at the left fifth ICS in the midclavicular line; and for the tricuspid valve at the left fifth ICS at the sternal border.

An adolescent is found to have type 1 diabetes. The nurse plans to teach the adolescent that dietary control and exercise can help regulate the disorder. What additional information should the nurse include in the teaching plan? Select all that apply.
1
Insulin therapy
2
Prophylactic antibiotics
3
Blood glucose monitoring
4
Oral hypoglycemic agents
5
Adherence to the treatment regimen
135
Because clients with type 1 diabetes have little or no endogenous insulin, they must take insulin. Blood glucose monitoring is an important aspect of therapy because it aids evaluation of the effectiveness of diabetic control. Dietary control and exercise reduce the amount of exogenous insulin needed. Although adhering to the diabetic regimen is difficult, especially for adolescents who need to identify with their peers, its importance in promoting euglycemia should be discussed. Although infection increases insulin requirements, prophylactic antibiotics are not needed. Oral hypoglycemics are ineffective in stimulating insulin secretion in clients with type 1 diabetes.

A client at 40 weeks’ gestation is admitted to the birthing unit in labor. During the initial examination the nurse uses Leopold maneuvers to palpate the abdomen. The purpose of this intervention is to assess the what?
1
Station of the fetus
2
Position of the fetus
3
Duration of the contractions
4
Frequency of the contractions
2
Leopold maneuvers are used specifically to identify the position of the fetus. The nurse palpates the abdomen to locate the head, back, and small parts of the fetus; the locations of these parts reveal the position of the fetus. The station can be ascertained during a vagin*l examination, which is not part of the Leopold maneuvers. Duration of the contractions is timed by lightly placing the examining hand on the fundus during a contraction or observing the electronic monitor. Frequency of contractions is timed by lightly placing the examining hand on the fundus for several minutes or by observing the electronic fetal monitor.

An expectant couple asks the nurse about the cause of low back pain in labor. The nurse replies that this pain occurs most often when the fetus is in what position?
1
Breech
2
Transverse
3
Occiput anterior
4
Occiput posterior
4
A persistent occiput posterior position causes intense back pain because of fetal compression of the maternal sacral nerves. The breech position is not associated with back pain. The transverse position does not usually cause back pain. Occiput anterior is the most common fetal position and does not cause back pain.

A nurse educates parents about how to communicate with their 14-year-old. Which statement should the nurse make?
1
“You should ask your child closed-ended questions.”
2
“You should avoid involving other individuals and resources.”
3
“You should avoid discussing sensitive issues with your child.”
4
“You should look for the meaning behind your child’s words or actions.”
1
The nurse should tell parents to search for the reasons or meanings behind their child’s words or actions. Parents should ask open-ended questions instead of closed-ended questions. Parents may involve other individuals and resources whenever necessary. Parents should not avoid discussing sensitive issues such as sex, drugs, or school problems.

A plan of care is created for a term small-for-gestational-age (SGA) neonate who has been admitted to the neonatal intensive care unit (NICU). The goal is for the newborn to reach 5 lb (2300 g) by a specified date. On the specified date the infant weighs 4 lb 2 oz (1871 g). What should the next step be in care planning for this infant?
1
Increase the daily number of calories
2
Change the goal to a more realistic number
3
Evaluate the problem before altering the plan
4
Postpone the evaluation date for another month
3
Before further intervention is undertaken, the reason for the inadequate weight gain should be evaluated. Evaluation should take place before the plan is changed or the goal altered to identify any barriers to achieving the goal. Postponing the evaluation date for another month is unsafe; the reason for the lack of goal attainment must be identified. The other interventions, increasing the daily number of calories or changing the goal to a more realistic number, are premature.

We have an expert-written solution to this problem!
Which finding indicates that a newborn has vernix caseosa?
1
Brown hair on the skin
2
Rosy to yellowish skin
3
Light-pink to reddish-brown skin
4
Cheese-like substance on the skin
4
Sebum and desquamating cells on the newborn’s skin give it a white, cheesy appearance, which is called vernix caseosa. Brown hair on a newborn’s skin is called lanugo. Newborns of Asian descent will have rosy to yellowish skin. Light-pink to reddish-brown skin indicates that the newborn is of Native-American (Indigenous) descent.

We have an expert-written solution to this problem!
What would the nurse describe as a similarity between the growth and development of toddlers and preschoolers?
1
Both gain 5 to 7 pounds per year.
2
Both need an equal amount of calories.
3
Both grow about 2.5 inches in height per year.
4
Both need at least 12 hours of sleep every night.
1
Toddlers and preschoolers gain 5 to 7 pounds each year. Toddlers need less calories and more protein for adequate neurologic development. On the other hand, preschoolers need at least 1800 calories in a day. Toddlers grow 2.5 inches per year, whereas growth per year ranges from 2.5 to 3 inches in preschoolers. Toddlers need a total of 12 hours of sleep in a day, whereas preschoolers need 12 hours of sleep during the night, and they rarely take daytime naps.

A client’s membranes ruptured 20 hours before admission. The client was in labor for 24 hours before giving birth. For which postpartum complication is this client at risk?
1
Infection
2
Hemorrhage
3
Uterine atony
4
Amniotic fluid embolism
1
When the membranes rupture, microorganisms from the vagin* may travel into the embryonic sac, causing chorioamnionitis. The longer the time between the rupture of the membranes and the birth, the greater the risk for infection. The temperature should be assessed every 1 to 2 hours, and any increase to 100.4° F (38° C) should be reported. If there are no other complications, hemorrhage and uterine atony are not expected. Amniotic fluid embolism is not likely to occur when the membranes rupture before birth, because the fluid exits by way of the vagin* rather than being forced upward.

A nurse performs Leopold maneuvers on a newly admitted client in labor. Palpation reveals a soft, firm mass in the fundus; a firm, smooth mass on the mother’s left side; several knobs and protrusions on the mother’s right side; and a hard, round, movable mass in the pubic area with the brow on the right. On the basis of these findings, the nurse determines that the fetal position is what?
1
LOA
2
ROA
3
LMP
4
RMP
1
The fetus is in a left occiput anterior (LOA) position because the buttock (firm mass) is in the fundus, the back is on the left, the small parts are on the right, and the head is flexed, indicating an anterior occiput. The right occiput anterior (ROA) position is indicated by the presence of the back on the right side and the cephalic prominence on the left side; the occiput is anterior. The left mentum posterior (LMP) position is marked by cephalic prominence. The back is on the same side, indicating an extended head and chin presentation. In the right mentum posterior (RMP) position, the back and cephalic prominence are situated on the same side (right), indicating an extended head and chin presentation.

Which suggestion should the nurse offer to parents who are concerned about caring for their toddler?
1
Refrain from giving more than five cups of milk a day.
2
Allow the toddler to choose a time to take medicine.
3
Let the toddler watch television if the parent is busy.
4
Encourage the toddler to drink from two-handled cups.
4
The rapid development of a toddler’s skills leads to a sense of autonomy. The toddler should be encouraged to drink from two-handled cups with a spout to prevent spills during the learning process. The toddler should not have more than two to three cups of milk per day. Increasing in the consumption of milk reduces the toddler’s appetite for essential solid foods, leading to inadequate iron intake. Parents should limit opportunities for the toddler to say no. The parents should be firm and ask the toddler to take medicine rather than offering choices. Parents should talk, read, or play with the toddler. Television should never be used in place of parent-child interaction.

We have an expert-written solution to this problem!
A couple seeking genetic counseling are heterozygous carriers of Tay-Sachs disease. They ask the nurse what the chances are that each of their children will inherit the disease. The nurse responds that the probability is what?
1
0%
2
25%
3
50%
4
100%
2
Because Tay-Sachs disease is an autosomal-recessive disorder, the probability of one of their offsprings inheriting this disorder is 25%. The risk is 0% if one parent is a Tay-Sachs carrier and the other does not have the gene. Risk is 50% if one parent is heterozygous and the other is hom*ozygous; however, this does not occur because children with Tay-Sachs disease do not live long enough to reproduce. Likewise, a 100% probability does not occur due to the shortened lifespan of children with Tay-Sachs disease.

The nurse is performing physical assessments for children in a daycare center. Which children should require a head circumference in order to monitor growth patterns? Select all that apply.
1
A 6-month-old infant who is breastfed
2
A 15-month-old toddler who has asthma
3
A 3-year-old child whose birthday was the day prior
4
A 5-year-old who will attend kindergarten in the fall
5
An 8-year-old child who will begin playing soccer next week
123
The nurse includes a head circumference in the physical assessment from birth to 36 months of age; therefore, the 6-month-old infant, the 15-month-old toddler, and the 3-year-old child whose birthday was the day prior will all have their head circumferences measured during the assessment. The 5-year-old and the 8-year-old will not have their head circumferences measured during the assessment process.

A newborn experiences a hypothermic period while being bathed and having clothing changed. Once the hypothermic episode has been identified and treated, what is the next nursing action?
1
Feeding the infant
2
Requesting a complete blood count
3
Monitoring the infant for hyperthermia
4
Allowing the infant to rest undisturbed
1
A newborn who experiences a hypothermic episode responds by becoming hypoglycemic; providing calories will increase the blood glucose level. The blood count will not change during a transient hypothermic episode. If the hypothermic period is treated adequately, hyperthermia is not expected to develop. Allowing the infant to rest undisturbed will result in a delay in meeting the newborn’s need for an increase in blood glucose.

We have an expert-written solution to this problem!
The nurse provides nutritional counseling to the parents of a 6-month-old formula-fed infant who will begin eating solid foods. Which statement by a parent indicates understanding of the nurse’s advice?
1
“I’ll keep giving him formula instead of regular cow’s milk.”
2
“I’ll buy plenty of pureed spinach so she gets enough iron.”
3
“Using a natural sweetener like honey is better than using table sugar.”
4
“Baby food is sterilized, so it’s better to feed directly from the jar than from a bowl.”
1
Infants should receive formula or breast milk for a full year; cow’s milk should not be introduced until 1 year of age. Commercially prepared spinach, collard greens, and certain other foods contain nitrates and so should be used very sparingly; if the infant is iron deficient, other sources should be used. Honey should not be given to infants because there is risk of botulism poisoning. Infants should not be fed from the jar because enzymes from saliva on the spoon will affect the quality of the food remaining in the jar.

A nurse is evaluating scenarios that are based on the responses of several clients. Which statement of a client confirms that he or she has reached the Intimacy versus Isolation stage according to Erikson’s theory of psychosocial development?
1
“I donate a large sum of money to the local school every year.”
2
“I want to enjoy my motherhood and that’s why I am leaving the job.”
3
“In the winter of my life, I feel that I do not have anyone to take care of me.”
4
“I did so much for my partner but I was dumped for someone more attractive.”
4
In the Intimacy versus Isolation stage, a young adult develops a sense of identity and deepens his or her capacity to love and care for others. A statement that exemplifies this stage is one from a client who says that he or she did so much for his or her partner but still got dumped for someone more attractive. An example of the Generativity versus Self-Absorption and Stagnation stage is if a client says that he or she donates a large sum of money to the local school every year. An older adult says that he/she feels that he/she does not have anyone to take care of him or her is in the Integrity versus Despair stage.

A nursing instructor asks a nursing student to explain the teaching methods to be used for adolescents. Which statement by the student indicates a need for further teaching? Select all that apply.
1
Keep teaching sessions short
2
Use teaching as a collaborative activity
3
Use problem-solving to help adolescents make choices
4
Encourage them to learn together, using pictures and short stories
5
Help adolescents learn about feelings and the need for self-expression
14
Teaching sessions should be kept shorter for an older adult, not an adolescent. Preschoolers, not adolescents, are encouraged to learn together through the use of pictures and short stories. This helps make learning interesting for them. Hence, the nursing student’s statements that teaching sessions should be kept shorter for the adolescent client and that adolescents should be encouraged to learn together through the use of pictures and short stories indicates a need for further teaching. The teaching should be such that it helps the adolescent understand his or her feelings and the need for self-expression. Teaching should be used as a collaborative activity in adolescents. A problem-solving approach can also be adopted to help adolescents make choices.

Which nursing actions reflect Leininger’s caring theory in practice?
1
The nurse supports and accepts the client’s feelings.
2
The nurse promotes beauty, comfort, dignity, and peace.
3
The nurse learns culturally specific behaviors to meet the client’s needs.
4
The nurse believes that every caring process has subparts that should be considered while making an effective strategy.
3
Leininger’s caring theory states the importance of understanding cultural behaviors in providing effective nursing care. According to Watson’s transpersonal caring theory, the nurse should support and accept the client’s feelings. It also states that the supportive, protective, and corrective mental environment should be provided to create a healing environment. It implies that the nurse should provide comfort, dignity, and peace to the client. Swanson’s theory states that the caring process has definitions and subdimensions that serve as the basis for the nursing process.

We have an expert-written solution to this problem!
What role is the nurse expected to have in a community-based nursing practice if there is a sudden spread of malaria?
1
Educator
2
Collaborator
3
Epidemiologist
4
Client advocate
3
As an epidemiologist, the nurse is responsible for community surveillance for risk factors such as the sudden spread of malaria. An epidemiologist nurse protects the health level of the community, develops sensitivity to changes in the health status of the community, and helps identify the cause of these changes. As an educator in a community-based setting, the nurse provides knowledge to clients and families so they can learn how to care for themselves. As a collaborator in a community-based nursing practice, the nurse collaborates with hospice staff, social workers, and pastoral care to initiate a plan to support end-of-life care for the client and support the family. As a client advocate in a community-based setting, the nurse provides necessary information for clients to make informed decisions in choosing and using services.

We have an expert-written solution to this problem!
A school nurse is teaching high school girls regarding the importance of immunizations. Which newborn anomaly can occur if rubella is contracted during the first trimester of pregnancy?
1
Limb abnormalities
2
Hydrocephalus
3
Down syndrome
4
Cardiac anomalies
4
Heart development occurs between the second and eighth weeks of gestation; any type of maternal infection during this time may result in cardiac anomalies in the newborn. The congenital absence of the proximal portion of a limb is associated with the intake of teratogenic drugs, not with rubella infection. Hydrocephalus is a neural tube defect that is not associated with rubella; however, the infant may have microcephaly. Down syndrome is a chromosomal disorder; it is not caused by a maternal infection.

While caring for a pregnant client with a body mass index of 32 during labor, the nurse observes that the second stage of labor lasts for about 11 minutes. The nurse also finds that the expected birth weight of the fetus is around 4200 g. Which complication does the nurse anticipate in the neonate after birth?
1
Erb palsy
2
Klumpke palsy
3
Strawberry hemangioma
4
Erythema toxicum neonatorum
1
Maternal body mass index of greater than 30, a second stage of labor lasting less than 15 minutes, and an infant birth weight higher than 4000 g indicates a risk of Erb palsy or Erb-duch*enne paralysis in the neonate. Klumpke palsy can result from severe stretching of the upper extremities while the trunk is relatively less mobile during labor. A maternal body mass index greater than 30, a second stage of labor lasting less than 15 minutes, and infant birth weight higher than 4000 g are not the indicators of strawberry hemangioma or erythema toxicum neonatorum.

A client who has missed two menstrual periods tells a nurse at the prenatal clinic that the home pregnancy test was positive. Her last menstrual period began on June 18. According to Nägele’s rule, what is the estimated date of birth (EDB)?
1
March 8
2
March 11
3
March 1
4
March 25
4
March 25 is the EDB. Using Nägele’s rule, take the first day of the last menstrual period (June 18), subtract 3 months, and then add 7 days. March 8, March 11, and March 1 are incorrect calculations according to Nägele’s rule.

What does a community-based nurse do as a change agent? Select all that apply.
1
The nurse empowers clients and their families to creatively solve problems.
2
The nurse works with clients to solve problems and helps clients identify an alternative care facility.
3
The nurse helps clients gain the skills and knowledge needed to provide self-care.
4
The nurse empowers clients to become instrumental in creating change within a health care agency.
5
The nurse does not make decisions but rather helps clients reach decisions that are best for them.
124
As a change agent, the nurse empowers clients and families to creatively solve problems. As a change agent, the nurse works with clients to solve problems and helps them identify an alternative care facility. As a change agent, the nurse empowers clients to become instrumental in creating change within a health care agency. As an educator, the nurse helps clients gain the skills and knowledge needed for self-care. As a counselor, the nurse does not make decisions, but rather helps clients reach decisions that are best for them.

We have an expert-written solution to this problem!
A nurse is providing discharge teaching for a client who recently had surgery for an abdominal perineal resection of the colon and the creation of a colostomy. Which condition will the nurse share with the client for when to call the healthcare provider immediately?
1
Intestinal cramps during fluid inflow
2
Difficulty inserting the irrigation tube
3
Passage of flatus during expulsion of feces
4
An inability to complete the procedure in one hour
2
Difficulty with inserting the irrigation tube indicates stenosis of the stoma and should be reported to the healthcare provider. Intestinal cramps during fluid inflow is a common response. Flatus is always present in the bowel to some degree, and a colostomy irrigation will facilitate its expulsion. Inability to complete the procedure within one hour is not indicative of a medical problem; a colostomy irrigation usually can be completed in one hour, but some individuals may need more time.

A pregnant client is prescribed heparin to prevent the risk of thromboembolism. Which adverse effects should the nurse anticipate with this medication? Select all that apply.
1
Osteoporosis
2
Suppress contractions in labor
3
Increased risk of serious bleeding
4
Stimulation of uterine contraction
5
Compression fractures of the spine
15
Heparin is an anticoagulant. When heparin is taken concurrently during pregnancy, it may cause osteoporosis. This in turn can cause compression fractures of the spine. The use of aspirin in the near term of pregnancy can suppress contractions in labor. The increased risk of serious bleeding also occurs with use of aspirin during pregnancy. The use of prostaglandin during pregnancy can cause stimulation of uterine contraction and can cause abortion.

A client with acquired immunodeficiency syndrome (AIDS) is receiving a treatment protocol that includes a protease inhibitor. When assessing the client’s response to this drug, which common side effect should the nurse expect?
1
Diarrhea
2
Hypoglycemia
3
Paresthesias of the extremities
4
Seeing yellow halos around lights
1
Diarrhea, nausea, and vomiting are common side effects; clients should take these medications with a meal or light snack. These drugs may cause hyperglycemia, not hypoglycemia. Circumoral (perioral), not peripheral, paresthesias may occur with protease inhibitors; peripheral paresthesias may occur with nucleoside reverse transcriptase inhibitors. Seeing yellow halos around lights does not occur with protease inhibitors; it may occur with digoxin toxicity.

A healthcare provider prescribes oral loperamide (Maalox) and intravenous ranitidine for a client with burns and crushing injuries caused by an accident. The client asks how these medications work. What is the nurse’s best response?
1
“They decrease irritability of the bowel.”
2
“They limit acidity in the gastrointestinal tract.”
3
“They are very effective in clients with multiple trauma.”
4
“They work in the way that antiemetics do.”
2
Increased acidity caused by the stress occurring with burns and crushing injuries contributes to the formation of Curling ulcer; ranitidine, an H2 antagonist, decreases the formation of gastric acid, and Maalox, an antacid, neutralizes gastric acid once it is formed. These drugs do not decrease irritability of the bowel; their purpose is to decrease gastrointestinal acidity. The response “They are very effective in clients with multiple trauma” does not answer the client’s question. The mechanisms of action for these drugs are not the same as those in antiemetics.

The nurse is preparing a teaching plan for clients receiving antitubercular medications. Which teaching plan needs correction?
1
Clients taking ethambutol should drink plenty of fluids.
2
Clients taking Isoniazid should take the drug with food.
3
Clients taking pyrazinamide should wear a hat while going out in the sun.
4
Clients taking rifampin should use other contraceptive methods even after stopping the medication.
2
Taking isoniazid with food should be corrected. The presence of food may slow or even prevent the absorption of isoniazid from the stomach. Therefore the client should take the medication on an empty stomach, either 1 hour before or 2 hours after eating. Ethambutol may increase uric acid formation. The client should drink plenty of water to reduce uric acid precipitation and kidney problems. Pyrazinamide is a photosensitive medication that may increase the risk of sunburn. Therefore the client should wear a hat and protect himself or herself from sun exposure. Rifampin may decrease the efficiency of oral contraceptives. Therefore, the nurse should instruct the client to use an additional method of contraception even after stopping the medication.

Which over-the-counter (OTC) drug is used as the first line of therapy for acne vulgaris in adolescents?
1
Tretinoin
2
Isotretinoin
3
Azithromycin
4
Benzoyl peroxide
4
Benzoyl peroxide is an effective and safe first-line agent and can be obtained over the counter. Tretinoin may also be recommended, but it is not as widely available in OTC products and is more potent. Isotretinoin is a strong drug that is not available over the counter and has potentially disruptive side effects. Azithromycin is not a drug of choice for treating acne.

We have an expert-written solution to this problem!
A client is to receive conscious sedation during a cardiac catheterization. Which route of administration should the nurse explain will be used to deliver the conscious sedation?
1
Via a face mask
2
Into the epidural space
3
Through an intravenous catheter
4
Around the nerves innervating the chest wall
3
Conscious sedation is administered by direct intravenous (IV) injection or IV push to dull or reduce the intensity of pain or awareness of pain during a procedure without loss of defensive reflexes. General anesthesia usually is administered via inhalation of the vapor of a volatile liquid or an anesthetic gas via a mask or endotracheal tube; as a result, the client is unconscious, unaware, and anesthetized. An epidural block, a type of regional anesthesia, involves the injection of a local anesthetic into the epidural (extradural) space; it works by binding to nerve roots as they enter and exit the spinal cord. A nerve block, a type of regional anesthesia, is achieved by injection of the anesthetic agent into or around the nerves supplying the area; it interrupts sensory, motor, and sympathetic transmission.

We have an expert-written solution to this problem!
A client in a detoxification unit has an alcohol withdrawal seizure. Diazepam 7.5 mg intramuscularly stat is prescribed. Diazepam is available as 5 mg/mL. How many milliliters will the nurse administer? Record your answer using one decimal place. _ mL
1.5

A child is diagnosed with classic hemophilia. A nurse teaches the child’s parents how to administer the plasma component factor VIII through a venous port. It is to be given three times a week. When should the parents administer this therapy?
1
Whenever a bleed is suspected
2
In the morning on scheduled days
3
At bedtime while the child is lying quietly in bed
4
On a regular schedule at the parents’ convenience
2
Factor VIII has a short half-life; therefore prophylactic treatment involves administering the factor on the scheduled days in the morning so the child will get the most benefit during the day, while he is most active. Prophylactic treatment is administered on a scheduled basis to prevent bleeds from occurring. Administering the drug at bedtime will limit its effectiveness because bleeds are more common when the child is active. Administering the medicine on a regular schedule at the parents’ convenience does not take into consideration the properties of the drug.

We have an expert-written solution to this problem!
A pregnant client with iron-deficiency anemia is prescribed iron supplements daily. To help the client increase iron absorption, the nurse should suggest that the client eat foods high in which substance?
1
Vitamin C
2
Fat content
3
Water content
4
Vitamin B complex
1
Vitamin C aids the absorption of iron. Fat content, water content, and vitamin B complex are all unrelated to the absorption of iron.

The laboratory international normalized ratio (INR) results of a client receiving warfarin have been variable. The nurse interviews the client to determine factors contributing to the problem. Which is most important for the nurse to identify?
1
Use of analgesics
2
Serum glucose level
3
Serum potassium levels
4
Adherence to the prescribed drug regimen
4
The dosage of warfarin is adjusted according to INR results; if the client fails to take the drug as prescribed, test results will not be reliable in monitoring the client’s response to therapy. Although some medications can affect the absorption or metabolism of warfarin and should be investigated, this is less likely to be a cause of fluctuations in laboratory values. Serum glucose level and serum potassium levels do not affect the absorption of warfarin.

A school-aged child who has just arrived from Africa has been exposed to diphtheria, and a nurse in the pediatric clinic is to administer the antitoxin. Which type of immunity does the antitoxin confer?
1
Active natural
2
Passive natural
3
Active artificial
4
Passive artificial
4
In the creation of passive artificial immunity an antibody is produced in another organism and then injected into the infected or presumed infected person to provide immediate immunity against the invading organism. Active natural immunity takes too much time to develop; this child needs immediate protection. Passive natural immunity is acquired from the mother and is effective only during the first few months of life. Active artificial immunity takes too much time to develop; the child needs immediate protection.

A client is admitted with head trauma after a fall. The client is being prepared for a supratentorial craniotomy with burr holes, and an intravenous infusion of mannitol is instituted. The nurse concludes that this medication primarily is given to do what?
1
Lower blood pressure
2
Prevent hypoglycemia
3
Increase cardiac output
4
Decrease fluid in the brain
4
Osmotic diuretics remove excessive cerebrospinal fluid (CSF), reducing intracranial pressure. Osmotic diuretics increase, not decrease, the blood pressure by increasing the fluid in the intravascular compartment. Osmotic diuretics do not directly influence blood glucose levels. Although there is an increase in cardiac output when the vascular bed expands as CSF is removed, it is not the primary purpose for administering the medication.

A monoamine oxidase inhibitor (MAOI) is prescribed, and the nurse is formulating a teaching plan. What will the nurse instruct the client to avoid while taking this drug?
1
Fermented foods
2
Prolonged sun exposure
3
Strenuous physical exercise
4
Over-the-counter antihistamine drugs
1
An MAOI can cause hypertensive crisis if food or beverages that are high in tyramine, such as fermented foods, are ingested. Prolonged exposure to the sun is hazardous for clients taking one of the phenothiazines. Strenuous physical exercise is not contraindicated. Antihistamines are not prohibited with MAOI medications.

Which drug is responsible for neonatal hypoglycemia?
1
Warfarin
2
Simvastatin
3
Tolbutamide
4
Methimazole
3
Tolbutamide is an oral hypoglycemic agent used in the treatment of type 2 diabetes mellitus. It is known to have teratogenic effects like neonatal hypoglycemia. Warfarin, a common blood thinner, may cause teratogenic effects such as skeletal and central nervous system defects. Simvastatin, an HMG-CoA reductase inhibitor used for the treatment of high cholesterol, may cause teratogenic effects like facial malformations and severe central nervous system anomalies. Methimazole, an antithyroid drug administered for the treatment of maternal hyperthyroidism, may cause teratogenic effects like neonatal goiter, cretinism, and hypothyroidism.

We have an expert-written solution to this problem!
A nurse is preparing to administer insulin to a client with diabetes. In which order should the nurse perform the actions associated with insulin administration?
1.
Wipe the top of the insulin vial with an alcohol swab.

2.
Wash hands with soap and water.

3.
Rotate the vial of insulin between the palms of the hands.

4.
Withdraw the correct amount of insulin from the inverted vial.

5.
Instill air into the vial of insulin equal to the desired dose.
Washing the hands prevents cross-contamination. Rotating the insulin vial distributes the drug evenly throughout the vial. Wiping the seal on the insulin vial prevents contamination of the needle and the fluid. Instilling air into the vial increases the pressure in the closed space so that the correct amount of fluid finally can be withdrawn.

A client who has been experiencing double vision, drooping of the eyelids, and fatigue visits the neurologic clinic. A diagnosis of myasthenia gravis is made, and the healthcare provider prescribes pyridostigmine. The nurse should teach the client that it is important to take this drug based on what schedule?
1
On an empty stomach
2
One hour before meals
3
According to muscle strength
4
At the exact time intervals prescribed
4
Taking the medication as prescribed promotes an even therapeutic blood level, which maintains muscle strength. Because of drug-related nausea and gastric irritation, the drug should be taken with crackers or milk. Thirty, not 60, minutes before meals is recommended for maximum chewing and swallowing function. Taking the drug according to muscle strength is unsafe because it will not maintain constant therapeutic drug levels.

Six hours after initiation of total parenteral nutrition, the client’s serum glucose level increases to 240 mg/dL (13.3 mmol/L). What does the nurse conclude is the most likely cause of the increase?
1
The solution is too concentrated.
2
The infusion is flowing too rapidly.
3
The solution is exacerbating preexisting diabetes.
4
The infusion is too slow to meet total nutritional needs.
2
Rapid infusion of concentrated glucose into the vascular system does not allow time for adequate insulin release to transport glucose to the cells. A hyperconcentrated solution usually results in hypervolemia rather than hyperglycemia. There is no evidence that the client has diabetes mellitus. If the infusion was too slow, the blood glucose level would not increase to 240 mg/dL (13.3 mmol/L).

We have an expert-written solution to this problem!
Methylphenidate is prescribed for a 6-year-old boy with the diagnosis of attention deficit-hyperactivity disorder (ADHD). The nurse teaches the father about safe medication administration and concludes that the instructions have been understood when the father says that he should administer it at which time?
1
At bedtime
2
After breakfast
3
When the child gets hungry
4
When the child’s behavior is out of control
2
Methylphenidate (Ritalin SR) may cause nausea, anorexia, and dry mouth, which interfere with appetite and adequate food intake; therefore it should be administered after the child has eaten breakfast. Methylphenidate is a cerebral stimulant that can interfere with sleep; it should not be administered within 6 hours of bedtime. It should be taken exactly as prescribed, not on an as-needed basis.

Which of the following drugs is commonly used as an adjunct during conscious sedation for minor surgeries?
1
Diazepam
2
Midazolam
3
Lorazepam
4
Clonazepam
2
When used in conjunction with an opioid analgesic, midazolam causes conscious sedation, which is a semiconscious state suitable for minor surgeries and endoscopic procedures. Diazepam and lorazepam are used for anesthetic induction, preanesthetic medication, and the production of conscious sedation when used in conjunction with opioid analgesics. Clonazepam is used to treat seizures and anxiety.

We have an expert-written solution to this problem!
Which drug acts as an abortifacient in female clients?
1
Mifepristone
2
Metyrapone
3
Cyproheptadine
4
Aminoglutethimide
1
Mifepristone is an antiprogesterone that blocks the progesterone receptors and acts as an abortifacient. Metyrapone, cyproheptadine, and aminoglutethimide are used to treat hyperfunctioning of the adrenal glands (Cushing’s disease/syndrome).

We have an expert-written solution to this problem!
An infant is receiving 22 mL/hr intravenous fluid by way of an infusion pump. How much fluid will be infused between 8:00 AM and 8:00 PM? Record your answer as a whole number. _ mL
264

A nurse understands that after the administration of alprazolam it is important to assess the client for side effects. What will the nurse do initially?
1
Measure urine output.
2
Check the blood pressure.
3
Look for abdominal distention.
4
Check the size of the pupils frequently
2
Hypotension is a major side effect of alprazolam that occurs early in therapy. An alteration in urine output is not a common side effect; however, urine retention may occur after prolonged use. Abdominal distention is not a common side effect, but abdominal distention from constipation may occur after prolonged use. Blurred vision, not dilated pupils, may occur. Dilated pupils associated with central nervous system depression are not a common side effect, although they may occur with overdose or prolonged use.

A client’s cardiac monitor indicates multiple multifocal premature ventricular complexes (PVCs). The nurse expects that the treatment plan will include a prescription for which medication?
1
Amiodarone
2
Epinephrine
3
Methyldopa
4
Hydrochlorothiazide
1
Amiodarone has an antiarrhythmic action that stabilizes cell membranes of the heart, reducing cardiac excitability; it is used for acute ventricular dysrhythmias. Methyldopa is not used to treat multiple PVCs. Epinephrine increases the contractibility of the heart; the effect is opposite to what is needed. Hydrochlorothiazide is used for hypertension, not for correcting multiple PVCs.

The nurse is caring for a client who is scheduled for an electrophysiology study (EPS) because of persistent ventricular tachycardia. Before the procedure the client is to receive a beta-blocker. What client’s response during the procedure best indicates that the beta-blocker is working effectively?
1
Decreased anxiety
2
Reduced chest pain
3
Decreased heart rate
4
Increased blood pressure
3
A decreased heart rate or sinus bradycardia is the expected response to a beta-blocker. Beta-blockers inhibit the activity of the sympathetic nervous system and of adrenergic hormones, decreasing the heart rate, conduction velocity, and workload of the heart. A beta-blocker is not an anxiolytic and does not reduce anxiety. A beta-blocker is not an analgesic and does not reduce chest pain. Beta-blockers reduce blood pressure.

A client is receiving hydrochlorothiazide. What should the nurse monitor to best determine the effectiveness of the client’s hydrochlorothiazide therapy?
1
Blood pressure
2
Decreasing edema
3
Serum sodium level
4
Urine specific gravity
1
Diuretics promote urinary excretion, which reduces the volume of fluid in the intravascular compartment, thus lowering blood pressure. Edema reflects multiple physiologic processes including venous competence, gravity, and disuse. The serum sodium level remains stable unless the dosage is excessive; an altered sodium level is not a therapeutic response. Although specific gravity decreases with increased urinary output, this does not reflect the desired reduction in intravascular pressure.

The nurse is caring for a client with deep partial-thickness burns who is receiving an opioid for pain management. What is the preferred mode of medication administration for this client?
1
Oral
2
Rectal
3
Intravenous
4
Intramuscular
3
The intravenous route provides for the quickest onset of action of the opioid; pain relief occurs almost immediately. Nausea, vomiting, and paralytic ileus may occur postburn, making oral medications impractical. The rectal route does not provide uniform absorption; also, relief of pain will be delayed. With the intramuscular route, medication may be sequestered in the tissues, and with fluid shifts it takes time for the medication to take effect.

We have an expert-written solution to this problem!
The nurse is administering medication through an implanted port. What nursing safety priority should the nurse follow in this scenario?
1
The nurse should use barrel syringes to flush any central line.
2
The nurse should use 20 mL of sterile saline to flush the port after drawing blood.
3
The nurse should use 10 mL of sterile saline to flush the port before and after medication administration.
4
The nurse should withhold the drug until patency and adequate noncoring needle placement of the port are established.
4
When administering medication through implanted ports, the nurse should withhold the drug until patency and adequate noncoring needle placement of the port are established. In case of a peripherally inserted central catheter (PICC), the nurse should use barrel syringes to flush any central line. The nurse should use 20 mL of sterile saline to flush the port after drawing blood as a nursing safety priority in case of PICC. The nurse should use 10 mL of sterile saline to flush the PICC before after and medication administration.

A client is brought by ambulance to the emergency department. The client’s signs and symptoms are indicative of opioid overdose. What does the nurse expect the primary healthcare provider to prescribe?
1
Naloxone
2
Methadone
3
Epinephrine
4
Amphetamine
1
This drug is an opioid antagonist that displaces opioids from receptors in the brain, reversing respiratory depression. Methadone is a synthetic opioid that causes central nervous system depression; it will accelerate the effects of the overdose. Epinephrine will have no effect on respiratory depression stemming from of an overdose of a narcotic. Amphetamine is a stimulant, not an opioid antagonist.

Several hours after administering insulin, the nurse is assessing a client for an adverse response to the insulin. Which client responses are indicative of a hypoglycemic reaction? Select all that apply.
1
Tremors
2
Anorexia
3
Confusion
4
Glycosuria
5
Diaphoresis
135
Confusion is typically the first sign of a hypoglycemic reaction. Tremors are a sympathetic nervous system response that occurs because circulating glucose in the brain decreases. Diaphoresis is a cholinergic response to hypoglycemia. Hypoglycemia causes hunger, not anorexia. Because blood glucose is low in hypoglycemia, the renal threshold is not exceeded and glycosuria does not occur.

A client diagnosed with asthma has received a prescription for an inhaler. The nurse teaches the client how to determine when the inhaler is empty, instructing the client to do what?
1
Count the number of doses taken.
2
Taste the medication when sprayed into the air.
3
Shake the canister.
4
Place the canister in water to see if it floats.
1
The only way to determine if the canister is empty is to count the number of doses taken. The client is tracking the number of daily doses. It is wasteful to spray medication into the air; tasting it from the air is not an effective method of determining if the canister is empty. Shaking the canister is not effective; even if there is no more medication, some propellant may be left. It is futile to place the canister in water; the flotation test is ineffective.

A nurse plans to teach a client with type 1 diabetes about the use of an insulin pump. What information will the nurse include in client teaching?
1
Insulin pumps mimic the way a healthy pancreas works.
2
The insulin pump’s needle should be changed every day.
3
Pumps are implanted in a subcutaneous pocket near the abdomen.
4
The insulin pump’s advantage is that it only requires glucose monitoring once a day.
1
The basal infusion rate mimics the low rate of insulin secretion during fasting, and the bolus before meals mimics the high output after meals. The subcutaneous needle may be left in place for as long as 3 days. Most insulin pumps are external to the body and access the body via a subcutaneous needle. Blood glucose monitoring is done a minimum of 4 times a day.

A nurse is providing discharge medication teaching to a client who will be taking furosemide and digoxin after discharge from the hospital. What information is most important for the nurse to include in the teaching plan?
1
Maintenance of a low-potassium diet
2
Avoidance of foods high in cholesterol
3
Signs and symptoms of digoxin toxicity
4
Importance of an adequate intake and output
3
The risk of digoxin toxicity increases when the client is receiving digoxin and furosemide, a loop diuretic; loop diuretics can cause hypokalemia, which potentiates the effects of digoxin, leading to toxicity. Digoxin toxicity can result in dysrhythmias and death. When a client is receiving a loop diuretic, the diet should be high in potassium. Although teaching the need to avoid foods high in cholesterol may be included in the teaching plan, it is not the priority. Although it is important to maintain adequate intake and output because potassium chloride should not be taken when there is a decreased urinary output, the priority is monitoring for signs of digoxin toxicity.

Which drug will most likely cause the client’s eyelids to itch and eyes to burn as side effects?
1
Ketorolac
2
Ofloxacin
3
Diclofenac
4
Vidarabine
4
Vidarabine is an antiviral agent that causes sensitive reactions such as itching eyelids and burning eyes. Ketorolac, ofloxacin, and diclofenac usually do not cause sensitive reactions such as itching eyes.

We have an expert-written solution to this problem!
Which gestational period is appropriate for the administration of corticosteroids during preterm labor?
1
Less than 20 weeks
2
20 to 24 weeks
3
24 to 34 weeks
4
More than 34 weeks
3
If preterm labor occurs during 24 to 34 weeks of gestation and if labor is unavoidable, corticosteroids should be administered to promote lung maturity. Labors occurring before 20 weeks of gestation usually results in a nonviable fetus; corticosteroids would not need to be administered. Corticosteroids do not promote lung maturation during 20 to 24 weeks of gestation. Fetal lungs mature at 34 weeks of gestation. A fetus in this period of gestation would not require the administration of corticosteroids if labor is unavoidable.

The nurse has administered lymphocyte immunoglobulin to a client. Which side effects are most likely to occur? Select all that apply.
1
Leukopenia
2
Peptic ulcer
3
Tachycardia
4
Serum sickness
5
Urinary infection
134
Lymphocyte immunoglobulin is an immunosuppressant directed against T-lymphocytes. Tachycardia, leukopenia, and serum sickness are side effects associated with lymphocyte immunoglobulin therapy. Peptic ulcers may occur when corticosteroids are injected. A urinary infection may occur when an immunosuppressant such as belatacept is administered.

A client who weighs 176 pounds (80 kg) is being immunosuppressed by daily maintenance doses of cyclosporine to prevent organ transplant rejection. The dose prescribed is 8 mg/kg each day. How many milligrams should the nurse plan to administer each day? Record your answer using a whole number. _ mg
640

Which points should be included when the mother of an epileptic child is being counseled? Select all that apply.
1
The child should wear a medical alert bracelet.
2
The child should be given valproic acid with milk to reduce gastric irritation.
3
The parent should keep a journal of signs and symptoms before, during, and after seizures.
4
The child should discontinue the drug immediately and the parent should notify the primary healthcare provider if a rash develops.
5
The parent should understand that chewable forms of antiepileptic drugs are recommended for once-a-day administration.
134
The nurse should educate the mother of an epileptic child as a means of improving the child’s quality of life. The child is encouraged to wear a medical alert bracelet for identification in emergencies. Keeping a record of signs and symptoms helps parents provide exact information about the onset and nature of symptoms to the healthcare provider. Telling the parents to discontinue the medication and consult the primary healthcare provider if a rash develops will help to ensure that medication allergies are detected as early as possible. Valproic acid should not be given with milk because the milk causes the drug to dissolve early and irritate the mucosa. Chewable forms of antiepileptic drugs are not recommended for once-a-day administration.

Which physiologic characteristics of newborns affect drug dosage considerations? Select all that apply.
1
A newborn’s less regulated body temperature
2
Immature liver and kidneys
3
Thick and less permeable skin
4
Lungs with weaker mucous barriers
5
Bacteria-killing acid in the stomach
124
The body temperature of newborns is less regulated and dehydration occurs easily. This characteristic affects the drug dose consideration in newborns. Metabolism and excretion are impaired in pediatric clients due to an immature liver and kidneys. The lungs in pediatric clients have weak mucous barriers; this characteristic also affects the drug dosage considerations in newborns. A newborn’s skin is thin and more permeable. The newborn has no acid in the stomach to kill the bacteria; therefore, drug absorption from the gastrointestinal tract is affected, thus impacting drug dosage considerations.

We have an expert-written solution to this problem!
An individual is bitten by a snake that belongs to the class of North American pit vipers. Which medication should be used as a nursing priority to treat the client?
1
Diazepam
2
Epinephrine
3
Acetaminophen
4
Crotalidae Polyvalent Immune Fab (CroFab)
4
CroFab is used for treating a victim bitten by a North American pit viper as a part of hospital care. Diazepam is used for the treatment of black widow spider bite. Diazepam is used as a muscle relaxant. Epinephrine is used as a nursing priority for bees and wasps sting. Acetaminophen is used for the treatment of fever related to scorpion bite.

A client is diagnosed with acute kidney failure secondary to dehydration. An intravenous (IV) infusion of 50% glucose with regular insulin is prescribed. What does the nurse recognize as the primary purpose of the IV insulin for this client?
1
Correct hyperkalemia
2
Increase urinary output
3
Prevent respiratory acidosis
4
Increase serum calcium levels
1
The 50% glucose and regular insulin infusion treats the hyperkalemia associated with kidney failure; it moves potassium from the intravascular compartment into the intracellular compartment. Insulin will not increase urinary output. Insulin is not a treatment for respiratory acidosis. Insulin and glucose do not increase serum calcium levels.

A client is receiving an antipsychotic medication. When assessing the client for signs and symptoms of pseudoparkinsonism, the nurse will be alert for which complication?
1
Drooling
2
Blurred vision
3
Muscle tremors
4
Photosensitivity
3
Drug-induced parkinsonism presents with the classic triad of adaptations associated with Parkinson disease: rigidity, slowed movement (bradykinesia), and tremors. The anticholinergic effects of antipsychotic medication cause dry mouth, not drooling. Neither dry mouth nor drooling is related to pseudoparkinsonism. Blurred vision and photosensitivity are side effects of anticholinergic, not antipsychotic, medications.

A client is prescribed uterine stimulants to augment labor. Which condition should be assessed in the client before initiating therapy?
1
Liver disease
2
Pituitary tumors
3
Pelvic inflammatory disease
4
Presence of an intrauterine device
3
Labor-inducing drugs are contraindicated in women who have pelvic inflammatory disease. Ovarian stimulants are contraindicated in women who have liver disease and pituitary tumors. Abortifacient drugs are contraindicated in women who have intrauterine devices.

A pregnant woman reports severe headaches, chest pain, and fatigue. Upon diagnosis, the woman has hypertension. Which drug can be prescribed to reduce hypertension?
1
Lithium
2
Miglitol
3
Calcium gluconate
4
Magnesium sulfate
4
Magnesium sulfate can be prescribed for pregnancy-induced hypertension. Lithium is used to treat body water retention. Miglitol is used to decrease blood sugar levels. Calcium gluconate is used to relieve magnesium toxicity associated with magnesium sulfate.

We have an expert-written solution to this problem!
At 12 weeks’ gestation a client who is Rh negative expels the total products of conception. What is the nursing action after it has been determined that she has not been previously sensitized to Rh-positive red blood cells?
1
Administering Rho(D) immune globulin within 72 hours
2
Making certain that Rho(D) immune globulin is administered at the first clinic visit
3
Withholding the Rho(D) immune globulin because the gestation lasted only 12 weeks
4
Withholding the Rho(D) immune globulin because it is not indicated after fetal death
1
Rho(D) immune globulin must be given within 72 hours of delivery if the client has not been sensitized previously, regardless of the duration of the gestation. It would not be effective at the first clinic visit, because antibodies have been produced already. Rho(D) immune globulin is always indicated at the termination of a pregnancy, even with a short-term pregnancy or one involving fetal demise.

A healthcare provider prescribes one unit of whole blood for a client after gastrointestinal surgery. What is an important nursing action when administering blood?
1
Warm the blood to body temperature to prevent chilling the client.
2
Obtain baseline vital signs prior to beginning blood administration.
3
Draw a blood sample from the client before each unit is transfused.
4
Maintain patency of the intravenous catheter with dextrose solution.
2
Baseline vital sign assessment is needed in order to have a basis for comparison should the client develop complications during administration. Dextrose solution will cause lysis of the red blood cells (RBCs); saline must be used. Warming the blood to body temperature may cause clotting and hemolysis. Blood samples may be drawn after, not before, a transfusion, but this is not routinely done.

A client on a psychiatric unit who has been hearing voices is receiving a neuroleptic medication for the first time. The client takes the cup of water and the pill and stares at them. What is the most therapeutic statement the nurse can make?
1
“You have to take your medicine.”
2
“This is the medication that your healthcare provider ordered.”
3
“This will help you not to hear the voices. It will only work if you take it.”
4
“There must be a reason that you don’t want to take your medicine.”
3
Telling the client that the provider wants him or her to take the medication and instructing him or her to do so presents reality and simply states the expected behavior. Explaining what the medication will do for the client is an appropriate nursing intervention. Telling the client that he or she has to take the medication is an authoritarian, not a therapeutic, statement. The client has the right to refuse medication. Telling the client that the medication has been prescribed for him or her does not tell the client what behavior is expected. Noting that there must be a reason that the client doesn’t want to take the medication assumes that the client does not want to take medication, even though the client may simply not understand what to do.

A pregnant woman is diagnosed with intrauterine fetal death. What would be the drug of choice for labor induction?
1
Oxytocin
2
Clomiphene
3
Dinoprostone
4
Methylergonovine
3
Dinoprostone is a uterine stimulant indicated for the uterine evacuation of fetal remains. Oxytocin is used to aid in uterine contractions during childbirth. Clomiphene is indicated for female infertility in some clients. Methylergonovine is an oxytocic ergot alkaloid used to reduce postpartum uterine hemorrhage.

A client with multiple myeloma who is receiving the alkylating agent melphalan returns to the oncology clinic for a follow-up visit. For which side effect should the nurse monitor the client?
1
Hirsutism
2
Leukopenia
3
Constipation
4
Photosensitivity
2
Melphalan depresses the bone marrow, causing a reduction in white blood cells (leukopenia), red blood cells (anemia), and thrombocytes (thrombocytopenia); leukopenia increases the risk of infection. Hirsutism occurs with the administration of androgens to women. Diarrhea, not constipation, occurs with melphalan. Photosensitivity occurs with 5-fluorouracil, floxuridine, and methotrexate, not with melphalan.

A postpartum woman treated with methylergonovine complained of dizziness, has low blood pressure, and has passed out. Which initial nursing action would help this client?
1
Document the findings
2
Encourage excess increased fluid intake
3
Advise to stop taking methylergonovine
4
Notify the primary healthcare provider
4
Methylergonovine may cause a sudden drop in blood pressure. Notifying the primary healthcare provider should be the priority action of the nurse in this condition in order to get proper treatment at the right time. Documentation of findings is also important, but it is not the initial step of the nurse. Excess fluid intake is necessary in this client, but it is not an initial step of the nurse. A nurse cannot advise stopping medication prescribed by primary healthcare provider unless instructed.

We have an expert-written solution to this problem!
Amitriptyline is an antidepressant medication used to treat anxiety disorders. The nurse recognizes that is it a member of which drug class?
1
Tricyclics
2
Monoamine oxidase inhibitors (MAOIs)
3
Selective serotonin reuptake inhibitors (SSRIs)
4
Serotonin-norepinephrine reuptake inhibitors (SNRIs)
1
Amitriptyline is one of several tricyclic antidepressants used to treat anxiety disorders. It is not an MAOI, SSRI, or SNRI.

An adult reports anxiety, palpitations, and a feeling of impending doom. After a thorough physical examination, the healthcare provider diagnoses a panic attack. Oral lorazepam 1.5 mg stat is prescribed. The lorazepam is available in 0.5-mg tablets. How many tablets will the nurse administer? Express your answer as a whole number. _ tablet(s)
3

A client’s serum lithium level is 0.2 mEq/L (0.2 mmol/L). What is the nurse’s interpretation of this finding?
1
Within toxic range
2
Borderline toxic range
3
Below therapeutic range
4
Borderline therapeutic range
3
The therapeutic range of serum lithium levels is 0.5 to 1.5 mEq/L (0.5 to 1.5 mmol/L). A serum lithium level of 0.2 mEq/L (0.2 mmol/L) is below the therapeutic range. The signs and symptoms of lithium toxicity usually occur when the serum lithium level exceeds 2.0 mEq/L (2.0 mmol/L). A serum lithium level of 0.2 mEq/L (0.2 mmol/L) does not approach the toxic range for lithium. A serum lithium level of 0.2 mEq/L (0.2 mmol/L) is not within the therapeutic range of 0.5 to 1.5 mEq/L (0.5 to 1.5 mmol/L).

A client with type 1 diabetes requests information about the differences between penlike insulin delivery devices and syringes. What information does the nurse provide about the penlike devices?
1
The penlike devices have a shorter injection time.
2
Penlike devices provide a more accurate dose delivery.
3
The penlike delivery system uses a smaller-gauge needle.
4
Penlike devices cost less by having reusable insulin cartridges.
2
Penlike insulin delivery devices are more accurate because they are easy to use; also, they promote adherence to insulin regimens because the medication can be administered discreetly. One disadvantage of the penlike insulin delivery device is that the injection time will be longer; the device must remain in place for several seconds after the insulin is injected to ensure that no insulin leaks out. The penlike insulin delivery device has a larger-gauge needle that has a smaller diameter. The insulin cartridges of a penlike insulin delivery device are single use and disposable.

The healthcare provider’s prescription for intravenous fluid states that the client is to receive 1 L of fluid every 8 hours. If the equipment delivers 15 drops/mL, at what rate should the nurse regulate the flow? Record your answer using a whole number. _ drops/minute
31

A client with diabetic ketoacidosis who is receiving intravenous fluids and insulin complains of tingling and numbness of the fingers and toes and shortness of breath. The cardiac monitor shows the appearance of a U wave. What complication does the nurse suspect?
1
Hypokalemia
2
Hypoglycemia
3
Hypernatremia
4
Hypercalcemia
1
These are classic signs of hypokalemia that occur when potassium levels are reduced as potassium reenters cells with glucose. Symptoms of hypoglycemia are weakness, nervousness, tachycardia, diaphoresis, irritability, and pallor. Symptoms of hypernatremia are thirst, orthostatic hypotension, dry mouth and mucous membranes, concentrated urine, tachycardia, irregular heartbeat, irritability, fatigue, lethargy, labored breathing, and muscle twitching or seizures. Symptoms of hypercalcemia are lethargy, nausea, vomiting, paresthesias, and personality changes.

What is the maximum length of time a nurse should allow an intravenous bag of solution to infuse?
1
6 hours
2
12 hours
3
18 hours
4
24 hours
4
After 24 hours there is increased risk for contamination of the solution and the bag should be changed. It is unnecessary to change the bag any less often, such as 6 hours, 12 hours, or 18 hours.

Propylthiouracil is prescribed for a client diagnosed with hyperthyroidism. The client asks the nurse, “Why do I have to take this medication if I am going to get radiation therapy?” What explanation does the nurse provide?
1
It binds previously formed thyroid hormones.
2
It decreases production of thyroid hormones.
3
Vascularity of the thyroid gland is decreased.
4
The need for thyroid iodine supplements is reduced.
2
Propylthiouracil is a thyroid hormone antagonist that inhibits thyroid hormone synthesis by decreasing the use of iodine in the manufacture of these hormones. PTU does not affect the vascularity of the thyroid gland. Iodine-containing agents are given for severe hyperthyroidism and before a thyroidectomy. PTU does not affect the amount of already formed thyroid hormones.

A client with severe preeclampsia is receiving magnesium sulfate therapy. What is the priority nursing assessment as the nurse monitors this client’s response to therapy?
1
Urine output
2
Respiratory rate
3
Deep tendon reflexes
4
Level of consciousness
2
Respiratory depression occurs with toxic levels of magnesium sulfate; calcium gluconate should be readily available to counteract toxicity. Although the other assessments (urine output, deep tendon reflexes, and level of consciousness) are important, none is the priority.

What are the pharmaco*kinetic reasons for drug sensitivity in infants? Select all that apply.
1
Small body size
2
Drug absorption
3
Renal drug excretion
4
Protein binding of drugs
5
Hepatic drug metabolism
235
Increased drug sensitivity in infants is a result of the immature state of pharmaco*kinetic processes such as drug absorption, renal drug excretion, protein binding of drugs, and hepatic drug metabolism. A small body is not a pharmaco*kinetic parameter.

While teaching a prenatal class about infant feeding, the nurse is asked about the relationship between breast size and ease of breastfeeding. How should the nurse respond?
1
“Breast size is not related to milk production.”
2
“Motivated women tend to breastfeed successfully.”
3
“You seem to have some concerns about breastfeeding.”
4
“Glandular tissue in the breasts determines the amount of milk you’ll produce.”
1
The question should be answered directly in the class to all participants; however, the mother’s statement indicates some concerns about breastfeeding that might be explored privately later. Stating that motivated women tend to breastfeed successfully constitutes false reassurance; successful breastfeeding requires mastery, and some women have difficulty. Although noting that the client seems to have concerns about breastfeeding indicates that the nurse perceives the client’s concerns, this response is inappropriate in a class setting; the nurse should elicit more information privately later. The infant’s suckling and emptying of the breasts determine the amount of milk produced.

We have an expert-written solution to this problem!
A nurse keeps track of the immunization schedules for a childbearing family. Which type of nursing intervention is executed in this situation?
1
Acute care
2
Home care
3
Health promotion
4
Restorative and continuing care
3
In family nursing, maintaining adequate prenatal care to a childbearing family and adhering to immunization schedules are performed by a nurse as a part of health promotion. In family nursing, acute care involves preparing family members to assist in health care, or locating appropriate community resources. Investigating medication adherence, neurological issues, and gait and helping clients adapt ways to perform daily activities are interventions performed in-home care. In family nursing, restorative and continuing care help to maintain a client’s functional abilities within the context of the family.

A nursing instructor asks a nursing student to outline the factors that predispose adolescents to substance use. Which of these statements outlined by the student are correct? Select all that apply.
1
Adolescents believe that substance use makes them more mature.
2
Adolescents believe that substance use will improve appetite and sleep disturbances.
3
Adolescents believe that using mood-altering substances creates a sense of well-being.
4
Adolescents believe that substance use will help them achieve increased levels of performance.
5
Adolescents think that using substances will help them cope with worsening performance in school.
134
Some adolescents believe that substance abuse makes them more mature. Adolescents often believe that the use of mood-altering substances creates a sense of well-being. Adolescents also believe that substance use will result in an increased level of performance. Hence, all these statements outlined by the student are correct. Appetite and sleep disturbances and a decrease in school performance are warning signs of suicide among adolescents.

What activity beyond the capabilities of a toddler does the nurse conducting an evaluation expect of a 4-year-old child?
1
Naming several colors
2
Using three- and four-word sentences
3
Asking for the definitions of new words
4
Having a vocabulary of about 1500 words
4
As a result of expanded experiences and developing cognitive ability, the 4-year-old child should have a vocabulary of approximately 1500 words. At 2½ to 3 years of age children can name colors. At 3 years of age children use three- to four-word sentences. At 5 years of age children ask the definitions of new words.

One minute after birth a nurse assesses a newborn and auscultates a heart rate of 90 beats/min. The newborn has a strong, loud cry, moves all extremities well, and has acrocyanosis but is otherwise pink. What is this neonate’s Apgar score?
1
9
2
8
3
7
4
6
2
A heart rate slower than 100 beats/min receives 1 point, and color (acrocyanosis—body pink, extremities blue) receives 1 point; the respiratory rate, muscle tone, and reflex irritability each get a score of 2, for a total of 8. A score of 9 is too high. An Apgar score of 7 is too low, as is a score of 6.

Which breathing technique should the nurse instruct the client to use as the head of the fetus is crowning?
1
Shallow
2
Blowing
3
Slow chest
4
Modified paced
2
Blowing forcefully through the mouth controls the strong urge to push and allows for a controlled birth of the head. A shallow breathing pattern does not help control expulsion of the fetus. Slow chest breathing is used during the latent phase of the first stage of labor; it is not helpful in overcoming the urge to push. Modified paced breathing is used during active labor when the cervix is dilated 3 to 7 cm; it is not helpful in overcoming the urge to push.

Which actions by the nurse help set the stage for a patient-centered interview during the first visit after admission to the healthcare facility? Select all that apply.
1
Close the door after entering the room.
2
Greet the client using his or her last name.
3
Open the curtains to allow plenty of light in the room.
4
Introduce oneself with a smile and explain the reason for the visit.
5
Obtain an authorization from the client after the interview.
124
The nurse should maintain the client’s privacy by closing the door after entering the room. The nurse should maintain the dignity of the client by greeting the client using his or her last name. Smiling is a positive sign of warmth and immediacy when first establishing the nurse-client relationship. The nurse should explain his or her role in the providing care for the client. The nurse should ensure the room is adequately lit, comfortable, and soothing for the client. The nurse need not open the curtains to allow plenty of light in the room. The Health Insurance Portability and Accountability Act (HIPAA) requires the nurse to obtain an authorization from the client before collecting personal health data.

Which statement is true about toddlers?
1
Toddlers grow about 4.2 cm each year.
2
Toddlers develop a sense of autonomy.
3
Toddlers are aware of the danger of water.
4
Toddlers gain approximately 2 to 4 pounds (0.9 to 1.8 kg) each year.
2
According to Erikson, a sense of autonomy emerges during toddlerhood. Children strive for independence by using their developing muscles to do everything for themselves and becoming the master of their bodily functions. The average toddler grows 6.2 cm per year. Toddlers’ lack of awareness regarding the danger of water and their newly developed walking skills make drowning a major cause of accidental death in this age group. Toddlers gain approximately 5 to 7 pounds (2.3 to 3.2 kg) each year.

We have an expert-written solution to this problem!
While assessing the development of a child, the nurse finds that the child is imitating the actions of her elders. Which stage, according to Erickson’s developmental theory, does the nurse expect the child is in?
1
Initiative versus guilt
2
Industry versus inferiority
3
Identity versus role confusion
4
Autonomy versus shame and doubt
4
According to Erickson’s developmental theory, a child who is 1 to 3 years old is in the autonomy versus shame and doubt stage. The child will have control over his or her body and environment. Therefore, the child starts imitating elders in an attempt to learn and do things for himself or herself. A 3- to 6-year-old child is in the initiative versus guilt stage according to Erickson’s theory of development. The child who is in the initiative versus guilt stage will try to explore things with all the senses. A 6- to 12-year-old child is in the industry versus inferiority stage according to Erickson’s theory of development. Industry versus inferiority is a higher stage of development in which the child will perform tasks to attain real achievements and not as mere imitation. A child who is 12 to 18 years old is in the identity versus role confusion stage of Erickson’s developmental theory. In this stage, the child becomes preoccupied with personal appearance.

A nurse educates a group of teenagers about the physical changes they will experience. What statement made by the teenager shows inadequate learning?
1
“My shoulders will change in width.”
2
“My reproductive system will develop.”
3
“My body fat distribution will be altered.”
4
“My bones will grow at a decreased rate.”
4
When the teenager states that his or her rate of bone growth will decrease, this statement indicates inadequate learning. Teenagers will experience a change in shoulder width. A teenager’s reproductive system develops, and his or her body fat distribution is altered.

The nurse is assessing the client admitted with diabetic ketoacidosis. Which statement made by the client indicates a need for further education on sick day management?
1
“I need to stop taking my insulin when I am ill because I am not eating.”
2
“I will check my urine for ketones when my blood sugar is over 250.”
3
“I will try and take in Gatorade and water when I am sick.”
4
“I will continue all my insulin including my glargine when I am sick.”
1
The diabetic client’s metabolic needs will require the same amount of insulin and sometimes more when in a stressed state, including illness. The client checking the urine for ketones when blood sugar is over 250, drinking water and Gatorade, and continuing insulin indicate that the client has an understanding of the basic sick day rules.

A client is admitted to the hospital with the diagnosis of acute salmonellosis. Which priority medication will the nurse prepare to administer?
1
Opioids
2
Antacids
3
Electrolytes
4
Antidiarrheals
3
Fluids of dextrose and normal saline and electrolytes are administered to prevent profound dehydration caused by an excessive loss of water and electrolytes through diarrheal output. Opioids and antidiarrheals are not used when there is a possibility of bacterial infection because slowed peristalsis decreases excretion of the salmonella organism. Salmonellosis is an infection, not a condition caused by hyperacidity and does not require antacids.

We have an expert-written solution to this problem!
The nurse is assessing a newborn of 33 weeks’ gestation. Which sign alerts the nurse to notify the health care provider?
1
Flaring nares
2
Acrocyanosis
3
Heartbeat of 140 beats/min
4
Respirations of 40 breaths/min
1
Preterm neonates are prone to respiratory distress; flaring nares are a compensatory mechanism in a neonate with respiratory distress syndrome, the body’s attempt to lessen resistance of narrow nasal passages and to increase oxygen intake. Acrocyanosis is not related to respiratory distress but is caused by vasomotor instability; this is an expected occurrence in the newborn. A heartbeat of 140 beats/min is an expected finding in the newborn. A respiratory rate of 40 breaths/min is an expected finding in the newborn.

A nurse in the coronary care unit (CCU) identifies ventricular fibrillation on a client’s cardiac monitor. What intervention is the priority?
1
Elective cardioversion
2
Immediate defibrillation
3
An intramuscular (IM) injection of digoxin
4
An intravenous (IV) line for emergency medications
2
When ventricular fibrillation is verified, the first intervention is defibrillation; it is the only measure that will terminate this lethal dysrhythmia. Elective cardioversion delivers a shock during the R wave; because there is no R wave in ventricular fibrillation, the dysrhythmia will continue and death will result. Digitalis preparations are not used to treat ventricular dysrhythmias. If not already in place, an IV line should be inserted after the client is defibrillated.

Which intervention should the nurse provide while caring for an older adult client who is reported to have decreased estrogen production?
1
Use minimal tape on client’s skin.
2
Cover the client with warm clothing.
3
Perform blood glucose test for the client
4
Monitor for bradycardia.
1
Decreased estrogen production associated with aging affects skin texture and makes the skin dry and thin. Therefore the nurse should refrain from using tape on the client’s skin to prevent skin injury. Warm clothing and monitoring heart rate are needed for older adult clients with decreased general metabolism or hypothyroidism but are not relevant with estrogen deficiency. A client exhibiting signs of decreased glucose tolerance, such as slow wound healing and recurrent yeast infections, should be tested for blood glucose levels.

An older adult who was in a motor vehicle collision is brought to the emergency department via ambulance. The client exhibits a decreased level of consciousness, and the nurse identifies serosanguineous drainage from the client’s left ear. Which action should the nurse take?
1
Irrigate the ear with normal saline.
2
Place a sterile pad over the external ear.
3
Gently insert a cotton-tipped swab in the ear canal.
4
Pack a cotton ball in the external meatus of the ear.
2
A lowered level of consciousness indicates a potential head injury, and drainage from an ear may be cerebrospinal fluid; a sterile pad gently affixed over the ear will absorb drainage and prevent infection and can help detect the halo sign. Irrigating the ear with normal saline is contraindicated if a cerebrospinal fluid leak is suspected. Packing a cotton ball in the external meatus of the ear or inserting a cotton-tipped swab may be traumatic and may injure the ear further; also, it will obstruct free flow of drainage.

A client cannot understand how syphilis was contracted because there has been no sexual activity for several days. Which length of time associated with the incubation of syphilis should the nurse include in the teaching plan?
1
1 week
2
4 months
3
2 to 6 weeks
4
48 to 72 hours
3
Although the usual incubation period of syphilis is about 3 weeks, clinical symptoms may appear as early as 9 days or as long as 3 months after exposure. The usual incubation period is 21 days.

The nurse suspects risk of dysfunctional uterine bleeding in an adolescent. Which statement made by the adolescent would support the nurse’s suspicion?
1
“I am losing weight unintentionally.”
2
“I take herbal medicine for menstrual pain.”
3
“I have severe pain during my menstrual periods.”
4
“I am excited to get pregnant and can’t wait.”
3
Acute or chronic pain during the menstrual cycle is a risk factor for dysfunctional uterine bleeding. Gaining weight, not losing weight, is a risk factor for dysfunctional uterine bleeding. Herbal medicine is a complementary and alternative medicine and can help relieve pain, but on its own, this statement does not necessarily support the nurse’s suspected diagnosis. Whether or not a client desires a pregnancy is unrelated to dysfunctional uterine bleeding.

While assessing a client during an ophthalmic pupillary physical examination, the primary healthcare provider observes a noticeable difference in the size of pupils in the client. Which term should the nurse use to describe this condition?
1
Mydriasis
2
Hyperopia
3
Anisocoria
4
Arcus senilis
3
A noticeable difference in the size of the pupils of the clients is known as anisocoria, a normal condition in about 5% of people. The normal diameter of the pupil is between 3 and 5 mm; clients with hyperopia have smaller pupils with a diameter of less than 3 mm. Mydriasis is the process of pupillary dilation. Arcus senilis is an opaque bluish-white ring within the outer edge of the cornea caused by the presence of fat deposits.

After assessing a dark-skinned client, the nurse concludes that the client has cyanosis. Which assessment color variation helped the nurse reach this conclusion?
1
Grey color
2
Purple color
3
Dark red color
4
Purple-to-brownish color
1
In dark-colored skin, cyanosis can be identified by a grey color, which is mostly seen in the conjunctiva of the eye. A purple skin color is an indication of erythema. In a light-skinned client, a dark red color is an indicator of ecchymosis. A purple-to-brownish color identifies ecchymosis.

What complications are associated with excessive weight gain during pregnancy in adolescents? Select all that apply.
1
Fetal anemia
2
Preterm labor
3
Cesarean delivery
4
Maternal mortality
5
Postpartum obesity
235
Excessive weight gain during pregnancy is associated with such complications of labor and delivery as preterm labor and cesarean delivery. It is also linked with postpartum obesity and its associated health risks. Excessive weight gain during pregnancy is associated with maternal (not fetal) anemia and infant (not maternal) mortality.

We have an expert-written solution to this problem!
A client has thin, dark-red vertical lines about 1 to 3 mm long in the nails. Which diseases are associated with this physiologic alteration in the client? Select all that apply.
1
Psoriasis
2
Trichinosis
3
Cardiac failure
4
Diabetes mellitus
5
Bacterial endocarditis
25
Thin, dark-red vertical lines about 1 to 3 mm long in the nails are associated with trichinosis (parasitic disease) and bacterial endocarditis (infection of the innermost layer of the heart and heart valves). Psoriasis, diabetes mellitus, and cardiac failure are associated with yellow-brown discoloration of the nails.

Which clinical indicator is the nurse most likely to identify when completing a history and physical assessment of a client with complete heart block?
1
Syncope
2
Headache
3
Tachycardia
4
Hemiparesis
1
With complete atrioventricular block, the ventricles take over the pacemaker function in the heart but at a much slower rate than that of the sinoatrial (SA) node. As a result, there is decreased cerebral circulation, causing syncope. Headache is not related to heart block. The heart rate usually is slow because the ventricular rhythm is not initiated by the SA node. Hemiparesis is associated with a brain attack (cerebrovascular accident).

We have an expert-written solution to this problem!
A nurse is teaching a client with pruritus about personal care interventions. Which statement made by the client indicates the nurse needs to intervene?
1
“I will trim my fingernails regularly.”
2
“I will wear mittens or splints at night.”
3
“I will apply moisturizing lotion after bath.”
4
“I will not file the edges of fingernails.”
4
The nurse should intervene if the client states that fingernail edges will not be filed to correct this misconception. Rough edges of fingernails should regularly be filed to prevent skin damage and secondary infection. All the other statements are correct and require no follow up. Regular trimming of the nails, wearing of splints at night, and application of moisturizing lotion after bath are some interventions to be taken by the client to protect skin in pruritus.

The nurse examines a client who has had a cesarean birth. It has been 3 days since the birth, and the client is about to be discharged. Where does the nurse expect the fundus to be located?
1
One fingerbreadth below the umbilicus
2
Two fingerbreadths below the umbilicus
3
Three fingerbreadths below the umbilicus
4
Four fingerbreadths below the umbilicus
3
The fundus descends one fingerbreadth per day from the first postpartum day. If the fundus is one or two fingerbreadths below the umbilicus, the nurse should suspect that involution has been delayed, and further investigation is required. Although a fundus four fingerbreadths below the umbilicus is not expected, it is a benign occurrence.

In the immediate period after admission to the burn unit with severe burns, a 5-year-old child requests a drink of milk. What is the most appropriate nursing intervention?
1
Giving ice chips as desired
2
Permitting milk if it has been iced
3
Maintaining NPO status for 24 to 48 hours
4
Limiting oral fluid to 15 mL every 4 hours
3
Nothing-by-mouth (NPO) status is maintained during the early emergency/resuscitative phase because of the probability of paralytic ileus. It is unsafe to offer ice chips because the fluid that is ingested interferes with monitoring and control of the child’s fluid and electrolyte status. It is unsafe to offer oral fluids, not only because of the danger of paralytic ileus but also because they interfere with monitoring and control of the child’s fluid and electrolyte status.

A client is admitted to the hospital with a recurrence of chronic arterial insufficiency of the legs. Which clinical manifestations does the nurse expect to identify when performing an admission history and physical?
1
Edema of the feet and ankles
2
Reddened and painful areas on the calves
3
Pain when exercising and thickening of the toenails
4
Ulcers around the ankles and reports of a dull ache in the legs
3
Inadequate oxygenation of tissues of the affected limb causes intermittent claudication and thickened toenails. Edema of the feet and ankles occurs with venous, not arterial, insufficiency in which the veins are unable to return blood adequately from the affected legs to the heart; also, dependent edema may be associated with decreased cardiac output related to heart failure. Reddened and painful areas on the calves are adaptations related to thrombophlebitis, a venous rather than arterial problem. Ulcers around the ankles and reports of a dull ache in the legs occur with venous, not arterial, insufficiency in which the veins are unable to return blood adequately from the affected legs to the heart; also, these adaptations may be associated with decreased cardiac output related to heart failure.

We have an expert-written solution to this problem!
Arrange the order of steps in bone healing.
1.
Hematoma formation

2.
Callus formation

3.
Invasion of granulation tissue

4.
Osteoblastic proliferation

5.
Bone remodeling
The first stage of bone healing is the formation of a hematoma because bone is vascular. In the next stage, the granulation tissue begins to invade the hematoma. Vascular and cellular proliferation then occurs and new tissue is formed called callus. Next, osteoblastic proliferation occurs: the callus gradually resorbs and transforms into a bone. Finally, bone remodeling occurs and bone healing is complete.

A nurse is performing an eye assessment in an older adult. The older adult is unable to see near objects. Which conditions may be suspected in the older adult? Select all that apply.
1
Cataract
2
Glaucoma
3
Hyperopia
4
Presbyopia
5
Macular degeneration
34
In hyperopia, the client has farsightedness. In this condition, the client is unable to see near objects. Presbyopia is an impaired near vision that may occur with aging. Therefore, the nurse can suspect either of the conditions. In cataracts, there is an increased opacity of the lens that blocks light rays from entering the eye, leading to impaired vision. Glaucoma is a condition in which there is intraocular structural damage resulting from elevated intraocular pressure. Macular degeneration is caused due to blurred central vision that often occurs suddenly. This is caused by a progressive degeneration of the center of the retina.

A client with cancer of the prostate requests the urinal at frequent intervals but either does not void or voids in very small amounts. Which does the nurse conclude is most likely the causative factor?
1
Edema
2
Dysuria
3
Retention
4
Suppression
3
An enlarged prostate constricts the urethra, interfering with urine flow and causing retention. When the bladder fills and approaches capacity, small amounts can be voided, but the bladder never empties completely. Edema does not cause the client to void frequently in small amounts. Dysuria is painful or difficult urination, which is not part of the client’s responses. The urge to void is caused by stimulation of the stretch receptors as the bladder fills with urine; in suppression, little or no urine is produced.

While assessing a client’s skin, the primary healthcare provider finds honey-colored crust surrounded by erythema. Which bacterial infection does the nurse anticipate in the client?
1
Impetigo
2
Folliculitis
3
Carbuncle
4
Erysipelas
1
Impetigo is a bacterial infection that has a honey-colored crust surrounded by erythema. Folliculitis is a bacterial infection with a small pustule at the hair follicle opening with minimal erythema. Carbuncle is a bacterial infection in which many pustules appear in an erythematous area, most commonly at the nape of neck. Erysipelas is a bacterial infection in which a red, hot, sharply demarcated plaque is indurated and painful.

A client is admitted to the cardiac care unit with a diagnosis of myocardial infarction. The client asks the nurse, “What is causing the pain I am having?” Which explanation of the cause of the pain is the most appropriate response by the nurse?
1
Compression of the heart muscle
2
Release of myocardial isoenzymes
3
Rapid vasodilation of the coronary arteries
4
Inadequate oxygenation of the myocardium
4
Cessation of the blood flow that normally carries oxygen to the myocardium results in pain because of ischemia of myocardial tissue. Myocardial infarction does not involve compression of the heart. The release of myocardial isoenzymes is an indication of myocardial damage; this does not cause myocardial pain. Vasodilation will increase perfusion and contribute to pain relief, not cause myocardial pain.

A nurse is writing a teaching plan about osteoporosis. How would the nurse best explain what osteoporosis is?
1
It is avascular necrosis.
2
It is caused by pathologic fractures.
3
It is hyperplasia of osteoblasts.
4
It involves a decrease in bone substance.
4
Osteoporosis involves as defect in bone matrix formation that weakens the bones, making them unable to withstand usual functional stresses. Avascular necrosis is death of bone tissue that results from reduced circulation to bone. Pathologic fractures can result from osteoporosis. Hyperplasia of osteoblasts is not related to osteoporosis. This occurs during bone healing.

A client is diagnosed with hyperthyroidism and is treated with 131I. Before discharge the nurse teaches the client to observe for signs and symptoms of therapy-induced hypothyroidism. Which signs and symptoms should be included in the teaching? Select all that apply.
1
Fatigue
2
Dry skin
3
Insomnia
4
Intolerance to heat
5
Progressive weight gain
125
Fatigue is caused by a decreased metabolic rate associated with hypothyroidism. Dry skin most likely is caused by decreased glandular function associated with hypothyroidism. Progressive weight gain is associated with hypothyroidism in response to a decrease in the metabolic rate because of insufficient thyroid hormone. Insomnia is associated with hyperthyroidism, not hypothyroidism, in response to an increase in the metabolic rate because of excessive thyroid hormone. Intolerance to heat is associated with hyperthyroidism, not hypothyroidism, in response to an increase in the metabolic rate because of excessive thyroid hormone.

A nurse is administering hydroxyzine to a client. The nurse will monitor the client for which common side effects of this drug?
1
Ataxia and confusion
2
Drowsiness and dry mouth
3
Vertigo and impaired vision
4
Slurred speech and headache
2
Hydroxyzine suppresses activity in key regions of the subcortical area of the central nervous system; it also has antihistaminic and anticholinergic effects. Ataxia and confusion, vertigo and impaired vision, and slurred speech and headache are not associated with hydroxyzine.

A client with myasthenia gravis is to receive immunosuppressive therapy. What assures the nurse that this therapy will be effective?
1
Inhibits the breakdown of acetylcholine at the neuromuscular junction
2
Stimulates the production of acetylcholine at the neuromuscular junction
3
Decreases the production of autoantibodies that attack acetylcholine receptors
4
Promotes the removal of autoantibodies that impair the transmission of impulses
3
Steroids decrease the body’s immune response, limiting the production of antibodies that attack acetylcholine receptors at the neuromuscular junction. Inhibiting the breakdown of acetylcholine at the neuromuscular junction is the action of anticholinergic medications. Stimulating the production of acetylcholine at the neuromuscular junction is not the action of immunosuppressives. Promoting the removal of autoantibodies that impair the transmission of impulses is the rationale for plasmapheresis.

A client develops thrombophlebitis in the right calf. Bed rest is prescribed, and an IV of heparin is initiated. What drug action will the nurse include when describing the purpose of this drug to the client?
1
Prevents extension of the clot
2
Reduces the size of the thrombus
3
Dissolves the blood clot in the vein
4
Facilitates absorption of red blood cells
1
Heparin interferes with activation of prothrombin to thrombin and inhibits aggregation of platelets. Heparin does not reduce the size of a thrombus. Heparin does not dissolve blood clots in the veins. Heparin does not facilitate the absorption of red blood cells.

Which client is contraindicated for the use of uterine stimulants in early pregnancy?
1
Client with hypertension
2
Client with bleeding disorder
3
Client with cardiovascular problem
4
Client with thromboembolic disorder
2
Clients with bleeding disorder are contraindicated for administration of uterine stimulants as they may increase the risk of bleeding. Clients with hypertension are contraindicated for methylergonovine therapy as it may precipitate seizures or stroke. Clients with cardiovascular problems are contraindicated for nifedipine therapy. Clients with thromboembolic disorders are contraindicated for administration of selective estrogen receptor modulators.

We have an expert-written solution to this problem!
What primary nursing actions are used to prevent pediatric medication errors? Select all that apply.
1
Carefully decipher illegibly written orders.
2
Have full knowledge of the medication, including its on- and off-label uses.
3
Assume that the prescriber is correct and administer the medications as ordered.
4
Always confirm information about the client before delivering a dose.
5
Make sure all the orders are clear and well understood during a shift change.
245
The nurse should know the label and off-label use of any drug prescribed to the client. The nurse should confirm information about the client every time before administering medication to ensure that the client receives the correct medication. The nurse should make sure all the orders are clear and well understood during shift changes to avoid miscommunication. The nurse should not try to decipher illegibly written orders. The nurse should never assume that the prescriber is correct; instead, the nurse should investigate until all ambiguities are resolved.

The nurse cares for a client with bipolar disorder who is receiving drug therapy. The laboratory report reveals that the client’s serum sodium level is 132 mEq/L (132 mmol/L). Which drug might have led to this condition?
1
Lithium
2
Bupropion
3
Fluoxetine
4
Nortriptyline
3
A serum sodium level of 132mEq/L (132 mmol/L) indicates hyponatremia. Fluoxetine is a serotonin reuptake inhibitor that may lead to hyponatremia. Lithium is a mood stabilizer used to treat bipolar disorder; it does not lead to hyponatremia. Bupropion is an atypical antidepressant that does not cause hyponatremia. Nortriptyline is a tricyclic antidepressant used to treat bipolar disorder that does not lead to hyponatremia.

A client receiving morphine is being monitored by the nurse for signs and symptoms of overdose. Which clinical findings support a conclusion of overdose? Select all that apply.
1
Polyuria
2
Sedation
3
Bradycardia
4
Dilated pupils
5
Slow respirations
235
The central nervous system (CNS) depressant effect of morphine causes sedation. The CNS depressant effect of morphine causes bradycardia. The CNS depressant effect of morphine causes bradypnea. Morphine does not increase urine output. Morphine causes constriction of pupils.

A client with a history of type 1 diabetes is diagnosed with heart failure. Digoxin is prescribed. What is an important nursing action associated with this drug?
1
Administer the digoxin 1 hour after the client’s morning insulin
2
Monitor the client for atrial fibrillation and first-degree heart block
3
Administer the medication with 8 ounces (240 mL) of orange juice
4
Withhold the medication if the apical pulse rate is greater than 60 beats/min
2
The speed of conduction is decreased when digoxin is given, and this can result in premature beats, atrial fibrillation, and first-degree heart block. Digoxin does not deplete potassium and therefore orange juice does not need to be given; orange juice is high in calories and needs to be calculated in the diet. Insulin and digoxin can be given at the same time. The purpose of the drug is to reduce a rapid heart rate and therefore should be administered; it should be withheld when the client’s heart rate decreases below a parameter set by the healthcare provider (e.g., 60 beats/min).

Which information is important to obtain during the nursing assessment of a female who is taking oral contraceptives? Select all that apply.
1
Family stability and socioeconomic status
2
Maternal history for estimated gestation
3
History of vascular or thromboembolic disorder
4
Drug interactions leading to a decreased effect of oral contraceptives
5
Prescription of a medication that may have its therapeutic effects decreased if taken with oral contraceptives
345
A history of vascular or thromboembolic disorders may increase the risk of complications with estrogen therapy. Drug interactions should be monitored and reduced. Drug interactions may decrease the therapeutic effects of concurrent drugs or oral contraceptives. An assessment of family stability and socioeconomic status is inappropriate. An assessment of maternal history for estimated gestation is irrelevant in oral contraceptive therapy.

A client has primary open-angle glaucoma. The nurse expects that the client will receive a prescription for which eyedrops?
1
Tetracaine
2
Cyclopentolate
3
Timolol maleate
4
Atropine sulfate
3
Timolol maleate is a beta-adrenergic antagonist that decreases aqueous humor production and increases outflow, thereby reducing intraocular pressure. Tetracaine is a topical anesthetic; it will not reduce the increased intraocular pressure associated with glaucoma. Cyclopentolate is contraindicated because it dilates the pupil and paralyzes ciliary muscles. Atropine sulfate, a mydriatic, is contraindicated because it dilates the pupil, obstructing drainage, which increases intraocular pressure.

A nurse is preparing a client for discharge from the emergency department. Which client statement provides evidence that the client understands the instructions for the prescribed high-dose ampicillin?
1
“I should take this medication with meals.”
2
“I can stop taking this medication when I feel better.”
3
“I will miss eating my yogurt while taking this medication.”
4
“I must increase my intake of fluids while taking this medication.”
4
Because penicillin in high doses is nephrotoxic, keeping hydrated helps flush the kidneys as the drug is excreted. It should be taken on an empty stomach for best absorption. Stopping this medication when the client feels better is contraindicated; completing the medication treatment as prescribed prevents the development of resistant strains of bacteria. Dietary restrictions are not imposed while this medication is taken.

Potassium supplements are prescribed for a client receiving diuretic therapy. What client statement indicates that the teaching about potassium supplements is understood?
1
“I will report any abdominal distress.”
2
“I should use salt substitutes with my food.”
3
“The drug must be taken on an empty stomach.”
4
“The dosage is correct if my urine output increases.”
1
Potassium supplements can cause gastrointestinal ulceration and bleeding. Most salt substitutes contain potassium, and their use with potassium supplements can cause hyperkalemia. Because they can be irritating to the stomach, potassium supplements should not be taken on an empty stomach. An increase in urine output is the therapeutic effect of diuretic therapy, not potassium supplements. An adverse effect of potassium supplements is oliguria.

A nurse is caring for a variety of clients. In which client is it most essential for the nurse to implement measures to prevent pulmonary embolism?
1
A 59-year-old who had a knee replacement
2
A 60-year-old who has bacterial pneumonia
3
A 68-year-old who had emergency dental surgery
4
A 76-year-old who has a history of thrombocytopenia
1
Clients who have had a joint replacement have decreased mobility; they are at risk for developing thrombophlebitis, which may lead to pulmonary embolism if the clot becomes dislodged into the circulation. Bacterial pneumonia and emergency dental surgery are not associated with an increased risk for pulmonary embolism. A history of thrombocytopenia leads to a decreased ability to clot, so it increases the risk of bleeding but decreases the risk of a thrombus or embolus.

A nurse is assessing a client with Parkinson disease. Which assessment finding indicates the presence of bradykinesia?
1
Intention tremor
2
Muscle flaccidity
3
Paralysis of the limbs
4
Lack of spontaneous movement
4
Bradykinesia is a slowing down in the initiation and execution of movement. Tremors are more prominent at rest and are known as nonintention, not intention, tremors. Cogwheel rigidity, not flaccidity, occurs because the disorder causes sustained muscle contractions. The limbs are rigid and move with a jerky quality; the limbs are not paralyzed.

Which clinical manifestations in a client indicate hyperfunctional thyroid gland? Select all that apply.
1
Anemia
2
Diarrhea
3
Weight loss
4
Decreased appetite
5
Distant heart sounds
23
Diarrhea and weight loss are the characteristic manifestations of a hyperfunctional thyroid gland. Anemia is seen in a client with a hypofunctional thyroid and decreased levels of thyroid hormone. Decreased appetite and distant heart sounds are symptoms of a hypofunctional thyroid gland.

We have an expert-written solution to this problem!
What is the effect of parathyroid hormone on bones? Select all that apply.
1
Increased bone breakdown
2
Increased serum calcium levels
3
Increased sodium and phosphorus excretion
4
Increased absorption of calcium and phosphorus
5
Increased net release of calcium and phosphorus
125
Parathyroid hormone increases bone breakdown, which increases serum calcium levels. Parathyroid hormone increases net release of calcium and phosphorus from bone into the extracellular fluid. It increases sodium and phosphorus excretion by the kidneys, not in the bone and increases absorption of calcium and phosphorus in the gastrointestinal tract by using activated vitamin D. However, this increased absorption of calcium and phosphorus is not related to the bone.

We have an expert-written solution to this problem!
The client diagnosed with a fistula between the bowel and urinary bladder reports passing air and bubbles during urination. What does the nurse anticipate the client’s condition to be?
1
Nocturia
2
Oliguria
3
Pneumaturia
4
Stress incontinence
3
The occurrence of gas along with urination is called pneumaturia and could result from the formation of a fistula between the bowel and urinary bladder. Frequent urination during the night is called nocturia and is associated with conditions such as heart failure and diabetes mellitus. In medical conditions such as severe dehydration and shock, the urine output is reduced to 100 to 400 mL/day and this is termed oliguria. Weak sphincter control, urinary retention, and estrogen deficiency are some causes for stress incontinence or involuntary urination during increased pressure situations.

The nurse is asking a client with arthritis questions in order to collect information. Which questions asked by the nurse are closed-ended questions? Select all that apply.
1
“Are you having pain?”
2
“Tell me how your pain has been.”
3
“Describe how your husband is helping you at home.”
4
“Do you think the medication is helping you to get pain relief?”
5
“Give me an example of a method which helps you to get pain relief at home.”
14
Closed-ended questions limit the answers to one or two words. These queries help to identify specific problem areas and provide additional questions. When the nurse asks, “Are you having pain?” the client can reply either yes or no. Similarly when the nurse asks the client “Do you think the medication is helping you to get pain relief?” the answers can be either yes or no. Therefore these two questions are closed-ended questions. Open ended-questions are client-centered and require a complete explanation.

The nurse is assessing a client with severe cirrhosis and discovers fetor hepaticus. What did the nurse assess?
1
Urine
2
Stool
3
Hands
4
Breath
4
The client’s breath has a sweet odor (fetor hepaticus) because the liver is not metabolizing the food, especially proteins. The urine is dark. The stool is clay-colored. The hands develop asterixis or flapping tremors.

The nurse is performing a skin assessment of a client. Which findings in the client may indicate a risk of skin cancer? Select all that apply.
1
Lesion
2
Lumps
3
Rashes
4
Bruising
5
Dryness
123
Lesions on the skin that take a long time to heal may indicate skin cancer. Lumps and rashes on the skin are characteristics of skin cancer. Bruising may indicate a bleeding disorder or injury. Dryness of the skin may be due to excessive bathing and use of harsh soaps.

A client diagnosed with Bell palsy has many questions about the course of the disorder. Which information should the nurse share with the client?
1
Cool compresses decrease facial involvement.
2
Pain occurs with transient ischemic attacks (TIAs).
3
Most clients recover from the effects in several weeks.
4
Body changes should be expected with residual effects.
3
The client should be assured that the symptoms are not caused by a stroke; the majority of clients recover in a few weeks. Moist heat, not a cool compress, increases blood circulation to the nerve. Bell palsy is not caused by a TIA. Paresis or paralysis of cranial nerve VII occurs; pain is usually present. The majority of clients recover without residual effects; occasionally some clients are left with evidence of Bell palsy. Exercises may help to maintain muscle tone; also, surgery may be necessary.

A client who is in the first trimester is being discharged after a week of hospitalization for hyperemesis gravidarum. She is to be maintained at home with rehydration infusion therapy. What is the priority nursing activity for the home health nurse?
1
Determining fetal well-being
2
Monitoring for signs of infection
3
Monitoring the client for signs of electrolyte imbalances
4
Teaching about changes in nutritional needs during pregnancy
3
Rehydration fluids contain only saline and dextrose; if the client continues to vomit, she will lose electrolytes. Monitoring the fetus is not the priority at this time. Although there is a danger of infection when an intravenous line is in place, monitoring for it is not the priority. Teaching about nutritional needs is a nontherapeutic nursing action while the client is still vomiting.

A client with a reddish-blue generalized skin alteration is hospitalized. Laboratory findings show an increase in the overall amount of hemoglobin. Which condition might the nurse suspect?
1
Albinism
2
Addison’s disease
3
Polycythemia vera
4
Methemoglobinemia
3
The generalized reddish-blue skin alteration is occurring due to increased overall hemoglobin and may be associated with polycythemia vera. Albinism may found with decreased pigmentation of the skin due to the genetically determined defect of melanocyte. Addison’s disease may be associated with increased melanin production, which may result in a brown skin discoloration. Cyanosis resulting in a blue discoloration of the skin may signify methemoglobinemia.

Which nursing intervention helps to prevent medication errors in children? Select all that apply.
1
Encouraging the use of brand names
2
Promoting the use of abbreviations and acronyms
3
Minimizing the use of verbal and telephone orders
4
Carefully reading all labels for accuracy and checking expiration dates
5
Recording the client’s weight before carrying out the medication order
345
All labels and warnings should be read carefully to avoid the administration of expired medications. The client’s weight should be recorded to help decrease errors in dosage. The use of generic names should be encouraged to avoid medication errors. Brand names may cause confusion and may lead to medication errors. The use of abbreviations and acronyms should be avoided because of the risk of confusion. Minimizing the use of verbal and telephone orders will also help prevent medication errors in children. Verbal and telephone orders should be read back to the healthcare provider prescribing the medication(s), but this is not always done, so errors are made.

What is the focus of the nurse’s anticipatory guidance during the first trimester of pregnancy?
1
Birthing process
2
Signs of complications
3
Physical changes of pregnancy
4
Role transition into parenthood
3
Increasing the client’s knowledge of physical changes during pregnancy is appropriate during the first trimester. The details of labor and birth are taught in the last trimester. The first trimester is too soon to be teaching danger signs; the client should be taught what to expect in a healthy pregnancy first. Concerns about role transition into parenthood should be addressed in the third trimester.

A nurse is discussing areas of potential concern, such as anemia, with several parents of 6-month-old infants. What should the nurse tell them is the most common cause of anemia in 1-year-olds?
1
Thalassemia
2
Lead poisoning
3
Iron deficiency
4
Sickle shape of blood cells
3
Breast milk and unfortified infant formulas increase the risk for iron-deficiency anemia in infants. Cow’s milk, which is introduced at 1 year of age, is also low in iron and may cause iron-deficiency anemia unless iron supplements or iron-rich solid foods are added to the diet. Thalassemia is a genetic disease that affects specific populations and is not a common disorder. Lead poisoning usually occurs in children older than 1 year, and its prevalence is less than that of iron-deficiency anemia. Sickle cell anemia is a genetic disease that affects specific populations and is not as common as iron-deficiency anemia.

A 45-year-old client is scheduled to undergo a hysterectomy and expresses concern because she has heard from friends that she will experience severe symptoms of menopause after surgery. What is the nurse’s most appropriate response?
1
“You’re right, but there are medicines you can take that will ease the symptoms.”
2
“Sometimes that happens in women of your age, but you don’t need to worry about it right now.”
3
“You should probably talk to your surgeon, because I am not allowed to discuss this with you.”
4
“Women may experience symptoms of menopause if their ovaries are removed with their uterus.”
4
A hysterectomy involves only removal of the uterus. The ovaries, which secrete estrogen and progesterone, are not removed. Therefore menopause will not be precipitated but will occur naturally. Surgical menopause is precipitated by the removal of the ovaries, not the uterus. When the ovaries are removed, an older woman might have less severe symptoms than a younger woman; however, in this instance the ovaries are not removed. Telling the client that she needs to talk to her surgeon does not answer the question. The nurse should serve as a resource.

After a cesarean birth a nurse performs fundal checks every 15 minutes. The nurse determines that the fundus is soft and boggy. What is the priority nursing action at this time?
1
Elevating the client’s legs
2
Massaging the client’s fundus
3
Increasing the client’s oxytocin drip rate
4
Examining the client’s perineum for bleeding
2
Gentle massage stimulates muscle fibers, resulting in firming the tone of the fundus; it also helps expel any clots that may be interfering with contraction of the fundus. Elevating the client’s legs will increase return of blood from the extremities but will not improve the tone of the client’s fundus. Increasing the client’s oxytocin drip rate will be done if uterine massage is ineffective. Examining the client’s perineum for bleeding should not be the first action at this time; gentle massage to contract the fundus is the priority.

The diet prescribed for a client allows for 190 grams of carbohydrates, 90 grams of fat, and 100 grams of protein. The nurse calculates that this diet contains approximately how many calories?
1
920
2
1970
3
2470
4
2970
2
This diet contains approximately 1970 calories. There are 9 calories in each gram of fat and 4 calories in each gram of carbohydrates and protein. 190 x 4 = 760; 90 x 9 = 810; 100 x 4 = 400; 760 + 810+ 400 = 1970. 920 calories is an incorrect calculation; this is too few calories. 2470 and 2970 are incorrect calculations; both are too many calories.

Which statement is true of the middle childhood stage according to Kohlberg’s development of moral reasoning? Select all that apply.
1
The child is in the conventional level.
2
The child is in the post-conventional level.
3
The child develops a social contract orientation.
4
The child develops a good boy-nice girl orientation.
5
The child develops a punishment-obedience orientation.
14
According to Kohlberg’s development of moral reasoning, in the middle childhood stage, the child is in the conventional level and develops a good boy-nice girl orientation. The child is in the post-conventional level and develops a social contract orientation in the adolescence developmental stage. The child develops punishment-obedience orientation in the toddler stage.

The nurse is admitting a confused 80-year-old client to the mental health unit. Which is one factor associated with the aging process?
1
Slowing of responses
2
Changing of personality
3
Lowering of intelligence
4
Diminution of long-term memory
1
Neurologic responses are slowed because of reduced sensory-receptor sensitivity. Excluding pathologic processes, the personality will be consistent with that of earlier years. There is no loss of intellectual ability unless there is a pathologic problem. Short-term, not long-term, memory is reduced because of a shortened attention span, delayed transmission of information to the brain, and perceptual deficits.

An older adult client who is accustomed to taking enemas periodically to avoid constipation is admitted to a long-term care facility. In addition to medications, the healthcare provider prescribes bed rest and a regular diet. Which action should be implemented initially to help prevent the client from developing constipation?
1
Arrange to have enemas prescribed for the client
2
Obtain a prescription for a daily laxative for the client
3
Place a commode by the bedside to facilitate defecation
4
Offer a large glass of prune juice with warm water each morning
4
Prune juice and warm water can be administered by the nurse to promote defecation. Prune juice irritates the bowel mucosa, stimulating peristalsis. Fiber in the diet increases fecal volume, which stimulates intestinal motility and the reflex for defecation. Enemas should be avoided because they can promote dependency and can result in electrolyte imbalance. The routine use of laxatives promotes dependency. The client is bedbound and is unable to use a commode.

A client seeking advice regarding contraception asks a nurse to explain how an intrauterine device (IUD) prevents pregnancy. How should the nurse respond?
1
“It covers the entrance to the cervical os.”
2
“The openings to the fallopian tubes are blocked.”
3
“The sperm are kept from reaching the vagin*.”
4
“It produces a spermicidal intrauterine environment.”
4
Intrauterine devices produce a spermicidal intrauterine environment. A copper IUD inflames the endometrium, damaging or killing sperm and preventing fertilization and/or implantation. A levonorgestrel-releasing IUD damages sperm and causes the endometrium to atrophy, thus preventing fertilization and/implantation. A diaphragm blocks the cervical os. The IUD does not act by blocking the openings to the fallopian tubes. Preventing sperm from reaching the vagin* is the function of a condom.

Which should the school nurse include in a teaching session with elementary school students related to nutritional intake and activity?
1
Eating the same types of food each day
2
Playing video games every day after school
3
Choosing high-fat foods for an after school snack
4
Consuming foods that are nutrient-rich, such as fruits
4
The school nurse should encourage elementary school students to consume foods that are nutrient-rich, such as fruits. A variety of foods should also be encouraged versus eating the same types of food each day. The nurse would discourage playing video games each day after school and would instead encourage physical activity. The nurse would also teach the importance of choosing low-fat foods, not high-fat foods.

We have an expert-written solution to this problem!
The nurse is admitting a client to the unit after fetal death was confirmed by ultrasound. While initiating intravenous (IV) therapy the nurse notes blood continually oozing from the puncture site. What is the nurse’s next action?
1
Restarting the line distal to the initial site
2
Informing the primary healthcare provider of this finding
3
Starting the prescribed infusion of oxytocin
4
Placing an oxygen mask on the client and setting the flow rate at 8 L/min
2
Oozing from a venipuncture site is a sign that disseminated intravascular coagulopathy (DIC) is developing. This pathologic form of clotting causes widespread bleeding and clotting. It is never a primary diagnosis; it always results from some problem that has triggered the clotting cascade. The primary healthcare provider must be informed immediately, because this diagnosis will change the client’s plan of care. There is no information indicating the need for a different IV site. Also, subsequent venipunctures must be proximal and not distal to previous sites. Delivery will likely be managed initially with oxytocin; however, this is not the first action to be taken in regard to an oozing IV site. Generally oxygen is started for signs of hypoxia, fetal or maternal; because there is no fetal indication for oxygen and no information in the stem indicating maternal hypoxia, application of oxygen is not the next intervention.

A client whose membranes have ruptured is admitted to the birthing unit. Her cervix is dilated 3 cm and 50% effaced. The amniotic fluid is clear, and the fetal heart rate is stable. Which outcome does the nurse anticipate?
1
A prolonged second stage of labor
2
A difficult birth resulting from delayed effacement
3
Birth of the fetus within a day
4
The stimulation of labor with an oxytocin infusion
3
In an uneventful full-term pregnancy, birth usually occurs within 24 hours after membranes have ruptured. If the birth does not occur within this timeframe, both the mother and fetus will be at risk for sepsis. At this juncture the provider will likely induce labor. There is no relationship between ruptured membranes and the second stage of labor. There are no data to indicate that effacement is delayed. Although it may be done eventually, it is too early to anticipate that labor will be stimulated.

The nurse is applying skin ointment to acne lesions for a client who has recently reached puberty. What does the nurse anticipate as the cause for the extensive acne?
1
Thelarche as the first indication of puberty
2
Influence of gonadal and adrenal androgens
3
Increased subcutaneous fat before a skeletal growth spurt
4
Highly active sebaceous glands in “flush areas” of the body
4
Acne is a common skin problem seen in clients of pubertal age. The pathogenesis of pubertal acne is linked with the hormonal influences on the skin and its appendages. The “flush areas” of the body include the face and neck, shoulders, back, and chest. The sebaceous glands in these “flush areas” become very active and secrete excessive sebum under the influence of hormones at the time of puberty. This hyperactivity of the sebaceous glands results in the development of puberty-related acne. In boys, just before the skeletal growth spurt, there is a transient increase in subcutaneous fat. Girls in whom thelarche is the first indication of puberty exhibit an early onset of menstruation and higher body fat. The hair at sites related to secondary sex characteristics becomes coarser, darker, and longer because of the influence of gonadal and adrenal androgens.

We have an expert-written solution to this problem!
After assessing a 4-day-old newborn, the nurse anticipates that the newborn has impaired vision. Which finding supports the nurse’s conclusion?
1
The newborn blinks in response to light.
2
The newborn has visual acuity of 20/100.
3
The newborn does not produce tears while crying.
4
The newborn has no corneal reflex after a light touch.
4
Corneal reflex in infants is activated by a light touch. Therefore the nurse anticipates that the newborn has impaired vision when there is no corneal reflex after a light touch. Visual acuity of 20/100 in a newborn is a normal finding; it does not indicate that the newborn has impaired vision. A positive blink reflex in response to stimulus is a normal finding; it does not indicate that the newborn has impaired vision. The tear glands begin functioning 2 to 4 weeks after birth. The absence of tears while crying is a normal finding in newborns and does not indicate that the newborn has impaired vision.

Which solid food can be given to a 6-month-old infant?
1
Raw pieces of fruit
2
Crackers or Zwieback
3
Firmly cooked vegetable
4
Well-cooked table foods
2
A 6-month-old infant is able to digest crackers or Zwieback. A 6-month-old infant cannot digest raw pieces of fruit or firmly cooked vegetables. Well-cooked table foods can be effectively digested by a 12-month-old child, but not a 6-month-old infant.

We have an expert-written solution to this problem!
The parents of a 2-year-old child tell the nurse that they are having difficulty disciplining their child. What is the most appropriate response by the nurse?
1
“This is a difficult age that your child is going through right now.”
2
“Tell me more about your difficulty. I’m not sure what you mean.”
3
“It’s important to be consistent with toddlers when they need discipline.”
4
“I totally understand what you mean. That’s why they call it the terrible twos.”
2
The nurse should obtain clarification of the parent’s specific concerns regarding the child’s behavior. Although many parents see this age as a difficult time, simply calling it that cuts off communication, and further communication should be encouraged. Emphasizing the need to be consistent assumes that the parents have been inconsistent; the nurse needs more information. Simply stating that the child is in the terrible twos is inappropriate because the nurse is explaining a developmental factor without exploring what the parent means.

A nurse educates a group of mothers about the nutritional needs of preschoolers. What statement made by a mother indicates the need for further teaching?
1
“I should give my child a variety of foods.”
2
“I should increase the protein in my child’s diet.”
3
“I should be concerned about my child’s weight gain.”
4
“I should limit my child’s calorie intake to 1,200 per day.”
4
The average daily intake of calories by a preschooler is 1,800 calories. The mother should include a variety of foods in the child’s diet for proper growth and development. The mother should increase the protein intake of a preschooler for healthy growth. The mother should be concerned about the amount of the food consumed by the child because overeating can lead to childhood obesity.

Which cognitive development is seen in early adolescence?
1
Having established abstract thoughts
2
Developing capacity for abstract thinking
3
Exploring the ability to attract opposite sex
4
Exploring a newfound ability for limited abstract thoughts
4
Exploring a newfound ability for limited abstract thoughts is observed in early adolescence. Formal operational thought includes being able to think in abstract terms, think about possibilities, and think through hypotheses, which are characteristic features of cognitive development during adolescence. Middle and late adolescents have abstract thoughts and develop the capacity for abstract thinking. Exploring the ability to attract opposite sex is part of adolescence, but it is not a cognitive development.

A couple, married for 5 years, wants to start a family. When speaking with the nurse the husband says, “Well, I guess we’re going to have to jump into bed three or four times a day, every day, until it works.” What is the nurse’s best response?
1
Telling them to continue intercourse as usual until conception occurs
2
Instructing them in the frequency and timing of intercourse to promote conception
3
Discouraging this because sperm production decreases with frequent sexual intercourse
4
Agreeing that the frequency of intercourse must increase but twice daily is sufficient to promote conception
2
Instructing the couple to have intercourse four times a week with at least 12 to 24 hours between ejacul*tions will increase the chance of conception and will correct the client’s misconceptions in a nonthreatening manner. Telling the couple to continue intercourse as usual until conception occurs is too vague; specific instructions should be given in a nonthreatening manner. To openly discourage the partner without providing instruction may be harmful to the relationship between the couple themselves or the couple and the nurse. Twice-daily intercourse is too frequent because it does not allow enough time between ejacul*tions for adequate spermatogenesis.

How should a nurse control the negative behavior of a toddler?
1
By allowing for more openness
2
By using invasive methods
3
By limiting his or her opportunities to say no
4
By giving the toddler choices
3
The nurse reduces the likelihood of a toddler engaging in negative behavior by reducing the child’s opportunities to give “no” responses. The nurse should avoid too much openness with the toddler. The nurse should not use invasive methods to control negative behavior because this may cause fear. The nurse should not give the toddler choices. For example, the nurse should not ask, “Do you want to take the medicine?” Instead, the nurse should say, “It’s time to take the medicine.”

We have an expert-written solution to this problem!
A registered nurse is evaluating the statements of a nursing student providing instructions to the partner of a client with Ebola. Which instruction given by the nursing student needs correction?
1
“You should avoid direct contact with your partner’s saliva.”
2
“You should be careful because the Ebola virus spreads through the air.”
3
“You should avoid having sex with your partner for 3 months even after the recovery of your partner.”
4
“You should immediately report symptoms of fever, headache, and vomiting.”
2
Ebola disease is caused by the Ebola virus, which does not spread through the air, water, or food. Ebola spreads through bodily fluids. A client with Ebola may have the Ebola virus in the sem*n for 3 months even after recovery. Early symptoms of Ebola disease are fever, headache, and vomiting.

A nurse is teaching a parent about the immunization schedule for babies. Between which months of age should the measles vaccine be given?
1
2 and 5
2
6 and 8
3
9 and 11
4
12 and 15
4
Between 12 and 15 months is the optimal age because maternal antibodies to measles are no longer present to block the formation of the child’s own antibodies. The measles vaccine is not given between 2 and 5 months, between 6 and 8 months, or between 9 and 11 months because of the questionable efficacy of the vaccination, due to the presence of maternal antibodies.

The nurse assesses and reports a disproportionate head-to-chest ratio for a 6-month-old infant. Which information should the nurse prepare to share with the infant’s parents as the most likely cause of this assessment data?
1
“Your infant most likely has an altered chest shape.”
2
“Your infant is likely experiencing abnormal head growth.”
3
“Your infant will most likely be diagnosed with brain cancer.”
4
“Your infant most likely had inaccurate measurements during the last visit.”
2
A disproportionate head-to-chest ratio during infancy is most likely caused by abnormal head growth. While an altered chest shape may cause the disproportionate measurements, this is not the most likely cause. Telling the parents that their infant will most likely be diagnosed with brain cancer is inaccurate and not therapeutic. The data obtained to determine the disproportionate measures was obtained during the current visit; previous measurements will not be taken into account in this circ*mstance.

We have an expert-written solution to this problem!
A nurse is advising a parent to teach the child impulse control and cooperative behaviors to avoid the risks of altered growth and development. In which stage does the nurse teach the parent about this, according to Erikson’s theory of psychosocial development?
1
Initiative versus Guilt
2
Intimacy versus Isolation
3
Autonomy versus Sense of Shame and Doubt
4
Generativity versus Self-Absorption and Stagnation
1
In the Initiative versus Guilt stage, the nurse advises the parent to teach the child impulse control and cooperative behaviors. This helps to avoid risks of altered growth and development. In the Intimacy versus Isolation stage, the nurse tries to understand the needs of a young adult. The Autonomy versus Sense of Shame and Doubt stage models empathetic guidance that provides support for and understanding the challenges of this stage. In the Generativity versus Self-Absorption and Stagnation stage, the nurse assists physically ill adults in choosing creative ways to foster social development.

A nurse is assessing the growth and development of a 3-year-old child. What does the nurse expect the child to be able to do?
1
Hop on one foot.
2
Button a sweater.
3
Cup the hands to catch a ball.
4
Go upstairs while alternating the feet.
4
The average 3-year-old child has the physical ability to climb the stairs with alternating feet, but may still use both feet on a step to go down the stairs. Cupping the hands to catch an object is an accomplishment of a 5-year-old child. Hopping on one foot is usually not accomplished until a child is 5 years old. A 3-year-old child may be able to self-dress partially, but buttoning is more easily accomplished by a 5-year-old.

The school nurse presents a program on Reye syndrome to the Parent-Teacher Association. After the program the nurse talks with a group of parents. Which statement by a mother indicates the need for additional education on Reye syndrome?
1
“Aspirin should be avoided in children with viral diseases.”
2
“It’s OK for me to give my kids Tylenol if they run a fever.”
3
“I should watch for my child’s skin to turn yellow during the recovery from varicella.”
4
“I need to seek medical help right away if my child starts vomiting profusely during the recovery period of a viral illness.”
3
Liver involvement is a component of Reye syndrome, but the skin will not turn yellow. Classic early symptoms are vomiting and confusion during the recovery from a viral illness. The mother needs to have the symptoms of Reye syndrome reviewed because her response contains incorrect information. There is an association between aspirin use for viral diseases and the development of Reye syndrome. Acetaminophen (Tylenol) and ibuprofen (Motrin) may be used to treat fevers because there has been no association between these drugs and the development of Reye syndrome. Profuse vomiting is an early sign of Reye syndrome. The child progresses quickly through the five stages of Reye syndrome, and early recognition is important.

A client who retired a year ago tells the nurse in the community health center, “I don’t have any hobbies or interests, and since I retired I feel useless and unneeded.” According to Erikson’s developmental theory, with which developmental conflict is the client faced?
1
Initiative versus guilt
2
Integrity versus despair
3
Intimacy versus isolation
4
Identity versus role confusion
2
Integrity versus despair is the task of the older adult; this client has not adapted to triumphs and disappointments, so there is no acceptance of what life is and was; this results in feelings of despair and disgust. Initiative versus guilt is the task of the preschool period. Intimacy versus isolation is the task of the young adult. Identity versus role confusion is the task of the adolescent.

A client tells a nurse in the prenatal clinic that she has vagin*l staining but no pain. Her history reveals amenorrhea for the last 2 months and pregnancy confirmation after her first missed period. What type of abortion is suspected?
1
Missed
2
Inevitable
3
Threatened
4
Incomplete
3
Spotting in the first trimester may indicate that the client is having a threatened abortion; any client with the possibility of hemorrhage should not be left alone, so her admission to the hospital helps ensure her safety. A missed abortion may not cause any outward signs or symptoms, except that the signs of pregnancy disappear. An inevitable abortion can be confirmed only if vagin*l examination reveals cervical dilation. With an incomplete abortion some, but not all, of the products of conception have been expelled.

A nurse educates a parent about Erikson’s theory of child development. At what age would the nurse advise the parent to begin instilling a sense of trust in his or her child?
1
Birth to 1 year
2
1 to 3 years
3
3 to 6 years
4
6 to 11 years
1
According to Erikson’s theory, the period between birth and 1 year is the most appropriate period for instilling a sense of trust in a child. The period between 1 and 3 years is the stage during which a child develops self-control and willpower. Establishing a basic sense of trust is essential for the development of a healthy personality. The period between 3 and 6 years is the best time for a child to achieve self-control and willpower. Between 6 and 11 years, a child is eager to learn socially productive skills and tools.

A nurse plans care of 4-year-old hospitalized children on the basis of their developmental level. What is the major vulnerability of children this age?
1
Separation anxiety
2
Altered family roles
3
Intrusive procedures
4
Enforced dependency
3
Preschool children fear procedures that intrude on their body integrity. Separation anxiety is more common in toddlers. Preschool children are too young to understand the concept of altered family roles; this is a fear that adults experience. Enforced dependency is a fear that school-age and adolescent children experience.

Which subdimension would form a part for the caring process “doing for” according to the Swanson’s theory of caring? Select all that apply.
1
Focusing
2
Protecting
3
Comforting
4
Seeking cues
5
Generating alternatives
23
Protecting and comforting are the subdimensions of “doing for” according to the Swanson’s theory of caring. Focusing is the subdimension of the caring process “enabling.” The subdimensions of “seeking cues” and “generating alternatives” are appropriate for the caring process “knowing.”

We have an expert-written solution to this problem!
An adolescent who has had type 1 diabetes for 5 years stops adhering to the therapeutic regimen. In light of the client’s developmental level, what does the nurse conclude that the behavior is a reflection of?
1
Need for attention
2
Struggle for identity
3
Denial of the diabetes
4
Regression related to the illness
2
Striving to attain identity and independence are tasks of the adolescent, and rebellion against established norms may be exhibited. Nonadherence to a regimen is not a bid for attention; rather, it is an attempt to establish an identity, which is a developmental task of adolescence. Although the adolescent may be using denial, denial is not developmentally related to adolescence. Noncompliance is not a sign of regression; it is an attempt to attain identity through rebellion against established norms.

The community health care nurse uses emails to remind the community about vaccines and regular diabetic checkups. Which Quality and Safety Education for Nurses (QSEN) competency does the nurse address?
1
Safety
2
Informatics
3
Quality improvement
4
Evidence-based practice
2
The QSEN competency informatics involves the use of information and technology to communicate, manage knowledge, and support decision-making. The community health care nurse uses computers to send emails about health care information to the community members. The nurse applies the safety competency by minimizing the risk of harm to the client and health care providers. The nurse conforms to the quality improvement competency while using data to monitor the outcomes of client care processes. Evidence-based practice is facilitated when the nurse reads nursing journals for current trends and practices in health care.

A nurse is teaching a class for staff members working in a group home about the cognitive development of children with cognitive impairments. What concept can these children probably learn the fastest?
1
Love versus hate
2
Life versus death
3
Large versus small
4
Right versus wrong
3
Children who are cognitively impaired can learn concrete concepts faster than they can learn abstract concepts. Love versus hate is an abstract concept that children begin to learn between the ages of 7 and 11 years. Life versus death is an abstract concept that children begin to learn between the ages of 7 and 11 years. Right versus wrong is an abstract concept that children begin to learn between the ages of 7 and 11 years.

The nurse finds that a child has developed the qualities of independence, self-control, and self-governance. Which type of development does the nurse anticipate in the toddler?
1
Social development
2
Spiritual development
3
Biologic development
4
Psychosocial development
4
Psychosocial development is indicated by the child’s awareness of the relationship between himself or herself and his or her environment. When the child develops independence, self-control, and self-governance, it indicates that the child has acquired a sense of autonomy, which is a component of psychosocial development. Social development is the ability to establish and maintain rewarding relationships with others. Spiritual development in children is heavily influenced by family members and others close to them; it is not specifically associated with the development of autonomy. Biologic development is the series of changes that occur at different stages of growth.

What are the goals of the Childhood Immunization Initiative Program? Select all that apply.
1
To reduce the costs of a vaccine
2
To involve more federal agencies
3
To improve vaccine delivery systems
4
To evaluate the impact of national immunization policies and practices
5
To prevent the occurrence of indigenous infections such as diphtheria, rubella, and measles
1235
The goal of the Childhood Immunization Initiative Program is to reduce the cost of vaccines to make them affordable. Federal agencies are involved in providing vaccines to underserved populations. Childhood Immunization Initiative Program also aims to improve vaccine delivery systems and preventing infections of diphtheria, rubella, and measles. The health department sends a report on the evaluation of national immunization policies to the Centers for Disease Control and Prevention; this action is unrelated to the immunization program.

We have an expert-written solution to this problem!
A nurse who is observing a sleeping newborn at 2 hours of age identifies periods of irregular breathing and occasional twitching movements of the arms and legs. The neonate’s heart rate is 150 beats/min; the respiratory rate is 50 breaths/min; and the glucose strip reading is 60 mg/dL (3.3 mmol/L). What does the nurse conclude that these findings indicate?
1
Hypoglycemia
2
Seizure activity
3
Expected adaptations
4
Respiratory distress syndrome
3
During periods of active or irregular sleep, healthy newborns have some twitching movements and irregular respirations; the heart rate, respirations, and blood glucose level are within expected limits. Hypoglycemia in newborns is characterized by a blood glucose level below 30 mg/dL (1.7 mmol/L). Twitching is a common finding in healthy neonates and does not indicate seizure activity; it often occurs with crying or stimulation. There are no signs of respiratory distress syndrome. The newborn respiratory rate ranges between 30 and 60 breaths/min; irregular breathing is expected.

We have an expert-written solution to this problem!
A couple interested in delaying the start of a family discuss the various methods of family planning. Together they decide to use the basal body temperature method. The nurse explains that the fertile period surrounding ovulation lasts from when to when?
1
12 hours before to 24 hours after ovulation
2
72 hours before to 24 hours after ovulation
3
72 to 80 hours before to 72 hours after ovulation
4
24 to 48 hours before to 48 hours after ovulation
2
The ovum is fertilizable for 12 to 24 hours, and sperm remain motile for about 72 hours. Therefore the period of fertility is a total of 96 hours (72 hours before ovulation plus 24 hours after ovulation). The fertility period before ovulation is longer than 12 hours. Seventy-two to 80 hours before to 72 hours after ovulation is too long before and after ovulation. The period of fertility is longer than 48 hours before ovulation and shorter than 48 hours after ovulation.

What is the difference between evidence-based practice and quality improvement?
1
Evidence-based practice is a part of a regular clinical practice.
2
Evidence-based practice aims to improve client care and outcomes.
3
Evidence-based practice focuses on the implementation of evidence already known into practice.
4
Evidence-based practice consists of internal funding and can be conducted by practicing nurses.
3
Evidence-based practice focuses on the implementation of evidence already known into practice whereas quality improvement measures the effect of changing practices on a specific population. Both evidence-based practice and quality improvement are a part of regular clinical practice. Both aim at to improve client care and outcomes. Both evidence-based practice and quality improvement consist of internal funding and can be conducted by practicing nurses and other healthcare professionals.

A nurse is teaching a client about a sodium-restricted diet. Which foods should the nurse encourage the client to consume? Select all that apply.
1
Fruits
2
Cold cuts
3
Condiments
4
Fresh vegetables
5
Processed foods
14
Most fruits and vegetables are allowed on a sodium-restricted diet. Cold cuts are processed meats that are high in sodium. Condiments such as ketchup are high in sodium. Most processed foods have sodium added to enhance taste and help preserve food.

What instructions should the nurse give to an adolescent to prevent sexually transmitted infections? Select all that apply.
1
“Remember to use condoms properly.”
2
“Abstain from any kind of sexual activity.”
3
“Make sure you are up-to-date with your vaccinations.”
4
“Have sexual contact only if you and your partner are monogamous.”
5
“Remember to have regular screenings for sexually transmitted disease.”
134
The safe use of condoms helps to avoid contact with body fluids and helps prevent sexually transmitted infections. Getting updated with vaccinations helps prevent vaccine-preventable sexually transmitted infections. Monogamous partners have a low risk of contracting sexually transmitted infections. Abstaining from sexual activity is not a practical approach. Regular screening for sexually transmitted infections helps to detect a disease at an early stage, but does not prevent contraction of the disease.

We have an expert-written solution to this problem!
During assessment of a newborn in the nursery, the nurse notices a large, dark pigmentation over the buttocks of one of the infants. What is the most important intervention?
1
Checking the infant’s platelet count
2
Calling social services about potential abuse
3
Checking the medical record regarding this finding at birth
4
Notifying the provider that the infant sustained trauma at birth
3
Large dark areas of pigmentation over the buttocks are a common birth defect known as Mongolian spots. These hyperpigmented areas can resemble bruising but lessen over time and usually disappear by the time the child reaches school age. The nurse taking care of this infant should check the medical record for documentation of this finding at birth in the medical record. Mongolian spots are not caused by bleeding, trauma, or abuse.

Which should the nurse share with the parents of a toddler-age client regarding a predominant change that occurs to the gastrointestinal system during this stage of development?
1
Food is rejected due to texture
2
Voluntary control of elimination
3
Capacity of the stomach decreases
4
Acidity of gastric contents increases
2
The most predominant change to the gastrointestinal system during the toddler stage of development is voluntary control of elimination; this is important to share with the parents because this is the stage of development where toilet training occurs. While food is rejected due to texture, this is not due to a change in the gastrointestinal system but due to maturation of the senses. The capacity of the stomach increases allowing for 3 meals per day. While the acidity of the gastric contents does increase, this is not a predominant change the nurse would include in the teaching session because it does not affect day to day care.

We have an expert-written solution to this problem!
New parents are asked to sign the consent form for their son to be circumcised. They ask for the nurse’s opinion of the procedure. How should the nurse respond?
1
“Let’s talk about it, because there are advantages and disadvantages.”
2
“It’s a safe procedure, and it’s best for male infants to be circumcised.”
3
“Although it may be a somewhat painful experience for the baby, I would allow it if I were you.”
4
“You should talk to the primary health care provider about this if you have any questions.”
1
Suggesting that they talk about it because there are advantages and disadvantages permits exploration of the parents’ wishes and helps them make their own decision. Stating that the parents should talk to the primary health care provider if they have any questions blocks further discussion; the nurse can answer some of the questions and refer those that cannot be answered to the primary health care provider. Stating that circumcision is a safe procedure and that it is best for male infants to be circumcised is a value judgment that denies the parents’ right to decide. Stating, “Although it may be a somewhat painful experience for the baby, I would allow it if I were you” might frighten the parents, and it denies them their power of decision.

We have an expert-written solution to this problem!
The registered nurse is teaching the nursing student about the realms of family life. Which component does the registered nurse include while teaching about integrity processes?
1
Family rituals
2
Family relationships
3
Family life stressors and daily hassles
4
Family care takings and responsibilities
1
The family health system includes five realms, or processes, of family life. These realms are interactive, developmental, coping, integrity, and health. This approach is a method for family assessment used to determine areas of concern and strengths and to help develop an effective care plan. The component of integrity includes family rituals. Family relationships are a part of interactive processes. Family life stressors and daily hassles are considered components of coping processes. Health processes include family care takings and responsibilities.

An older adult tells the nurse, “I regret so many of the choices I’ve made during my life.” Which of Erikson’s developmental conflicts has the client probably failed to accomplish?
1
Ego integrity versus despair
2
Identity versus role confusion
3
Generativity versus stagnation
4
Autonomy versus shame and doubt
1
The sense of ego integrity comes from satisfaction with life and acceptance of what has been and what is. Despair reflects guilt or remorse over what might have been. During puberty adolescents attempt to find themselves and integrate their own values with those of society; an inability to solve conflict results in confusion and hinders mastery of future roles. During early and middle adulthood the individual is concerned with the ability to produce and to care for that which is produced or created; failure during this stage leads to self-absorption or stagnation. Autonomy, the ability to control the body and environment, is developed during the toddler period; doubt may result when the child is made to feel ashamed or embarrassed.

A nurse teaches a group of teenage girls about puberty. What information should the nurse refrain from providing?
1
“Your voice will change.”
2
“Your shoulders will increase in width.”
3
“You will grow until you are 16 to 17 years of age.”
4
“Your body will gain height and weight disproportionately.”
4
The nurse should refrain from telling teenage girls that the rate of height and weight gain will not be equal. This rate is proportional. During puberty, a girl’s voice changes, and there is an increase in the width of shoulders and hips. Girls grow until the age of 16 to 17 years of age.

We have an expert-written solution to this problem!
A school-aged child is admitted to the pediatric unit with hypertensive acute glomerulonephritis. In addition to hydralazine, what medication does the nurse anticipate will be prescribed initially?
1
Digoxin
2
Furosemide
3
Alprazolam
4
Phenytoin
2
Furosemide is a loop diuretic that is recommended for the treatment of acute glomerulonephritis; it promotes the excretion of fluid and thus limits fluid retention. Digoxin is not used because there is no cardiac involvement. An anxiolytic is unnecessary. Phenytoin may be used only if hypertensive encephalopathy causes seizures.

A client has been taking the prescribed dose of clozapine. The nurse will assess the client for which life-threatening side effect of this drug?
1
Polycythemia
2
Agranulocytosis
3
Hypertensive crisis
4
Pseudoparkinsonism
2
Agranulocytosis occurs in 1% to 2% of clients receiving clozapine and is potentially fatal; weekly blood counts are necessary. Polycythemia is not a side effect of clozapine. Clozapine may cause hypotension; hypertensive crisis is a side effect of monoamine oxidase inhibitors. Pseudoparkinsonism may occur, but it can be managed with anticholinergic medications.

A nurse is taking the health history of a client who is to have surgery in one week. The nurse identifies that the client is taking ibuprofen for discomfort associated with osteoarthritis and notifies the healthcare provider. Which drug does the nurse expect will most likely be prescribed instead of the ibuprofen?
1
Naproxen
2
Aspirin
3
Ketorolac
4
Acetaminophen
4
Acetaminophen is a nonopioid analgesic that inhibits prostaglandins, which serve as mediators for pain; it does not affect platelet function. Naproxen, aspirin, and ketorolac are nonselective nonsteroidal antiinflammatory drugs (NSAIDs) that are contraindicated for clients undergoing surgery; nonselective NSAIDs have an inhibitory effect on thromboxane, a strong aggregating agent, and can result in bleeding.

Methylphenidate is prescribed to treat a 7-year-old child’s attention deficit-hyperactivity disorder (ADHD). The nurse understands that methylphenidate is used in the treatment of this disorder in children because it has what effect?
1
Diuretic effect
2
Synergistic effect
3
Paradoxical effect
4
Hypotensive effect
3
Methylphenidate, a stimulant, has an opposite effect on hyperactive children; the reason for this action is as yet totally unexplained. Although methylphenidate has a hypotensive effect, this is not why it is given to hyperactive children. Methylphenidate does not induce diuretic or synergistic effects.

A client with leukemia who is receiving a chemotherapeutic regimen that includes vincristine reports numbness and loss of feeling in the legs below the knees. The client asks the nurse about what is causing these problems. What fact forms the basis for the nurse’s response?
1
Enlarged lymph nodes in the groin related to the cancer may cause these symptoms.
2
Most chemotherapeutic regimens do not affect the nervous or peripheral vascular system.
3
Vascular occlusion may be the cause, and immediate medical evaluation is indicated.
4
Peripheral neuropathies can result from chemotherapy and usually are reversible if promptly treated.
4
Muscle weakness, tingling, and numbness are related to drugs like vincristine; neuropathies usually are transient if the drug is stopped or reduced. Nodal enlargement produces vascular rather than neural side effects. Most chemotherapeutic regimens do affect the nervous or peripheral vascular system; neuropathies and peripheral vascular adaptations are potential side effects of chemotherapy. Tingling and numbness are characteristic of neuropathy, not vascular occlusion.

A nurse is caring for a client who is receiving serum albumin. What indicates that the albumin is effective?
1
Improved clotting of blood
2
Formation of red blood cells
3
Activation of white blood cells (WBCs)4
Effective cardiac output
4
Serum albumin, a protein, establishes the plasma colloid osmotic (oncotic) pressure because of its high molecular weight and size. Indicators of adequate osmotic pressure include an effective cardiac output. Blood clotting involves blood protein fractions other than albumin; for example, prothrombin and fibrinogen are within the alpha- and beta-globulin fractions. Red blood cell formation (erythropoiesis) occurs in red marrow and can be related to albumin only indirectly; albumin is the blood transport protein for thyroxine, which stimulates metabolism in all cells, including those in red bone marrow. Albumin does not activate WBCs; WBCs are activated by antigens and substances released from damaged or diseased cells.
44%of students nationwide answered this question

We have an expert-written solution to this problem!
A nurse is caring for a client who is addicted to opioids and who has undergone major surgery. The client is receiving methadone. What is the purpose of this medication?
1
Allows symptom-free termination of opioid addiction
2
Switches the user from illicit opioid use to use of a legal drug
3
Provides postoperative pain control without causing opioid dependence
4
Counteracts the depressive effects of long-term opioid use on thoracic muscles
2
Methadone may be dispensed legally; the strength of this drug is controlled and remains constant from dose to dose, unlike illicit drugs. Methadone is used in the medically supervised withdrawal period to treat physical dependence on opiates; methadone therapy substitutes a legal for an illegal drug. Methadone may be administered over the long term to replace illegal opioid use. If methadone treatment is abruptly stopped, there will be withdrawal symptoms. Methadone is a synthetic opioid and can cause dependence; it is used in the treatment of heroin addiction, but may be prescribed for people who have chronic pain syndromes. It is not used for acute postoperative pain. Methadone is not known to counteract the depressive effects of long-term opioid use on thoracic muscles.

We have an expert-written solution to this problem!
A 2-month-old infant with the diagnosis of heart failure is discharged with a prescription for oral digoxin 0.05 mg every 12 hours. The bottle of digoxin is labeled “0.05 mg/mL.” Which item should the nurse teach the mother to use when administering the medication?
1
Nipple
2
Calibrated syringe
3
Plastic measuring spoon
4
Bottle containing an ounce of water
2
A calibrated syringe or dropper provides the most accurate measurement of the medication. Using a nipple or spoon is not an accurate way to measure medication. If the dose of medication is diluted and the infant does not drink the entire ounce, the resulting dose will be insufficient.

The nurse is explaining insulin needs to a client with gestational diabetes who is in her second trimester of pregnancy. Which information should the nurse give to this client?
1
Insulin needs will increase during the second trimester.
2
Insulin needs will decrease during the second trimester.
3
Insulin needs will not change during the second trimester.
4
Insulin will be switched to an oral antidiabetic medication during the second trimester.
1
The second trimester of pregnancy exerts a diabetogenic effect on the maternal metabolic status. Major hormonal changes result in decreased tolerance of glucose, increased insulin resistance, decreased hepatic glycogen stores, and increased hepatic production of glucose. Increasing levels of human chorionic somatomammotropin, estrogen, progesterone, prolactin, cortisol, and insulinase increase insulin resistance through their actions as insulin antagonists. Insulin resistance is a glucose-sparing mechanism that ensures an abundant supply of glucose for the fetus. Maternal insulin requirements gradually increase from about 18 to 24 weeks of gestation to about 36 weeks’ gestation. The use of oral antidiabetes agents is currently not recommended by the American Diabetes Association for use during pregnancy.

We have an expert-written solution to this problem!
A woman reports irregular menses, back pain, and weight gain. Upon diagnosis, the woman has low levels of estrogen and progesterone. Which drug may be prescribed if the woman wants to conceive?
1
Raloxifene
2
Methergine
3
Clomiphene
4
Dinoprostone
3
Clomiphene is a fertility drug that can cause ovulation by stimulating the production of the gonadotropin hormone. Raloxifene is used to prevent postmenopausal osteoporosis. Methergine is used to reduce postpartum uterine hemorrhage. Dinoprostone may cause a termination of a pregnancy.

A client was administered oxytocin to induce labor, but the client was not responsive to the treatment. Which oral uterine stimulant is the next choice in this condition?
1
Misoprostol
2
Mifepristone
3
Dinoprostone
4
Methylergonovine
1
Misoprostol is an oral uterine stimulant that helps in inducing labor when the uterus is insensitive to oxytocin. Mifepristone can induce elective abortion. Dinoprostone induces the uterine contractions but is only available for vagin*l use. Methylergonovine helps to prevent postpartum hemorrhage.

Which drug does the nurse recognize as an effective mood-stabilizing drug used in clients with bipolar disorder and in the acute treatment of mania and prevention of recurrent mania and depressive episodes?
1
Doxepin
2
Clozapine
3
Amitriptyline
4
Lithium carbonate
4
Lithium carbonate is often the first choice of treatment, once primary acute mania has been diagnosed, to calm acute manic symptoms and relieve recurrent mania. Doxepin and amitriptyline are antidepressants used to treat depression but not mania. Clozapine is an antipsychotic medication used to control hallucinations and delusions in patients with psychosis but is not a first-line drug because of its side effects, which include seizures and significant weight gain.

We have an expert-written solution to this problem!

HESI CAT EXAM 700 QUESTIONS AND CORRECT ANSWERS - Learnexams (2024)

References

Top Articles
Live Scan 90003
Money blog: House prices hit two-year high - see the average cost in your region
Chs.mywork
Golden Abyss - Chapter 5 - Lunar_Angel
Duralast Gold Cv Axle
Top 11 Best Bloxburg House Ideas in Roblox - NeuralGamer
Television Archive News Search Service
What spices do Germans cook with?
Air Canada bullish about its prospects as recovery gains steam
Bucks County Job Requisitions
Wild Smile Stapleton
Arrests reported by Yuba County Sheriff
Braums Pay Per Hour
PGA of America leaving Palm Beach Gardens for Frisco, Texas
Tight Tiny Teen Scouts 5
R Tiktoksweets
Nexus Crossword Puzzle Solver
Cooktopcove Com
Roster Resource Orioles
Nurse Logic 2.0 Testing And Remediation Advanced Test
Healthier Homes | Coronavirus Protocol | Stanley Steemer - Stanley Steemer | The Steem Team
Football - 2024/2025 Women’s Super League: Preview, schedule and how to watch
Обзор Joxi: Что это такое? Отзывы, аналоги, сайт и инструкции | APS
Fiona Shaw on Ireland: ‘It is one of the most successful countries in the world. It wasn’t when I left it’
Arrest Gif
Craigslist Rentals Coquille Oregon
Claio Rotisserie Menu
Valley Craigslist
Robert A McDougal: XPP Tutorial
Ghid depunere declarație unică
Davita Salary
Alima Becker
Rund um die SIM-Karte | ALDI TALK
Mumu Player Pokemon Go
404-459-1280
Workday Latech Edu
Pillowtalk Podcast Interview Turns Into 3Some
Synchrony Manage Account
“Los nuevos desafíos socioculturales” Identidad, Educación, Mujeres Científicas, Política y Sustentabilidad
Gun Mayhem Watchdocumentaries
Improving curriculum alignment and achieving learning goals by making the curriculum visible | Semantic Scholar
Vintage Stock Edmond Ok
8776725837
Babykeilani
Gli italiani buttano sempre più cibo, quasi 7 etti a settimana (a testa)
Walmart Front Door Wreaths
Star Sessions Snapcamz
Kushfly Promo Code
Meee Ruh
Makemkv Key April 2023
A Snowy Day In Oakland Showtimes Near Maya Pittsburg Cinemas
OSF OnCall Urgent Care treats minor illnesses and injuries
Latest Posts
Article information

Author: Otha Schamberger

Last Updated:

Views: 6495

Rating: 4.4 / 5 (75 voted)

Reviews: 90% of readers found this page helpful

Author information

Name: Otha Schamberger

Birthday: 1999-08-15

Address: Suite 490 606 Hammes Ferry, Carterhaven, IL 62290

Phone: +8557035444877

Job: Forward IT Agent

Hobby: Fishing, Flying, Jewelry making, Digital arts, Sand art, Parkour, tabletop games

Introduction: My name is Otha Schamberger, I am a vast, good, healthy, cheerful, energetic, gorgeous, magnificent person who loves writing and wants to share my knowledge and understanding with you.